初等整数論の問題A

このエントリーをはてなブックマークに追加
1132人目の素数さん
初等数論の問題をいろいろ挙げてみましょう^^
(なるべくは初等数論の領域で解ける問題が望ましいです^^)
そもそも初等数論とは何かがわからない人は
次のURLにあるページのElementary number theoryの項を参照に^^
http://en.wikipedia.org/wiki/Number_theory
ただ曖昧な部分もあるので その点は融通を利かせましょう^^

なるべくそこらへんに転がっていない問題を挙げましょう^^
有名どころの問題の解答はネットで すぐみつかっちゃいます^^
たとえば、国際数学オリンピックの問題などはまさにその典型です^^
そのような有名すぎる問題を挙げても それは大抵は既に十分に議論されています^^
(しかしながら、何らかのオリジナル要素(改変,拡張etc)を加えたものなら歓迎^^)
また、解答が事前に用意されていない問題を挙げるときは そのことを明記しておきましょう^^

初等数論の問題を解くときはいくらかの道具を想定しておくのが基本です^^
以下のURLにたくさんの良く知られた道具(記号の説明含む)がありますので参考にどうぞ^^
http://www.artofproblemsolving.com/Forum/viewtopic.php?t=76610&ml=1
2132人目の素数さん:2010/11/16(火) 23:32:46
前スレで未解決の問題です^^

@ (m,n)=(2,3),(2,5),(3,2),(5,2)のとき、
x+y+z=0を満たす任意の実数x,y,zに対して
(x^{m+n}+y^{m+n}+z^{m+n})/(m+n)=(x^m+y^m+z^m)(x^n+y^n+z^n)/mn
が成り立つことが分かっている。他に整数解(m,n)は存在するか?全て求めよ

A 以下の条件を満たす正の整数(a,x,y,n,m)の組をすべて求めよ
a(x^n−x^m)=(ax^m−4)y^2
m≡n (mod 2) かつ ax は奇数

B 自然数Nの10進表示を a[1]a[2]..a[n] 等とする。
N = (a[1]+2)*(a[2]+2)*...*(a[n]+2)
を満たすような自然数Nを全て求めよ。

C フィボナッチ数列類似の多項式列{F(n)}を次のように定義する:
F(0) = 0, F(1) = 1, F(n+2) = F(n+1) + x^n F(n).
このとき、任意の自然数nに対して、
F(5^n) は 1 + x + x^2 + … + x^(5^n - 1) で割り切れることを示せ。

Df^3=g^2+1 を満たすf,g∈C(X)\C は存在するか。
(C(X)は複素係数の有理関数体のことです^^)
3132人目の素数さん:2010/11/16(火) 23:41:15
皆さんおやすみです^^

E123456a+234567b+345678c+456789d+567890e+678901f+789012g
+890123h+901234i=123456789 を満たすような
0以上の整数a,b,c,d,e,f,g,h,iを求めよ。

F自然数x_1, x_2, ..., x_nが
1/x_1+1/x_2+...+1/x_n=1 を満たすとき
x_1+x_2+...+x_n の最大値を求めよ。

Gx^2+ax+b, x^2+bx+a がともに整数根をもつような整数a,bの組を全て求めよ。

H2^x+3^yが19で割り切れるような正整数x,yの組は
どのくらいの割り合いで存在するか。

Ia^a*b^b=c^c を満たす整数a,b,c>1の組は無限に存在するか。

J次の条件を満たす素数p,qの組を全て求めよ。(New! 容易)
[条件]
p*q は三角数であり、p+q は平方数である。
4132人目の素数さん:2010/11/17(水) 01:24:56
猫に小判、まで読んだ。
5アゲ猫 ◆MuKUnGPXAY :2010/11/17(水) 02:58:40

6132人目の素数さん:2010/11/17(水) 05:35:05

    /\        /\
    /:::::::ヽ____/::::::::ヽ、
   丿 ::.__  .:::::::::::::   __ ::::ヽ_       ,. 、       /   /
  / /● ヽ_ヽv /: /●ヽ  ::::::ヽ    ,.〃´ヾ.、  /  /
 / / ̄ ̄√___丶  ̄ ̄\  ::::| / |l     ',  / /
 | .:::::::::: / / tーーー|ヽ     ..::::: ::|r'´  ||--‐r、 ',
 | .:::::.  ..: |    |ヽ   .,..ィ'´     l',  '.j '.    おれは猫さまだw
 | :::    | |⊂ニヽ| |  'r '´         ',.r '´ !|  \
 | :    | |  |:::T::::| !  l!     ....:.:.:.:.:.:ヽ、   ,l    \
 \:    ト--^^^^^┤   ゝ、.,_ ---‐‐‐----ゝ、ノ
7132人目の素数さん:2010/11/17(水) 08:21:16
321
8132人目の素数さん:2010/11/17(水) 09:58:01
三ケタ以内の素数の中で、もっともエロイ
自然数は何か。

また 三ケタ以内の自然数の中で
最もニートな自然数は何か。
9132人目の素数さん:2010/11/17(水) 11:42:57
127はエロいね
0721に似ている
210はまさにニートだね
10猫は口先だけ ◆MuKUnGPXAY :2010/11/17(水) 16:16:37

    /\        /\
    /:::::::ヽ____/::::::::ヽ、
   丿 ::.__  .:::::::::::::   __ ::::ヽ_       ,. 、       /   /
  / /● ヽ_ヽv /: /●ヽ  ::::::ヽ    ,.〃´ヾ.、  /  /
 / / ̄ ̄√___丶  ̄ ̄\  ::::| / |l     ',  / /
 | .:::::::::: / / tーーー|ヽ     ..::::: ::|r'´  ||--‐r、 ',
 | .:::::.  ..: |    |ヽ   .,..ィ'´     l',  '.j '.    ワシは口先猫やがなw
 | :::    | |⊂ニヽ| |  'r '´         ',.r '´ !|  \
 | :    | |  |:::T::::| !  l!     ....:.:.:.:.:.:ヽ、   ,l    \
 \:    ト--^^^^^┤   ゝ、.,_ ---‐‐‐----ゝ、ノ


11132人目の素数さん:2010/11/18(木) 18:33:54
Wilsonの定理: 
n≧3 とする。このとき、
 (n-1)!≡-1 mod n <=> n:素数
12132人目の素数さん:2010/11/20(土) 22:38:47
>>11

・pが素数のとき
 {1,2,…,p-1} は乗法群をなし、位数1の元は1, 位数2の元は p-1 のみ。
 位数3以上の元x では x≠x^(-1) だから {x,x^(-1)} をペアにすれば
 (n-1)! ≡ Π (位数2の元) = p-1 ≡ -1 (mod p),

・n=4 のとき
 (n-1)! = 3! = 6 ≡ 2 (mod 4)

・nが4より大きい合成数のとき、
 {1,2,……,n-1} 中に n|ab…, a≠b なる {a,b,…} があるから
 (n-1)! ≡ 0 (mod n)

 n=p^2 (p≧3) のとき {p, 2p}
 n=p^k (k≧3) のとき {p, p^(k-1)}
 n=Π[i=1,m] (p_i)^(e_i) のとき {(p_1)^(e_1), (p_2)^(e_2), ……, (p_m)^(e_m)}
13shin hattori:2010/11/21(日) 00:40:35
>>1
H
[2],[3]∈(Z/19Z)* は生成元であるから、
答えは 1/18 の割り合いで存在している。

G
x^2+ax+b, x^2+bx+a (|a|≧|b|)が共に整数根を持っていたと仮定する。
このとき、a^2-4b, b^2-4a は共に平方数(0も含む)であるといえる。
(逆に a^2-4b, b^2-4aが平方数ならば 問題の2つの多項式は整数根を持つ)
A=a^2-4b, B=b^2-4a とおく。a,bの符号に応じて場合をわける。

1) b=0 であるとき
A=a^2, B=-4a であるから、a= -m^2(m:非負整数)となる。

2) a,b>0 であるとき
a≦6ならば (a,b)=(4,4),(6,5)であることはすぐ確認できる。
a≧7ならば (a-3)^2<a^2-4a≦A<a^2 の成立がいえる。
これとAのパリティから A=(a-2)^2 の成立がいえる。
A=(a-2)^2 ⇔ a^2-4b=a^2-4a+4 ⇔ -4b=-4a+4 ⇔ b=a-1
このとき、B=b^2-4a=(a-1)^2-4a=a^2-6a+1 であるから、
(a-5)^2<B<(a-3)^2 がいえるので、B=(a-4)^2 となるが、
両辺のパリティが一致することはない。
14shin hattori:2010/11/21(日) 00:41:52

3) a>0 かつ b<0 であるとき
a^2<A≦a^2+4a<(a+2)^2 がいえるから
A=(a+1)^2 がいえるが 両辺のパリティは一致しない。

4) a<0 かつ b>0 であるとき
a≧-4 ならば (a,b)=(-2,1),(-3,2),(-4,3) がすぐ確認できる。
a<-4 ならば (a+4)^2<a^2+4a≦A<a^2 がいえるから、
Aのパリティもみることで A=(a+2)^2 ⇔ a^2-4b=a^2+4a+4 ⇔ b=-(a+1)
このとき、B=b^2-4a=(a+1)^2-4a=(a-1)^2 となっている。

5) a,b<0 であるとき
a^2<A≦a^2+4a<(a+2)^2 がいえるから矛盾となる。

以上より、求める整数a,b(|a|≧|b|)の組は
以下に挙げるもので全てであると結論できる。
(a,b)=(4,4),(6,5),(-n,n-1),(-n^2,0) (n:正整数)
15shin hattori:2010/11/21(日) 00:45:27
失礼。結論の部分にだけ少しミスがあった。
(-n^2,0)のところだけはn=0を許すとする。
16132人目の素数さん:2010/11/21(日) 19:43:11
11番は単純だけど綺麗だね。
忘れられた頃に大学入試の問題として採用(≒パクる)されてそうw
17132人目の素数さん:2010/11/21(日) 22:47:36
もっと問題追加しろカスが
18132人目の素数さん:2010/11/21(日) 23:43:46
文句いうなら自分でやってみろよカスが
19132人目の素数さん:2010/11/22(月) 00:09:04
更新&問題追加しました^^

@ (m,n)=(2,3),(2,5),(3,2),(5,2)のとき、
x+y+z=0を満たす任意の実数x,y,zに対して
(x^{m+n}+y^{m+n}+z^{m+n})/(m+n)=(x^m+y^m+z^m)(x^n+y^n+z^n)/mn
が成り立つことが分かっている。他に整数解(m,n)は存在するか?全て求めよ

A 以下の条件を満たす正の整数(a,x,y,n,m)の組をすべて求めよ
a(x^n−x^m)=(ax^m−4)y^2
m≡n (mod 2) かつ ax は奇数

B 自然数Nの10進表示を a[1]a[2]..a[n] 等とする。
N = (a[1]+2)*(a[2]+2)*...*(a[n]+2)
を満たすような自然数Nを全て求めよ。

C フィボナッチ数列類似の多項式列{F(n)}を次のように定義する:
F(0) = 0, F(1) = 1, F(n+2) = F(n+1) + x^n F(n).
このとき、任意の自然数nに対して、
F(5^n) は 1 + x + x^2 + … + x^(5^n - 1) で割り切れることを示せ。

Df^3=g^2+1 を満たすf,g∈C(X)\C は存在するか。
(C(X)は複素係数の有理関数体のことです^^)
20132人目の素数さん:2010/11/22(月) 00:09:44
おやすみです^^

E123456a+234567b+345678c+456789d+567890e+678901f+789012g
+890123h+901234i=123456789 を満たすような
0以上の整数a,b,c,d,e,f,g,h,iを求めよ。

F自然数x_1, x_2, ..., x_nが
1/x_1+1/x_2+...+1/x_n=1 を満たすとき
x_1+x_2+...+x_n の最大値を求めよ。

Ga^a*b^b=c^c を満たす整数a,b,c>1の組は無限に存在するか。

H次の条件を満たす素数p,qの組を全て求めよ。
[条件] p*q は三角数であり、p+q は平方数である。

Iτ(n^2-2)=22 を満たす正整数nは存在するか。

Jlim[n→∞]v_2(Π_[k=1,n](3^k-1))/n を計算せよ。

K次の条件を満たす奇数n(>3)を全て求めよ。
[条件] どんな奇数m(1<m<n)に対しても、
(n^m-n)/(m^n-m) は奇分数に約分できない。
(奇分数とは分母分子がともに奇数の有理数のこと)
21132人目の素数さん:2010/11/22(月) 00:17:47
あ、記号τを説明なしに用いていました^^;;
各正整数nに対して、τ(n)はnの正の約数の個数を表すとします。
(人によっては τの代わりに σ_0 や dなどを用いるかもです)

記号v_2の意味も一応説明してきますね^^
各正整数nに対して、v_2(n)は 2^k|n を満たす最大の整数kを表すとします。
たとえば、v_2(60)=2 となります。なぜなら 60=2^2*3*5 と分解できるので。
22132人目の素数さん:2010/11/22(月) 01:49:56
σ_0
23132人目の素数さん:2010/11/22(月) 01:55:22
σ_0 は お顔にみえるね。
σ_1 は正の約数の総和。(例:σ_1(6)=1+2+3+6=12)
σ_2 は正の約数の2乗和。(例:σ_2(6)=1^2+2^2+3^2+6^2=50)
σ_3 は正の約数の3乗和。(例:σ_2(6)=1^3+2^3+3^3+6^3=252)
この視点でみると σ_0(6)=1^0+2^0+3^0+6^0=4 だから
σ_0 を正の約数の個数とするのはとても自然におもえる。
なお、一般にσ_k(k:非負整数)は乗法的関数になる。
つまりσ_k(mn)=σ_k(m)*σ_k(n) (when gcd(m,n)=1) である。
24132人目の素数さん:2010/11/22(月) 11:11:33
wikipedia「合同式」で前の版の方がよかったのではないかと議論があります
履歴を見れば熾烈な編集合戦があったこともわかります
専門的コメントあればこちらに
http://toki.2ch.net/test/read.cgi/hobby/1290361067/
このスレは総本山的スレで、管理者が複数名書き込んでいます
管理者や常連がwikipediaの利用者ページでトリップを公開してる通り、本物です
25132人目の素数さん:2010/11/22(月) 12:54:07
^^は前スレURLくらい貼れや
26132人目の素数さん:2010/11/22(月) 18:48:00
>>25
言うだけじゃなく張れ。

初等整数論の問題
http://kamome.2ch.net/test/read.cgi/math/1286119277/
27132人目の素数さん:2010/11/22(月) 18:52:00
990
>直接関係するわけじゃないけど、
>構成的に双射Z→Qを作るときは
>p^{2n} --> p^n
>p^{2n+1} --> p^{-n}
>とするのが好き。

p^0-->p^0
p^1-->p^0
28132人目の素数さん:2010/11/22(月) 19:09:36
双射になってねえよね。
しかしそんな初等的な話をされてもねw
"問題"のほうがずっと頭つかうわな。
29132人目の素数さん:2010/11/22(月) 20:20:34
直接関係するわけじゃないけど、
構成的に双射Z→Qを作るときは
p^{2n} --> p^n
p^{2n-1} --> p^{-n}
-1 --> -1
0 --> 0
とするのが好き。
30:2010/11/22(月) 20:27:23
Jの解答
v_2公式を用いて計算できます。
各正整数rに対して、f(r)=v_2(3^r-1)とおきます。
rが奇数のとき、f(r)=1 であります。
rが偶数のとき、v_2の公式より、
f(r)=v_2(3^2-1)+v_2(r)-1=v_2(r)+2 となります。
また、vはまるで対数のように計算できるところにも注意です。
たとえば、v_2(32)=v_2(4)+v_2(8)=2+3=5 とできます。
ですから、ΠをΣに変えることができるわけです。(逆もしかり)
以上に述べたことに基づき、計算を実行してみます。

F(n):=v_2(Π_[k=1,n](3^k-1))=Σ[k=1,n]f(k) (∵掛け算を足し算にした)
=Σ[k=1,[(n+1)/2]]f(2k-1)+Σ[k=1,[n/2]]f(2k)
=[(n+1)/2]+Σ[k=1,[n/2]](v_2(2k)+2)
=[(n+1)/2]+2[n/2]+Σ[k=1,[n/2]](v_2(2k))

ここで、v_2(2k)=v_2(2)+v_2(k)=1+v_2(k) であり、
Σ[k=1,[n/2]]v_2(k)=v_2(Π[k=1,[n/2]]k) (∵足し算を掛け算にした)
=v_2([n/2]!)=[n/2]-s_2([n/2]) (∵階乗の指数公式)
であることから、結局、F(n)=[(n+1)/2]+4[n/2]-s_2([n/2]) となる。
ということは求める極限値は有限の値として存在していて、
それは 5/2 であるという結論に達することをみるのは易しい。
(注意: sのオーダーはlogであることに注意してください)
31:2010/11/22(月) 20:34:16
5/2 デスカ・・・
ということは 3^1-1, 3^2-1, ... , 3^n-1 とn個の数を並べて、
それぞれが2で何回割り切れるかどうかをみたとき、
その回数の平均値は 2.5(=5/2)ということですよね。
32132人目の素数さん:2010/11/22(月) 21:17:28
>>28
その三行が>>27より頭を使ってるのか。
33132人目の素数さん:2010/11/22(月) 22:25:42
直接関係するわけじゃないけど、
構成的に双射Z_p→Q_pを作るときは
x = p^e u と素元と単元の積にして、
p^{2n} --> p^n
p^{2n-1} --> p^{-n}
とできるね。
34132人目の素数さん:2010/11/22(月) 23:01:32
>>20
ちょっと早すぎるかもしれないけど
Iの回答おしえてくれない?
存在しないことの証明は厳しい気がする。
35132人目の素数さん:2010/11/22(月) 23:30:23
>>34
まだ1日も経っていないよ
さすがに時期尚早だろjk
36132人目の素数さん:2010/11/22(月) 23:32:04
>>34はいつもの人だからスルー推奨
37132人目の素数さん:2010/11/22(月) 23:59:47
>>11
頭の悪い書き込みですまないが、
wilsonの定理のもっとも早い証明方法の1つとしては
X^(p-1)-1∈F_p[X] に根と係数の関係を使うことだとおもう。
これだと一発で終わる。いわゆるワンラインソルーションだね。

まあただし、これの群論的な一般化、
たとえば、「有限可換群の元の総積は位数2以下の元となる」
を証明するときには 良く知られているように、
有限生成アーベル群の構造定理を適用するのだけど、
(適用すれば、たちまちに、そのことが確認できるが)
wilsonの定理みたいに簡潔すぎる解答はないわけだ。
38132人目の素数さん:2010/11/23(火) 00:06:34
@Fermat(フェルマ小定理)という単語は
度々、初等数論の問題の解説に使われるわけだけど、
(初等)群論的にはほとんど明らかだから、
(Z/pZ (p:素数)は有限体であるから 乗法群(Z/pZ)*に
ラグランジュの定理(の系)を適用することで 小定理が直截従う)
そういう意味で Fermatの小定理という単語を用いる理由はない。
なぜならば、群論において ラグランジュの定理は
ほとんどの場合において、(初歩的すぎる問題は除く)
ことわりをいれられずに暗黙のうちに用いられているから。
39132人目の素数さん:2010/11/23(火) 00:26:00
進学校の高校3年だが、
modの分数演算で試験の答案を書いたら、
×にされたんだが。。。

[どんな自然数nに対しても 3^(2n-1)+2^(n+1) が7で割り切れることを示せ]
3^(2n-1)+2^(n+1)≡(1/3)(9^n)+2*2^n≡(1/3)2^n+2*2^n≡(7/3)2^n≡0 mod 7
証明終わり。(この方法をごく最近2chで知ったあとに同じ問題がでたw)

modは整数じゃないと使えないとかいわれたんだが。
こういう計算をしていいのは gcd(3,7)=1 より明らかだからと反論したら、
modは整数同士の計算にしか使えないといわれた。鬱だ。
40132人目の素数さん:2010/11/23(火) 00:46:51
>>39
君の方法はとても簡潔で正しい。
ただそれはmodのことをなにもわかっていないと勘違いされる可能性がある諸刃の剣。
大学入試などの試験には使わないほうが良いよ。

ちょっとスレチだとおもうよ?
41132人目の素数さん:2010/11/23(火) 01:52:54
何の説明もなしに1/3 て書いたらそりゃ不正解にされるな。
1≡3×5 をうまく入れて、本当は逆元知ってるけど使わないんだぜ、
という余裕を見せなきゃ。
42132人目の素数さん:2010/11/23(火) 02:10:47
41のいっているとおり、
もとの式を3倍したものが7で割り切れることをいえばいいね。
そうすれば 3^(2n)+3*2^(n+1)≡2^n+3*2*2^n=(1+6)*2^n≡0
分数計算を回避できたわけだ。まあスレチだ。
43オナ豚:2010/11/23(火) 02:58:40
>>19
>>20
難易度指標(1〜10)

@ 5
A 9?
B 9
C 8
D 6
E 2
F 5
G 8
H 3
I ?
J 4
K 4

解けていないのがほとんどだけど最低限の思考はしたよ
6以上のはオレには解けそうにない問題ね 完全にお手あげ
44132人目の素数さん:2010/11/23(火) 07:51:16
誰かこれを解いてくれ

10^210/10^10+3 の整数部分の一の位の数字を求めよ
ただし3^21=10460353203 を用いてよい
45:2010/11/23(火) 08:00:26
>>44
既出ですよ。
前スレのhattoriさんの解答をほぼそのまま貼り付けますね。

d=10^10+3 とおく。
10^210=(10^10)^21≡(-3)^21= -10460353203 ≡ -460353200 (mod d)
よって 10^210 = dk-460353200 を満たす正整数kが取れる。
(これから 0≡3k (mod 10)を得るので kは10で割りきれる)
これから 10^210/d = k - 46035320/d がいえる。
これから 10^210/(10^10+3)の整数部分は k-1に等しいといえる。
10|kだったので k-1の下1桁は9であるといえる。答えは 9
46132人目の素数さん:2010/11/23(火) 15:24:25
>>20
F
x[1]=2
x[m+1]=Π[k=1,m](x[k])+1(m<n)
x[m+1]=Π[k=1,m](x[k])(m=n)
のとき
x[1]+x[2]+・・・+x[n]が最大になると
思うんですが証明できない。
47132人目の素数さん:2010/11/23(火) 15:29:39
>>46
訂正

>>20
F
x[1]=2
x[m+1]=Π[k=1,m](x[k])+1(m+1<n)
x[m+1]=Π[k=1,m](x[k])(m+1=n)
のとき
x[1]+x[2]+・・・+x[n]が最大になると
思うんですが証明できない。
48132人目の素数さん:2010/11/23(火) 19:00:27
>>45
ありがとうございます。。
49132人目の素数さん:2010/11/24(水) 00:07:01
Eって コンピュータを用いて、力技でやりたいろころだけど、
大雑把にカウントしてみると 100^9 = 10^18 通りぐらいあるね。
もしかして これはある意味むずかしいということでは?
50132人目の素数さん:2010/11/24(水) 04:09:41
880
1
1
1
9
5
1
2
3
51132人目の素数さん:2010/11/24(水) 04:39:28
>>50
それで全て? どのくらい時間かかった?
検索方法は 上界を抑えての全数調査?
52132人目の素数さん:2010/11/24(水) 04:42:57
ヤマカン
53132人目の素数さん:2010/11/24(水) 04:48:00
じゃあそれで全てとは言い切れないね。
勘でいわせてもらうと、解は結構あるとおもうよ。
この問題はコンピュータで全数調査しようとすると
ざっと見積もって(粗いが) 10^18通りぐらいあるから、
それなりにアルゴリズムを工夫しないと このPCじゃ無理っぽい。実は難問か
54132人目の素数さん:2010/11/24(水) 05:08:12
>>53
いわゆるコインの問題は 変数の個数が一般の場合は(4変数以上)
NP-hard ということが知られているから 注意が必要。
とくにフロベニウス数が閉じた式で得られるのは2変数までで、
3変数以上(3変数ですら!)は存在しないとされている。絶望的。

幸運なことにこの問題は"全て"求めることを要求していない。
ゆえに >>50 のレスはEの回答として扱われ、
この問題はオナ豚さんの言っている通り 確かに易しいとなる。
55132人目の素数さん:2010/11/25(木) 09:54:02
>>43
4以下の解けよ。
4以下が本当ならな。
56132人目の素数さん:2010/11/25(木) 15:38:22
H次の条件を満たす素数p,qの組を全て求めよ。
[条件] p*q は三角数であり、p+q は平方数である。

p=11, q=5 は条件を満たす組の1つである。
条件を満たす素数p,q(p≧q)の組を任意に取る。
p*q=m(m+1)/2 ⇔ 2pq=m(m+1) なる整数m>2が取れる。
よって、 p-2q=1 または p-2q=-1 である。
後者の場合、p+q=3q-1≡-1 (mod 3) であるから、
p+qは平方数になりえず、矛盾であるといえる。
よって、p-2q=1 であり、このとき、p+q=3q+1 である。
3q+1=n^2 ⇔ 3q=(n+1)(n-1) を満たす整数n>2が取れる。
ということは n-1=3 ⇔ n=4 の成立がいえる。
このとき、q=5 であり、p=11 であるといえる。
以上より、求める素数p,q(p≧q)の組は p=11,q=5のみである。
57132人目の素数さん:2010/11/25(木) 20:04:16
あげまん
58132人目の素数さん:2010/11/26(金) 21:28:53
GIVE UP(´・ω:;.:...

問題提出
1) 下3桁が同じ数字の平方数で、2番目に小さいものを求めよ。(オリジナル?)
2) 2^29は9桁の数であり、桁に重複はない。
  抜けている数字を求めよ。(電卓等を使うな) (オリジナル?)
3) [√n]+[√(n+1)]+[√(n+2)]=[√(9n+8)] を示せ。(有名問題)
59132人目の素数さん:2010/11/26(金) 21:32:59
(1)(3)はすぐわかった
60132人目の素数さん:2010/11/26(金) 21:35:56
>>59
せめて方法を書け。
そのほうが他の人がうれしがるぜ?
61132人目の素数さん:2010/11/26(金) 21:40:49
以下、58の問題を叩く流れになると予想。
そこで事前にオレが食い止める。
いいか、オリジナルかどうかは本人が
そう思っているなら なんら問題ない。
謙虚に?なぞ付けなくて良い。

3)は √(9n+8)<√n+√(n+1)+√(n+2)<√(9n+9)
をテクニカル(とくに中辺<右辺は)に示すことができて、
さらに、mod 9で 8は平方数でないことから
√(9n+8)は整数になりえないといえるので
上の不等式とあわせて終了となる。
62132人目の素数さん:2010/11/26(金) 21:48:58
>>60
modmodmodだ
63132人目の素数さん:2010/11/26(金) 21:50:36
(1)も有名だからな
64132人目の素数さん:2010/11/26(金) 21:53:24
調子にのってふたたび投稿

4) ax+by(x,yは非負整数)で表現でない正整数の個数が97となるような
  正の整数a,b(a≧b)の組を全て求めよ。
65132人目の素数さん:2010/11/26(金) 21:54:54
>>58
10000
一番めは1444
66132人目の素数さん:2010/11/26(金) 21:56:13
>>64
うはっチョー難問
67132人目の素数さん:2010/11/26(金) 21:58:29
1)は0を除いたら 462^2 = 213444 というのが2番目だね。
213444 って面白くねえ? 見た目的にppp
68shin hattori:2010/11/26(金) 22:04:10
>>64
良く知られた結果より、
(a-1)(b-1)/2 = 97 を解けばよい。
よって、(a,b)=(195,2),(98,3)

trivialな一般化
正整数の定数a,b(a≧b)に対して、
ax+by(x,yは非負整数)で表現でない正整数の
個数が素数となるとき、b≦3 が示せる。
69132人目の素数さん:2010/11/26(金) 22:12:57
(問題)
一般に自然数Nが与えられたとき、
それを反転させたものをNの反転数N'と呼ぶことにする。
たとえば、N=426 のとき、N'=624 である。

N-N' が3の冪乗になるとき、Nをおちんちんナンバーと呼ぶことにする。
全ての おちんちんナンバーを求めよ。 (確実にオリジナル問題です)
70132人目の素数さん:2010/11/26(金) 22:16:15
ちょっと次のように変更しておきます。
このままだと解の表現がいびつになるので。

N-N'の形で表現できる数を "おちんちんナンバー" とします。
そこで おちんちんナンバーな3の冪乗を全て求めよとします。
71132人目の素数さん:2010/11/26(金) 22:17:36
>>69
表現自重汁
キチガイすぎるw
72132人目の素数さん:2010/11/26(金) 22:22:13
s(N)=s(N') ゆえ N-N'≡s(N)-s(N')≡0 (mod 9)
よって、3はおちんちんナンバーではない。
21-12 = 9
41-14 = 27
81はどうなのか調べ中。
73132人目の素数さん:2010/11/26(金) 22:37:36
おてぃん
てぃんage
74132人目の素数さん:2010/11/26(金) 22:46:08
>>58
2)
「2^29は9桁の数であり、桁に重複はない。」より
抜けている数をxとすると、2^29を9で割った余りは
0+1+2+3+4+5+6+7+8+9-x=45-x≡9-x mod 9
また、二項定理を利用して
2^29≡(3-1)^29≡C[29,1]*3-1≡86≡5 mod 9
2^29を9で割った余りは5
9-x=5よりx=4 抜けているのは4
75132人目の素数さん:2010/11/26(金) 23:13:10
φ(9)=6より、2^29≡1/2≡5
これが 2^29をmod 9で計算する最速の方法
76132人目の素数さん:2010/11/26(金) 23:21:34
77132人目の素数さん:2010/11/27(土) 02:12:39
今日はもう遅い(2時^^;)ですが更新しました^^
>>70 OT-numberでいいでしょうか?^^ そのままだと困ります^^;

@ (m,n)=(2,3),(2,5),(3,2),(5,2)のとき、
x+y+z=0を満たす任意の実数x,y,zに対して
(x^{m+n}+y^{m+n}+z^{m+n})/(m+n)=(x^m+y^m+z^m)(x^n+y^n+z^n)/mn
が成り立つことが分かっている。他に整数解(m,n)は存在するか?全て求めよ

A 以下の条件を満たす正の整数(a,x,y,n,m)の組をすべて求めよ
a(x^n−x^m)=(ax^m−4)y^2, m≡n (mod 2) かつ ax は奇数

B 自然数Nの10進表示を a[1]a[2]..a[n] 等とする。
N = (a[1]+2)*(a[2]+2)*...*(a[n]+2)
を満たすような自然数Nを全て求めよ。

C フィボナッチ数列類似の多項式列{F(n)}を次のように定義する:
F(0) = 0, F(1) = 1, F(n+2) = F(n+1) + x^n F(n).
このとき、任意の自然数nに対して、
F(5^n) は 1 + x + x^2 + … + x^(5^n - 1) で割り切れることを示せ。

Df^3=g^2+1 を満たすf,g∈C(X)\C は存在するか。
78132人目の素数さん:2010/11/27(土) 02:33:59
おやすみです^^

E自然数x_1, x_2, ..., x_nが
1/x_1+1/x_2+...+1/x_n=1 を満たすとき
x_1+x_2+...+x_n の最大値を求めよ。

Fa^a*b^b=c^c を満たす整数a,b,c>1の組は無限に存在するか。

Gτ(n^2-2)=22 を満たす正整数nは存在するか。

H次の条件を満たす奇数n(>3)を全て求めよ。
[条件] どんな奇数m(1<m<n)に対しても、
(n^m-n)/(m^n-m) は奇分数に約分できない。
(奇分数とは分母分子がともに奇数の有理数のこと)

I一般に自然数Nが与えられたとき、
それを反転させたものをN'などと書くことにする。
たとえば、N=1234 のとき、N'=4321 である。
N-N'の形で表現できる数を OT-number と呼ぶことにする。
3の巾で書けるようなOT-numberを全て求めよ。
79132人目の素数さん:2010/11/27(土) 03:31:13
>>20K=>>78H
x,yが正整数でxが奇数かつx>1、yが偶数のときv_2(x^y-1)=v_2(x-1)+v_2(x+1)+v_2(y)-1だから
v_2(n^m-n)=v_2(n-1)+v_2(n+1)+v_2(m-1)-1, v_2(m^n-m)=v_2(m-1)+v_2(m+1)+v_2(n-1)-1
[条件]⇔どんな奇数m(1<m<n)に対してもv_2(n^m-n)≠v_2(m^n-m)
⇔どんな奇数m(1<m<n)に対してもv_2(n+1)≠v_2(m+1)………………(*)
今nが[条件]を満たす奇数とし、n+1=(2^a)*b (b:正の奇数)と素因数分解されるとする
(n> 3よりb=1かつa≧3、またはb=3かつa≧2、またはb> 3かつa≧1であることに注意)
もしb> 1なら m=(2^a)(b-2)-1 とおくとmは奇数で1<m<nかつv_2(n+1)=a=v_2(m+1)となり(*)に反する
よって b=1 で n=2^a-1 (a≧3)
逆に n=2^a-1 (a≧3) ならどんな奇数m(1<m<n)に対しても v_2(m+1) < a = v_2(n+1)なので[条件]を満たす
以上より求めるnは2^a-1 (a≧3)
80132人目の素数さん:2010/11/27(土) 20:43:31
9=10-01=9
18=42-24
27=41-14
36=73-37
45=72-27
54=71-17
63=70-07
72=91-19
81=90-09
90=????
99=100-001

90はOT-numberじゃないのかな??
3桁でやると 99の倍数にならざるをえないから、
90がOT-numberだとすると4桁以上で考える必要がある。
81132人目の素数さん:2010/11/27(土) 20:58:35
>>80
90 = 1101-1011

どんな9の倍数もOT-numberというのは正しいだろうか。
もしそうだとしたら、Iは自動的に解けたことになる。
82shin hattori:2010/11/27(土) 22:16:27
解決には至っていないけど >>81 は明確に否定できる。

[命題] (9の倍数の部分は >>72 が言及している)
nがOT-numberであるとし、n=N-N'とする。
Nの最上位の桁と最下位の桁をa[m],a[0]とする。
nは9の倍数であり、n≡a[0]-a[m] (mod 10)が成立する。
さらに mが偶数ならば nは11の倍数であるともいえる。

(証明)
N=Σ[i=0,m]a[i]*10^i とするとき、n=N-N'より、
n = (1-10^m)a[0]+(10^m-1)a[m]+Σ[i=1,m-1](10^i-10^(m-i))a[i] ・・(*)
右辺は9で割り切れるので、nは9で割り切れることがいえる。
(これを示すだけならば >>72 の方法が明快であるといえる)
(*)のΣ部分は10で割り切れるので、両辺をmod10でみることで 10|n がいえる。
mが偶数のときは 10^2≡1 (mod 11)に注意して、
(*)の両辺を mod11でみることで 11|n がいえる。
83shin hattori:2010/11/27(土) 22:17:53
[主張]
108+9d (d∈Z, 0≦d≦7)の形の数は OT-numberではない。

(証明)
上記の形の数が OT-number(=n)とし、n=N-N'とする。
nはその定め方から、10でも99でも割り切れず、n<189 である。
また、nは3桁以上なので、Nは3桁以上であるといえる。
Nが3桁であるとすると、[命題]より、99|n がいえる。
これは不適ゆえ、Nは4桁以上であるといえる。
Nの最高位の桁と最下位の桁をa[m],a[0]とする。
a[m]≠a[0]だとすれば、すぐわかるように n≧189がいえる。
これは不適ゆえ、a[m]=a[0] であることがいえる。
ここで [命題]を用いれば 10|n がいえる。これは矛盾。
84132人目の素数さん:2010/11/28(日) 03:29:17
今日はリスト更新は無しです^^;
(∵最後に更新してから 10レスもないので)
おやすみです^^;(もっと早く寝ないと^^;)

Iは(少なくとも私には)解けそうにないですが、
いくつか得た簡単な結果を問題にしてみました^^

J 次の(1),(2)を証明せよ。
(1) 10^n+8がOT-numberとなるような整数n>1は存在しない。
(2) 9*10^n は常にOT-numberである。
(2) OT-numberでない3の巾(つまり 3^m(m:正整数)の形の数)は無限に存在する。

(3)はIに直接関係しているとおもわれますが、Iの回答には遠い^^;
85132人目の素数さん:2010/11/28(日) 03:55:43
(2)が2つもありますね^^;
(3)に相当する問題は次のように改変しておきます^^;

(3)連続する4つの3の巾が全てOT-numberになることはない。
たとえば、3^2, 3^3, 3^4, 3^5 のうち、
3^2,3^3,3^4はOT-numberであるが 3^5はそうでない。
86132人目の素数さん:2010/11/28(日) 17:57:44
Iの提案者ですが 私の用意していた解答は誤っているようです。
当初は「全ての3の累乗はOT-numberである」と思っていたのですが...
お騒がせしてすみませんでした... それと次も正しくないですね
「n≡1(mod 4)ならば 3^nはOT-numberではない」大きな数の反例があるようです。
しかしながら >>85の(3)は正しいようです。>>84のレス内容から当然ですが...
87132人目の素数さん:2010/11/29(月) 01:57:43
自然数Nを1つ取る。Nに左から3を1個くっつけできる数をMとする。
さらにMに左から3を何個(≠0個)かくっつけてできる数をM'とする。
もしMが素数だとするならば、M'はMの倍数にできることを証明せよ。

たとえば、N=7とすれば、M=37 であり、M'=33337=901M とできる。
8887:2010/11/29(月) 02:04:23
問題修正。
M'は本質的でないから次のようにする。

素数Nを1つ取る。
Nに左から3を何個(≠0個)かくっつけてできる数をMとする。
MはNの倍数にできることを証明せよ。

たとえば、N=13とすれば、M=33333313=2564101*N とできる。
89132人目の素数さん:2010/11/29(月) 03:00:33
>>88
N=2または5のときはそれぞれM=32, 35でOK
N≠2, 5のときは(10,N)=1だから10^k≡1 (mod N)を満たす正整数kが存在する
ここで9|10^k-1でNは素数だから(9,N)=1または3、よって3N|10^k-1
Nがd桁ならM=N+10^d*(10^k -1)/3とすればN|M
90132人目の素数さん:2010/11/29(月) 03:59:15
>>88
以下、>>89 をわずかに改良したバージョン。
Nは素数である必要はない。
ただし、10と互いに素であることが重要。
10と互いに素でさえあれば、可能である。
10と互いに素でない場合、すぐに反例がみつかる。
ただし、素数に限定した場合、10と互いに素でないのは
N=2,5だけであり、この場合は可能であることがいえる。

(N,10)=1のとき、10^k≡1(mod N) かつ
v_3(k)≧v_3(N)-1を満たす正整数kが取れる。
このとき、v_3((10^k-1)/3)=1+v_3(k)≧v_3(N) であるから、
10^k≡1(mod N)とあわせて N|(10^k-1)/3 がいえる。
Nがd桁ならば M=N+10^d*(10^k -1)/3とすれば N|M
91132人目の素数さん:2010/11/29(月) 12:22:13
1から9までならなにを連続で付け加えても同じだね。
92132人目の素数さん:2010/11/29(月) 16:45:43
<問題>
10進表記で、ある4桁の整数を9倍したら元の数の各桁の数字を逆に入れ替えた整数になった。元の整数はいくつか。

<解説>
小学生の算数問題なので解説は省略。1089です。
しかし、何倍かして逆に入れ替えた数となるのは、
1089を9倍したときの9801と、2178を4倍したときの8712になるときだけだそうです。

さぁ、これを証明してみせなさい。
嘘です^p^

じゃ、10進法以外だったらどんなのがあるのかなぁ…
^^さんが楽しい問題に改題してくれることを祈ってます(笑)
93132人目の素数さん:2010/11/29(月) 17:28:30
>何倍かして逆に入れ替えた数となるのは、
>(1)を(2)倍したときの(3)と、(4)を(5)倍したときの(6)だけだそうです


これで十分立派な東大入試問題に大変身w
94132人目の素数さん:2010/11/29(月) 17:53:25
95132人目の素数さん:2010/11/29(月) 18:34:45
>>92 の自身の才能を少しだけ使って改題してみようぜ?

>>93
それはウソ。桁数の制限をはずした場合、あきらかに無限にある。
4桁の場合を任意回数連結させればいい。たとえば
871287128712... は 217821782178... で割り切れる。

じゃあ、こういう明らかなものを除いた場合の可能性は?
そういう問題にした場合、それはかなりの有名問題であり、
基本解は 1089 と 2178 しかないことが証明できる。
96132人目の素数さん:2010/11/29(月) 20:16:03
>>95
どこで有名なんだよw
書けよw

>>92
2進数の場合は明らかに成り立たないw
97132人目の素数さん:2010/11/29(月) 20:17:10
>>92はn進数4桁の一般解まで拡張できる
98132人目の素数さん:2010/11/29(月) 20:18:20
>>92はn進数4桁m倍の一般解まで拡張できる
99132人目の素数さん:2010/11/29(月) 20:28:29
>>98
ちょっと書いてくれない?
100132人目の素数さん:2010/11/29(月) 20:45:43
>>96
たとえば次のURLの23番目のコメントに回答があるよ
http://trickofmind.com/?p=265

解法はかなり素朴で(ただし少し長い)
要は帰納的に話を進めることができる。
詳細はpost23を見てほしい。丁寧に書いてある。
日本人のサイトでも回答みたことあるけど、
おおむねそれと同じ方法だった。
おそらくこういう素朴な方法しかないのだろう。
101132人目の素数さん:2010/11/29(月) 20:58:01
>>99
すこしは自分で考えろよ。
ヘッド比較すれば あとは基本的な解析だけだよ。
解析という言葉を使うのも嫌なぐらい。
>>92 はだからもっと難しい一般化を訊いているのでは。
102132人目の素数さん:2010/11/29(月) 21:22:32
私女だけど、http://trickofmind.com/?p=265 のURLを最初から書いていればこんなに荒れなかったと思う
答える方って、全体的に余裕ないよね
103132人目の素数さん:2010/11/29(月) 21:59:18
>>101
いやいや難しくない?
たとえば 2桁の場合ですら全ての解をパラメタp表示することは困難では??

あまり人を疑いたくないけど、>>98 は拡張できていないとおもうよ。
つまり解答を持っていないと。
104132人目の素数さん:2010/11/29(月) 22:22:23
>>103
一般解の意味が違うだけでは?
一般解=全ての解 ではないでしょ
105132人目の素数さん:2010/11/29(月) 23:29:18
>>103は^^以下の数学力しかないんだから、^^の判断を仰げやw
106132人目の素数さん:2010/11/29(月) 23:31:05
>>103みたいなのって本当気持ち悪いな
そろそろ消えろよお前

問題解くだけのマシーンかよおまえ
107132人目の素数さん:2010/11/29(月) 23:41:52
>>103
お前まだいたのかよ

問題を拡張できる ⇒ 解答が用意されている
というのは正しくないだろ
108御手洗景子:2010/11/30(火) 22:36:13
それよりおまえら草むしれよ
109132人目の素数さん:2010/11/30(火) 23:56:30
>>103
数検スレででかい顔してんじゃねーよ
110132人目の素数さん:2010/12/01(水) 19:07:29
お前も草むしれよ が正しいかと
111132人目の素数さん:2010/12/02(木) 21:24:30
>>20I=>>78G

条件を満たす正整数nをいっこ求めてみた。

τ(n^2-2)=22 ⇔ n^2-2=p^21 (pは素数)または n^2-2=p*q^10 (p,qは相異なる素数)

後の場合、mod 4で見ればq≠2が分かり、さらに2がmod qで平方剰余だからq≡±1 (mod 8)

q=7とすると、x^2≡2 (mod 7^10) ⇔ x≡±266983762 (mod 7^10)

n=266983762+(7^10)a (a:奇数)とおいて(n^2-2)/(7^10)が素数かどうかをチェックしていくと

a=7,n=2244310505のとき(n^2-2)/(7^10)=17831401727は素数となり、このnが条件を満たすことがわかる。

112132人目の素数さん:2010/12/02(木) 21:49:49
>>111
GJ! なるほどね!
次の予想(名前忘れた)は正しいとされているからね

[予想]
整数係数の多項式(最高次の係数は正)をPとしよう
Pに関して簡単な理由がないかぎり(曖昧であるが)
P(n)が素数となるような整数nは無限に存在する

この予想を考えればτ(n^2-2)の問題は
存在するが答えであると目星がつくわけだ
(先にqを適切にとれば n^2-2がpq^10の形になる整数nが取れると思うから)

以上の考えのもとで >>111 の回答をみてみれば いかに無駄がないかわかる
さきにqを決めておくのが重要であり、pを先に固定すると絶望できる。
そうなると q=2,3,5,7,... とやっていくわけだが、
平方剰余の第2補充法則から q=7が最初の候補だとわかる。
n^2-2は微分すると 2nであり よって多項式とmodに関するHasseの定理から
n^2≡2 (mod 7^10)を満たす整数nは存在していることがいえて、
具体的にその形を全て求めることができる
113132人目の素数さん:2010/12/02(木) 23:37:45
更新しました^^ おやすみでした^^

@ (m,n)=(2,3),(2,5),(3,2),(5,2)のとき、
x+y+z=0を満たす任意の実数x,y,zに対して
(x^{m+n}+y^{m+n}+z^{m+n})/(m+n)=(x^m+y^m+z^m)(x^n+y^n+z^n)/mn
が成り立つことが分かっている。他に整数解(m,n)は存在するか?全て求めよ

A 以下の条件を満たす正の整数(a,x,y,n,m)の組をすべて求めよ
a(x^n−x^m)=(ax^m−4)y^2, m≡n (mod 2) かつ ax は奇数

B 自然数Nの10進表示を a[1]a[2]..a[n] 等とする。
N = (a[1]+2)*(a[2]+2)*...*(a[n]+2)
を満たすような自然数Nを全て求めよ。

C フィボナッチ数列類似の多項式列{F(n)}を次のように定義する:
F(0) = 0, F(1) = 1, F(n+2) = F(n+1) + x^n F(n).
このとき、任意の自然数nに対して、
F(5^n) は 1 + x + x^2 + … + x^(5^n - 1) で割り切れることを示せ。

Df^3=g^2+1 を満たすf,g∈C(X)\C は存在するか。
114132人目の素数さん:2010/12/02(木) 23:40:16
インターセプト!
115132人目の素数さん:2010/12/02(木) 23:44:54

E自然数x_1, x_2, ..., x_nが
1/x_1+1/x_2+...+1/x_n=1 を満たすとき
x_1+x_2+...+x_n の最大値を求めよ。

Fa^a*b^b=c^c を満たす整数a,b,c>1の組は無限に存在するか。

G任意の素数pに対して、適当に符号を取れば、
pn^2±2 が平方数となる整数nの存在がいえる。

H4桁の自然数Mをp進法において反転させたものをNとする。
どのような整数k>1に対しても M=kN を満たすような
正整数p,M(k<p)の組が存在するというのは正しいか?

I4桁の自然数Mをp進法において反転させたものをNとする。
M=2N が成立するような正整数p,M(p>1)の組を全て求めよ。
116:2010/12/03(金) 00:16:37
H 正しい:
p=k^2+k-1, L=(k+1)p+1(2桁) として LL(4桁)を考えればよい
117132人目の素数さん:2010/12/03(金) 02:14:38
>>116
kLが3桁になるから不味いんじゃ?

>>113D
118117:2010/12/03(金) 02:17:02
途中で送信ボタン押してしまった(汗)

>>113D
楕円曲線y^2=x^3-1は有理関数でパラメトライズできない・・・てのはだめ?
119132人目の素数さん:2010/12/03(金) 03:00:02
>>117
LLはLを2つ並べたものじゃない?
p=k^2+k-1, L=(k+1)p+1 のとき
L=kL' (L'はLを反転させたもの)
よって、LL(Lを2つ並べたもの)に対して M=LLとすると
M'=L'L'=L'p^2+L'=(1+p^2)L'=(1+p^2)kL=k(Lp^2+L)=kM
120132人目の素数さん:2010/12/03(金) 03:03:36
k(k+1)がpを越えてしまうから不味いということかw
121132人目の素数さん:2010/12/03(金) 03:07:47
ああでも kL'は3桁にならないからOKじゃね?
kLは別に3桁になってもいいけど
122119:2010/12/03(金) 03:13:20
最後の行がおかしいから修正
kM'=kL'L'=k(L'p^2+L')=(1+p^2)kL'=(1+p^2)L=M
123132人目の素数さん:2010/12/03(金) 03:23:20
>>118
たとえば X^3+Y^3=Z^3 だとすると、
X,Y,ZはZ係数の2変数多項式でパラメトライズできる。
それをみると際どい感じがするけど 多分問題のはできないだろうね
それをどうやって証明するかだけど
初等数論の問題といっているわけだから初等的な道具だけで
証明を完成させることができるとおもうのだけど(俺は思いつかないが)
124132人目の素数さん:2010/12/03(金) 03:24:23
正)X^3+Y^3=Z^2
誤)X^3+Y^3=Z^3
125132人目の素数さん:2010/12/03(金) 04:06:30
>>115 のDはpに関する条件が抜けているとおもう。
このままだと Dの主張は誤っているとなる。(p=5)
126名無しさん@英語勉強中:2010/12/03(金) 08:57:58
n∈N∩Z[√(-1)]とすると
n=2^tΠ_{i=1..r}p_i^{e_i}Π_{j=1..s}q_j^{f_j}
(但し, p,q∈Zは夫々p≡1(mod 4),q≡3(mod 4)なる素数とする)
と表せる。この時,

nがGaussian integerの二つの平方の和とし表せる ⇒ f_1,f_2,…,f_sは全て偶数になる

の反例を探しているのですがどなたか反例を教えて下さい。
127117:2010/12/03(金) 09:28:46
6=(2+√(-1))^2+(2-√(-1))^2
128132人目の素数さん:2010/12/03(金) 10:22:38
>>126
前スレで必要十分条件の問題があったよ
129132人目の素数さん:2010/12/03(金) 10:33:52
貼り付け(スレチということで番号付けされなかったけど)

179 :132人目の素数さん :2010/10/12(火) 00:49:14
Dはもし出題するとすれば次のようになるでしょう^^;

D Z[√-1] においては次のことがいえる。p=1+√-1 とおく。
x=a+b√-1が2つの平方数の和で表現できることは
2|b かつ v_p(x)≠1,3 であることと同値である。
(Z[√-1]においては平方数の差も和も表現できる数は同じです
というのも z→iz という対応を考えればよいわけですから^^;)
130132人目の素数さん:2010/12/03(金) 10:40:28
だから >>126 の条件n∈N∩Z[√(-1)] の中で考えるならば
最小の反例は自動的に n=3=2^2+i^2 になる
131132人目の素数さん:2010/12/03(金) 12:19:33
>>125
>>115 のGは酷いことになっていました^^; すみません^^;
G素数pに対して、p=2,p≡-1(mod 4) であることは
pn^2±2が平方数となるように整数nと符号が選べることの必要十分条件である。
これを証明せよ

>>129
懐かしいものを挙げてきましたね^^;
mod 4=λ^4 で決まると言ったほうが簡潔だったかも^^;
mod 4というのはもちろんZ[√-1]の中でということです。
そういっておけば ZをZの中で2つの平方数の差に分解できるような
ものを分類せよというよく知られた問題の結論との類似がみえますね^^;
132132人目の素数さん:2010/12/03(金) 17:52:28
>>131
できた

p=2のときはほぼ明らか。p≧3,p≡-1(mod 4)とする。
X^2-pY^2=1 の最小の正整数解(X,Y)を取る。
(X+1)(X-1)=py^2 であるから次の4通りの場合がある。

1) X+1=pA^2, X-1=B^2 を満たす正整数A,Bが取れる
2) X-1=pA^2, X+1=B^2 を満たす正整数A,Bが取れる
3) X+1=2pA^2, X-1=2B^2 を満たす正整数A,Bが取れる
4) X-1=2A^2, X+1=2pB^2 を満たす正整数A,Bが取れる

3),4)のときは 2=|2pA^2-2B^2| ⇔ B^2-pA^2=±1 を得る。
p≡-1(mod 4)より B^2-pA^2=1 であることがいえる。
(∵一般に p≡-1(mod 4)のときは S^2-pT^2=-1は整数解を持たない)
しかしこれはX,Yの最小性に反する。

1),2)のときは |pA^2-B^2|=2 となるので ゆえに示された
133shin hattori:2010/12/03(金) 18:10:39
>>132
逆を示す必要があるとおもうのですが。
といっても 逆はすごく簡単みたいです。
pn^2±2 が平方数だとします。
2|n ならば pn^2±2≡±2(mod 4)であり 矛盾。
そうでないなら pn^2±2≡p±2(mod 4)であり p≡1(mod 4)でないといえる
134132人目の素数さん:2010/12/03(金) 20:26:04
>>132
∵の部分の理由をおしえてくれないかい?
135132人目の素数さん:2010/12/03(金) 20:50:17
>>134
第1補充法則から明らか
136132人目の素数さん:2010/12/03(金) 23:18:19
以下の数列 a(n) を考える
「n^k + 1 が素数とならない最小の自然数をkとするとき a(n) = k と定義する」

例えば、n=10のとき 10^1 + 1 = 11 (素数)、10^2 + 1 = 101 (素数)、10^3 + 1 = 1001 (合成数)
なので、a(10) = 3 である。
実は、これは、http://oeis.org/A102368 にある数列なのですが、nが小さいうちは取りうる値としては
1,2,3のみです。 そこで問題は、果たして、この数列の取りう上限値、あるいは条件を定めることが出
来るかどうか? です。フェルマーの小定理とか使えますか。
137132人目の素数さん:2010/12/04(土) 00:10:51
>>136
n^3+1 = (n+1)(n^2−n+1)


138132人目の素数さん:2010/12/04(土) 00:24:09
>>136
非常に難しい
すぐにわかることは >>137 が分解しているように
n>1ならば a(n)≦3 であるということぐらい

n+1, n^2+1 がともに素数となるような
正整数nは無限に存在すると予想するの妥当だから
一番頻度の低い3は無限回でてくると予想されるわけだ
139132人目の素数さん:2010/12/04(土) 17:29:01
[問題]
f[1]=f[2]=1, f[n+2]=f[n+1]+f[n]によって数列を定めるとき
任意の素数pに対して f[p]≡(5/p) (mod p) であることを示せ
140132人目の素数さん:2010/12/04(土) 17:55:44
>>139, p≠2,5とする。
f[p]=(1/√5)(φ^p-(φ*)^p)={(1+√5)^p-(1-√5)^p}/(2^p√5)
ここで mod pで考えるならば 分子≡1^p+(√5)^p-(1^p-(√5)^p)≡2(√5)^p
よって、f[p]≡{(√5)^(p-1)}/(2^(p-1))≡{5^{(p-1)/2}}/1≡(5/p) ■
141132人目の素数さん:2010/12/04(土) 18:05:56
>>140
それは初等数論の方法に含まれるかい?
mod pの意味が高校数学のそれとは違う
142132人目の素数さん:2010/12/04(土) 18:10:09
どう見ても初等的。
143132人目の素数さん:2010/12/04(土) 18:14:54
代数的にはとても簡単だが初等数論的かどうかなら際どいか(√5の部分)
144132人目の素数さん:2010/12/04(土) 18:18:04
>>140
高校生:「分子の部分は無理数だよね。
無理数部分をmodでみていいのですか?
問題文のmodと あなたのmodは整合性がとれているのですか?」
145132人目の素数さん:2010/12/04(土) 18:23:26
くだらねえよ
ここは高校生スレじゃねえよ
すこし勉強したら誰だってわかることなんだから
いちいちつっこまんでええわ
高度なことやっているわけじゃないんだからさ
146132人目の素数さん:2010/12/04(土) 18:23:29
√5が出てくるのがよほどいやなら2項定理で分子を展開してやればいいだろ。
結局>>140と同じになる。
147132人目の素数さん:2010/12/04(土) 18:30:33
>>146  で終了だな
分子を展開すれば √5は分母と相殺できる。
さらに分数をmodでみるのが嫌な人は
両辺に2^pやらを掛け算すれば万事OK
148132人目の素数さん:2010/12/04(土) 18:41:36
>>146
サンクス
149132人目の素数さん:2010/12/05(日) 22:35:53
またいつもの人が暴れたのか
150132人目の素数さん:2010/12/05(日) 23:09:48
おやすみでs^^

@ (m,n)=(2,3),(2,5),(3,2),(5,2)のとき、
x+y+z=0を満たす任意の実数x,y,zに対して
(x^{m+n}+y^{m+n}+z^{m+n})/(m+n)=(x^m+y^m+z^m)(x^n+y^n+z^n)/mn
が成り立つことが分かっている。他に整数解(m,n)は存在するか?全て求めよ

A 以下の条件を満たす正の整数(a,x,y,n,m)の組をすべて求めよ
a(x^n−x^m)=(ax^m−4)y^2, m≡n (mod 2) かつ ax は奇数

B 自然数Nの10進表示を a[1]a[2]..a[n] 等とする。
N = (a[1]+2)*(a[2]+2)*...*(a[n]+2)
を満たすような自然数Nを全て求めよ。

C フィボナッチ数列類似の多項式列{F(n)}を次のように定義する:
F(0) = 0, F(1) = 1, F(n+2) = F(n+1) + x^n F(n).
このとき、任意の自然数nに対して、
F(5^n) は 1 + x + x^2 + … + x^(5^n - 1) で割り切れることを示せ。

Df^3=g^2+1 を満たすf,g∈C(X)\C は存在するか。
151132人目の素数さん:2010/12/05(日) 23:10:59
E自然数x_1, x_2, ..., x_nが
1/x_1+1/x_2+...+1/x_n=1 を満たすとき
x_1+x_2+...+x_n の最大値を求めよ。

Fa^a*b^b=c^c を満たす整数a,b,c>1の組は無限に存在するか。

G4桁の自然数Mをp進法において反転させたものをNとする。
M=2N が成立するような正整数p,M(p>1)の組を全て求めよ。

H|P(n)|が素数となるような整数nが5つ存在するような
3次の可約な整数係数多項式Pを1つ求めよ。(可約に注意)

I各整数n>1に対して、nの最小素因数をp(n)と表すとき、
min{p(n^2+37)|n∈Z,n>1}を計算せよ。(駄問失礼です^^;)
152132人目の素数さん:2010/12/05(日) 23:15:31
Hはタイプミスがありました^^;
p(n):= (nの最小の奇素因数) としてください^^;
(nが奇素因数を持たないときは p(n)=1 とします)

n^2+37>2は4で割り切れることはないので、問題の値は2より大です^^;
153132人目の素数さん:2010/12/05(日) 23:30:04
>>152
HじゃなくてIだね。
Hは存在自体非自明だね。
154132人目の素数さん:2010/12/07(火) 01:21:45
(2x+5)(2x^2-1):x=-3,-2,-1,0,1
(2x+3)(x^2-3x+1):x=-2,-1,0,1,2
(x+2)(x^2-3x+1):x=-3,-1,0,1,3
155132人目の素数さん:2010/12/07(火) 03:17:22
>>154
すげえな どうやってみつけたか聞いていいか?
156132人目の素数さん:2010/12/07(火) 03:49:29
すごくないよ
157132人目の素数さん:2010/12/07(火) 04:00:22
(6x+11)(x^2-x-1)
x=-2,-1,0,1,2.
158132人目の素数さん:2010/12/07(火) 04:45:04
>>155
キモはPの2次因子の形が決定できるということ。

P=QR, deg(Q)=1, deg(R)=2とすると、|P|が素数なら
(1) |Q|=1で、|R|は素数 --高々2通り
または
(2) |R|=1で、|Q|は素数 --高々4通り
特に、|P(n)|が素数となるnが5つあるなら、(2)の場合が少なくとも3通りなければならない。
Rの2次項係数が正であるとしておく。
|R(a)|=|R(b)|=|R(c)|=1(a,b,cは整数でa<b<c)とすると、R(a)=R(c)=1, R(b)=-1
(i) a+c=2bのとき
d=c-b=b-aとおくとdは正整数でR(x)=k(x-b+d)(x-b-d)+1(kは正整数)
このとき-1=R(b)=1-kd^2⇔kd^2=2 これよりk=2, d=1、よってR(x)=2(x-b)^2-1.
(ii) a+c≠2bのとき
2次関数の軸に関する対称性よりR(b')=-1, a<b'<c、b≠b'となる整数b'が存在する。b<b'としてよい。
-2=R(b')-R(a)はb'-aで割り切れるからb'-a=1 or 2。同様にc-b=1 or 2。
a<b<b'<cだから、この4数は隣り合う数の差が1でなければならない。
R(x)=k(x-a-1)(x-a-2)-1(kは正整数)とおくと1=R(a)=2k-1⇔k=1、よってR(x)=(x-a-1)(x-a-2)-1

こうやってRの形を決めたらあとは1次因子Qが条件を満たすように決めればいい。
159132人目の素数さん:2010/12/08(水) 01:28:12
>>158
サンクス!
素数6個は不可能ぽいが5個の場合はいくらでも例が作れそうなんだな
160132人目の素数さん:2010/12/08(水) 22:10:04
I 19が答えである。1^2+37=2*19 であるが、
(-37/p)をp=3,5,7,11,13,17に対して計算してみると全て-1になるから。
実際に計算してみると次のようになる(補充法則および相互法則を利用)

(-37/3)=(-1/3)= -1
(-37/5)=(-2/5)=(2/5)=-1
(-37/7)=(-2/7)=-(2/7)=-1
(-37/11)=(-2^2/11)=-1
(-37/13)=(2/13)=-1
(-37/17)=(-3/17)=(3/17)=(17/3)=(2/3)=-1

問題として考えられるもの。
「もうちょっとminの値の19が大きくなるような
なるべく小さな定数は37のほかになにがあるか」
161132人目の素数さん:2010/12/11(土) 15:30:00
-58.
p<=23.

-163.
p<=37.

-11302.
p<=43.

-30493.
p<=61.

-106177.
p<=71.
162132人目の素数さん:2010/12/11(土) 19:30:00
-2430943.
p<=73.
163132人目の素数さん:2010/12/11(土) 22:50:36
pに対して定数の絶対値は爆発的に大きくなっていく.
164132人目の素数さん:2010/12/12(日) 19:24:24
[問題]
各整数n≧6に対して、τ(n!)|n! が成立することを示せ。
165132人目の素数さん:2010/12/13(月) 02:08:11
おやすみです^^

@ (m,n)=(2,3),(2,5),(3,2),(5,2)のとき、
x+y+z=0を満たす任意の実数x,y,zに対して
(x^{m+n}+y^{m+n}+z^{m+n})/(m+n)=(x^m+y^m+z^m)(x^n+y^n+z^n)/mn
が成り立つことが分かっている。他に整数解(m,n)は存在するか?全て求めよ

A 以下の条件を満たす正の整数(a,x,y,n,m)の組をすべて求めよ
a(x^n−x^m)=(ax^m−4)y^2, m≡n (mod 2) かつ ax は奇数

B 自然数Nの10進表示を a[1]a[2]..a[n] 等とする。
N = (a[1]+2)*(a[2]+2)*...*(a[n]+2)
を満たすような自然数Nを全て求めよ。

C フィボナッチ数列類似の多項式列{F(n)}を次のように定義する:
F(0) = 0, F(1) = 1, F(n+2) = F(n+1) + x^n F(n).
このとき、任意の自然数nに対して、
F(5^n) は 1 + x + x^2 + … + x^(5^n - 1) で割り切れることを示せ。

Df^3=g^2+1 を満たすf,g∈C(X)\C は存在するか。
166132人目の素数さん:2010/12/13(月) 02:10:28
E自然数x_1, x_2, ..., x_nが
1/x_1+1/x_2+...+1/x_n=1 を満たすとき
x_1+x_2+...+x_n の最大値を求めよ。

Fa^a*b^b=c^c を満たす整数a,b,c>1の組は無限に存在するか。

G4桁の自然数Mをp進法において反転させたものをNとする。
M=2N が成立するような正整数p,M(p>1)の組を全て求めよ。

H各整数n≧6に対して、τ(n!)|n! が成立することを示せ。

I次の条件を満たす整数m>1を全て求めよ。
[条件]あるf∈Z[X]が存在していて、(1),(2),(3)を満たしている
1) ∃n∈Z f(n)≡0 (mod m)
2) ∃n∈Z f(n)≡1 (mod m)
3) ∀n∈Z f(n)≡0 ∨ f(n)≡1 (mod m)
167132人目の素数さん:2010/12/15(水) 01:01:35
任意の正整数∋nに対し、

Σd =Σn/d を示せ。
d|n   d|n

Σdとはnの約数の和である。
d|n

例:n=12のとき

Σd=1+2+3+4+6+12=28
d|12

Σ12/d=12/1+12/2+12/3+12/4+12/6+12/12=28
d|12

すいませんがわかる方、証明お願いします。
168132人目の素数さん:2010/12/15(水) 01:33:25
>>167
nの正の約数(k個あるとします)を
d_1,d_2,..,d_k とおきますと、
n/d_1, n/d_2, ..., d/d_k も
k個の異なるnの約数となります。

さて、Σ[d|n]d はnの約数を全て足し合わせるということですから、
したがって、Σ[1≦i≦k]n/d_i と一致することがいえるわけです。
Σ[1≦i≦k]n/d_i = Σ[d|n]n/d は 明らかですから これで終わり。
169訂正:2010/12/15(水) 01:37:33
d/d_k は n/d_k の間違いです。

各n/d_i がnの約数になっていることは
n/(n/d_i) = d_i(正整数) の成立からいえます。
n/d_1, n/d_2, ..., n/d_k がそれぞれ異なっているのは
d_1,d_2,..,d_k が異なっていることからいえます。
ですから個数を比較することで、
{d_1,d_2,..,d_k} = {n/d_1, n/d_2, ..., n/d_k} がいえるのです。
170132人目の素数さん:2010/12/15(水) 16:21:36
a,b,c,d∈Z+でab-cd=1を満たすとする。
このとき(a+b)/(b+d)は既約分数であることを示せ。

171132人目の素数さん:2010/12/15(水) 16:24:24
なめとんのか?
172猫はCat ◆MuKUnGPXAY :2010/12/15(水) 16:25:52
見てるよ。


173132人目の素数さん:2010/12/15(水) 17:31:06
>>170
入試問題で言えば、設問の小問@ってとこじゃね。
もちろんDQN大
174132人目の素数さん:2010/12/15(水) 17:35:06
なにこの素敵な問題群
175:2010/12/15(水) 18:52:08
>>170
反例) a=2,b=5,c=1,d=9 とすると、
ab-cd=1 であるが、(a+b)/(b+d)=7/14 となってしまう

おそらく正しい問題文は次の通りかと。

a,b,c,d∈Z+ でad-bc=1を満たすとする。
このとき(a+b)/(c+d)は既約分数であることを示せ。

[解答例]
a+b,c+dの共通素因子が存在したとし、それをpとすると、
b≡-a, c≡-d (mod p)であるから、
1≡ad-bc≡ad-(-a)(-d)≡0 (mod p)
これは明らかに矛盾である。
176132人目の素数さん:2010/12/15(水) 20:18:16
>>175
だな、俺もそう思う。
177132人目の素数さん:2010/12/16(木) 08:48:36
x,y,z∈Zとし,x^2+y^2≠0でz≡3 (mod 4)でzは平方数でないとする。
この時,(x^2+y^2)zは平方数の和(つまり,(x^2+y^2)z=a^2+b^2(但し,a,b∈Z, a^2+b^2≠0))として表せない。

は正しいでしょうか? 正しくないなら反例をお願い致します。m(_ _)m
178132人目の素数さん:2010/12/16(木) 12:15:53
うわー、>>173顔真っ赤だなー、真っ赤だわ
179132人目の素数さん:2010/12/16(木) 12:17:48
178 132人目の素数さん[sage]:2010/12/16(木) 12:15:53
うわー、>>173顔真っ赤だなー、真っ赤だわ
180:2010/12/16(木) 14:12:54
>>177
正しいです

z≡-1(mod 4)より、v_p(z)が奇数となるような素数p≡-1(mod 4)が取れます。
そのとき、v_p((x^2+y^2)z)も奇数ですから、2つの平方数の和で表現不可です。
181:2010/12/16(木) 20:23:40
>>166
I問題文を次のように見やすくしておきます。
f(Z/mZ)={0,1}を満たすf∈Z[X]が存在するような
整数m>1を全て求めよ(Z/mZ={0,1,2,..,m-2,m-1})

[回答]
求める整数m>1は素数冪だけである。これを示す。

mが素数冪であるとき、f(X)=X^(φ(m))とすればよい。
mが素数冪でないとき、m=a*b なる互いに素な整数a,b>1の組が取れる。
f(Z/mZ)={0,1} を満たすf∈Z[X]が存在したと仮定する。
このとき、f(Z/aZ)={0,1} かつ f(Z/bZ)={0.1} が成立する。
f(α)=0 in Z/aZ かつ f(β)=1 in Z/bZ
を満たすα+aZ∈Z/aZ, β+bZ∈Z/bZ の組が取れる。
中国剰余定理より、x≡α(mod Z/aZ)かつx≡β(mod Z/bZ)
を満たすようなx∈Zが取れるが、そのようなxに対して、
f(x)≡0,1 (mod Z/mZ) は成立していない。
182132人目の素数さん:2010/12/16(木) 22:21:25
>180
どうも有難うございました。お蔭様で解決できました。
183132人目の素数さん:2010/12/17(金) 17:28:49
1つの天秤とそれぞれ
3^0=1,3^1,3^2,....,3^(n-1)キロのn個の錘が与えられたとする。
何個かを一方の皿に乗せ、別の何個かをもう一方のさらに乗せることによって、
全ての重さN個を量ることが出来る。このことを示せ。
ただしNは1≦N≦((3^n)-1)/2の範囲の整数とする。
(ヒント:次の形の整数の和を考えよ。
 e_0+3e_1+3^2e_2+.....+3^(n-1)e_n-1
 ただし、e_i=-1,0,1)
184132人目の素数さん:2010/12/17(金) 17:50:39
>>183
>>1

まあ3進展開の一意性より明らかといっておこう。
(係数は0,1,2から-1,0,1にずらしても性質は不変)
185132人目の素数さん:2010/12/20(月) 19:26:50
<答え>
k を整数とすると
p を満たすのは 4k+2
k=2l(エル)+1
よって解は (5-√21)/2 です


池沢春人「黒鉄−クロガネ−」より抜粋(笑)
http://a-draw.com/contents/uploader2/src/a-draw1_1191.jpg


<問題>
以上の解を満たすような数IAの整数問題を作成しなさい^p^
186132人目の素数さん:2010/12/20(月) 19:45:14
>>183-184
 平衡3進法
187132人目の素数さん:2010/12/20(月) 20:32:15
>>185
おもしろいw どっからとってきた?その漫画w
188132人目の素数さん:2010/12/20(月) 21:10:20
>>185
f(p):={(p-2)-4√(p-1)}/8 が
X^2+aX+b∈Z[X] の正の根となるような
最小の自然数pに対して、f(p)の値を求めよ。
189132人目の素数さん:2010/12/20(月) 21:26:38
>>187
あぁ、書き忘れてました
今週の週刊少年ジャンプです^p^
190132人目の素数さん:2010/12/20(月) 21:27:27
>>188
もっと数IAっぽく書いてくれ
もしくは東大入試問題っぽくw
191132人目の素数さん:2010/12/20(月) 21:27:54
>>185
ちょっと解答が足りないけれど.

パラメータpを次のようにとる.
・pは平方数を因子に持たない正の偶数でかつ[p/4]が奇数になるような数であり,
 さらにf(x)=x^2-5x+p/6=0がx>0に2つの解を持つ.
このとき,実数の方程式f(x)=0の解になりうる最小の数を求めよ.
192132人目の素数さん:2010/12/20(月) 21:48:56
{(p-2)-4√(p-1)}/8∈R が正の代数的整数となる
最小の自然数pに対して、上記の数を求めよ。
193132人目の素数さん:2010/12/20(月) 22:01:09
黒鉄が凡人でないことを考慮すると、
p=4k+2, k=2l+1 などはあまり明らかでないほうがよい。
なんらかの飛躍により、この2つが得られたと思うべき。
(明らかなのにpがなんとかこうとかとワザワザ言うのはおかしい)
ここまでにでた例は全て簡単すぎる。
しかも >>188>>191 だいたい似ている。
>>191 は数IAっぽいが pが明らかすぎる。
>>188 は数IAぽくない書き方をしている。
194132人目の素数さん:2010/12/20(月) 22:11:04
>>188,>>192
p=6で(1-√5)/2はx^2-x-1の根では?

ダメ出しされてしまったので,もう少しよいのを考えてみよう.出来るかわからないけど.
195132人目の素数さん:2010/12/20(月) 22:16:42
>>193
東大入試レベルにとどめておけよw
196132人目の素数さん:2010/12/20(月) 22:28:57
>>194
正と書いてあるので p=6は弾かれる。((1-√5)/2は負である!)
同様にp=14も弾かれる。
197132人目の素数さん:2010/12/20(月) 22:34:04
>>196
なるほど,失礼しました.
198132人目の素数さん:2010/12/20(月) 23:08:04
「よって『解』は」で受けるような問いの種類はかなり限られると思うのだが。
199132人目の素数さん:2010/12/20(月) 23:08:25
ヨロシクは素数である. (4649)
このような面白い素数をなるべくたくさんみつけよ。
200132人目の素数さん:2010/12/20(月) 23:14:41
訳注: 田代はヤク中である
201132人目の素数さん:2010/12/20(月) 23:18:39
>>200
ぐうにゅうふいたww
202132人目の素数さん:2010/12/20(月) 23:47:08
>>199
8753(はなこさん) は素数
203132人目の素数さん:2010/12/21(火) 00:30:19
0721ならTENGA
204132人目の素数さん:2010/12/21(火) 00:33:09
>>203
それは素数じゃねえぞ
0127なら素数なんだけどね??
205132人目の素数さん:2010/12/22(水) 17:14:43
答えが(5-√21)/2になる問題を考える時点で超高難度
k=2l(エル)+1みたいな要素入れるのなんて無理
206132人目の素数さん:2010/12/22(水) 17:44:31
スレが伸びなくなってきたな。
新作問題が出ない。
数版の限界か?
207132人目の素数さん:2010/12/22(水) 18:44:07
        ゴガギーン
             ドッカン
         m    ドッカン
  =====) ))         ☆
      ∧_∧ | |         /          / ̄ ̄ ̄ ̄ ̄ ̄ ̄ ̄ ̄ ̄ ̄
     (   )| |_____    ∧_∧   <  おらっ!出てこい>>1
     「 ⌒ ̄ |   |    ||   (´Д` )    \___________
     |   /  ̄   |    |/    「    \
     |   | |    |    ||    ||   /\\
     |    | |    |    |  へ//|  |  | |
     |    | |    ロ|ロ   |/,へ \|  |  | |
     | ∧ | |    |    |/  \  / ( )
     | | | |〈    |    |     | |
     / / / / |  /  |    〈|     | |
    / /  / / |    |    ||      | |
   / / / / =-----=--------     | |
208132人目の素数さん:2010/12/22(水) 20:51:00
>>206
2chにおいて板はばんって読むんじゃなくていたってよむんだぜ?
209132人目の素数さん:2010/12/22(水) 22:15:10
ハァ?他力本願なユトリは精神病院でも行けよ〜
210132人目の素数さん:2010/12/22(水) 22:19:08
その他力本願の使い方は誤用
211132人目の素数さん:2010/12/22(水) 22:55:33
212132人目の素数さん:2010/12/22(水) 22:59:11
>>211
個人のブログがソースかよ
213132人目の素数さん:2010/12/22(水) 23:34:48
バカか?文系の話するスレじゃねぇから
誤用かどうかは個人個人のブログでやってろよっていう
ソースだろ?もしかしておまえそれ反論でもしてるつもりなのか?
214132人目の素数さん:2010/12/22(水) 23:42:13
>>213
そのブログにも辞書に誤用だって載ってるって書いてあるのに
215132人目の素数さん:2010/12/22(水) 23:56:59
ソースは誤用
216132人目の素数さん:2010/12/23(木) 00:33:47
おっやみです^^

@x+y+z=0を満たすような任意の実数x,y,zに対して
(x^{m+n}+y^{m+n}+z^{m+n})/(m+n)=(x^m+y^m+z^m)(x^n+y^n+z^n)/mn
が成立するような整数m,nの組を全て求めよ

A 以下の条件を満たす正の整数(a,x,y,n,m)の組を全て求めよ
a(x^n−x^m)=(ax^m−4)y^2, m≡n (mod 2) かつ ax は奇数

B 自然数Nの10進表示を a[1]a[2]..a[n] 等とする。
N = (a[1]+2)*(a[2]+2)*...*(a[n]+2)
を満たすような自然数Nを全て求めよ。

C フィボナッチ数列類似の多項式列{F(n)}を次のように定義する:
F(0) = 0, F(1) = 1, F(n+2) = F(n+1) + x^n F(n).
このとき、任意の自然数nに対して、
F(5^n) は 1 + x + x^2 + … + x^(5^n - 1) で割り切れることを示せ。

Df^3=g^2+1 を満たすf,g∈C(X)\C は存在するか。

E自然数x_1, x_2, ..., x_nが
1/x_1+1/x_2+...+1/x_n=1 を満たすとき
x_1+x_2+...+x_n の最大値を求めよ。
217132人目の素数さん:2010/12/23(木) 00:35:14
Fa^a*b^b=c^c を満たす整数a,b,c>1の組は無限に存在するか。

G4桁の自然数Mをp進法において反転させたものをNとする。
M=2N が成立するような正整数p,M(p>1)の組を全て求めよ。

H各整数n≧6に対して、τ(n!)|n! が成立することを示せ。

Ia^2+b^2+b^2=2(ab+bc+ca)+p^n を満たすような
素数p,整数n,平方数a,b,cの組(p,n,a,b,c)は存在するか。
(ここでは平方数はm^2(m:正整数)の形で表現される数)

J|2^x-3^y|=p を満たす整数x,yの組が存在するような
p≡-7(mod 24)なる素数pはp=17以外に存在するか。

Kx^4+y^4=z^2+1 を満たす整数x,y,zの組は無限に存在するか。
(x^4+y^4=z^2 が非自明な整数解を持たないことは良く知られている)
218132人目の素数さん:2010/12/23(木) 00:43:18
Iにタイプミスが^^;;
×a^2+b^2+b^2
〇a^2+b^2+c^2
219132人目の素数さん:2010/12/23(木) 03:10:00
存在しない。
220132人目の素数さん:2010/12/23(木) 03:20:00
x^4+1^4=(x^2)^2+1。
221132人目の素数さん:2010/12/23(木) 03:38:41
すみません^^; Kは訂正します^^;
寝ている間に問題の誤りに気づいてしまいました^^;

Kx^4+y^4=z^2+1 を満たす整数x,y,z>1の組は無限に存在するか。
(x^4+y^4=z^2 が非自明な整数解を持たないことは良く知られている)

1を許してしまうと >>220 の例により無限にあるとなります^^;
222132人目の素数さん:2010/12/23(木) 16:55:30
>>221
Jの設定の理由は? なぜ mod24? なぜ2と3の累乗? なぜ -7?
理由があるなら(あるとおもうけど)教えてちょ。
223132人目の素数さん:2010/12/23(木) 19:27:32
>>222
a^m-b^n(a,bはa,b>1なる整定数)はどんな素数を取りうるか
という問題がさきにあったわけです。
そこで、a,bとして最小なペアはa=2,b=3があるわけです。
絶対値がついているのは取りうる値の個数を増やすため。
このままでは問題はとても難しいので、
さきに素数をmodで限定することを考えるわけです。
mod mで考えるとして、mはなるべく小さいほうがいいでしょう。

このとき、次のことがいえるわけです。

任意に互いに素な整数m,s(1<m<24)の組を固定したとき、
|2^x-3^y|=p を満たす整数x,yの組が存在するような
p≡s(mod m)なる素数pは少なくとも2つ存在する。
(おそらく無限に存在するのだとはおもいますが、
未解決問題が絡むゆえ、そのことの証明は困難でしょう)

そのことに注目したので、mod 24で考えているわけです。
-7の理由は単純です。p≡-7(mod 24)でないとすれば
条件を満たすpは少なくとも2つ存在するからです。
224132人目の素数さん:2010/12/23(木) 19:35:21
mod mなどと書いてしまいましたが これは別の文字を使うべきでした^^;
-7の理由も書きなおしておきます^^;
「-7≡s を満たさない24と互いに素な整数sを任意に固定するとき、
|2^x-3^y|=p を満たす整数x,yの組が存在するような
p≡s(mod m)なる素数pは少なくとも2つ存在するから」
225132人目の素数さん:2010/12/23(木) 19:54:00
2=3^1-2^0
3=2^2-3^0
5=2^3-3^1
7=3^2-2^1
11=3^3-2^4
13=2^4-3^1
17=3^4-2^6
19=3^3-2^3
23=2^5-3^2
29=2^5-3^1
31=2^5-3^0
37=2^6-3^3
226132人目の素数さん:2010/12/25(土) 11:53:00
自作問題。いわゆる"初等的な"解答があるかは分からない。

問題:
Nから{0,1}への写像全体の集合族をXと置く。
Xの2元f , g:N → {0,1} に対して、f+g∈Xを次のように定義する。

(f+g)(n):= 「f (n)+g(n) を2で割った余り」 (n∈N)

次に、f∈Xに対して

E(f):=lim[n→∞]#{ i|1≦i≦n, f (i)=1}/n

と置く(fによっては、E(f)は存在しないこともある)。
次が成り立つことを示せ。


 「Xの任意の点列{f_n}_{n∈N}に対して、あるg∈Xが存在して
  E(g+f_n)=1/2 (n∈N)が成り立つ。」
227132人目の素数さん:2010/12/25(土) 16:01:57
集合論の問題か。1/2であるこも本質ではないし、
{0,1}も別に本質ではない。
228132人目の素数さん:2010/12/25(土) 16:17:48
>>227
{0,1}は重要では??
適当に構造をみて 良く知られた定理をアプライする問題だとおもう。
それをあてるクイズゲームだとおもえばいい。
229132人目の素数さん:2010/12/25(土) 16:47:49
(問題)
XをR^nのコンパクトな凸集合とし、Int(X)≠φとする。
Xの境界はR^nの球面と同相であることを証明せよ。
230132人目の素数さん:2010/12/25(土) 16:57:27
>>226
選択公理は必要ですか?
選択公理がなくとも解けますか?
231226:2010/12/25(土) 17:26:24
>>227
{0,1}である必要は無いけど、「二元集合」であることは大切。
そして、「二元集合」の場合は「1/2」が大切で、1/2以外の値だと
反例が作れる。例えば「0」に置き換えて

 「Xの任意の点列{f_n}_{n∈N}に対して、あるg∈Xが存在して
  E(g+f_n)= 0 (n∈N)が成り立つ。」

という主張を考えてみると、実はこれは成り立たない。

f_1(i) = 1 (i∈N)
f_n(i) = 0 (i∈N,n≧2)

としてXの点列{f_n}_{n∈N}を定義すると、これが反例になっていて、
どのようなg∈Xをとってきても「 E(g+f_n)= 0 (n∈N) 」が
成り立つようには出来ない。
232132人目の素数さん:2010/12/25(土) 17:35:13
>>226
用意してある解答例は選択公理と独立?
233226:2010/12/25(土) 17:40:30
>>230,>>232
使ってる道具の性質上、可算選択公理は使ってるはず。

でも、いわゆるあの「選択公理」とは独立。
234132人目の素数さん:2010/12/26(日) 04:02:32
問題5.

pを3以上の素数とする。
(1) nを正の整数とするとき、Σ[i=1_n] i・p^i を p,n を用いて表せ。
(2) p-1以下の正整数kに対して、正整数a(k)をb(k)を
 b(k) = {Σ[j=1,p-1] j^k}・a(k) と定める。
 1≦k≦p-1 の全てのkについてb(k)がpの倍数であるとき a(k)も全てpの倍数だ
と言えるか?

東大入試作問者スレ19-153
235132人目の素数さん:2010/12/26(日) 04:21:55
>>234

(1) 与式を f_n(p) とおく。
 (p-1)f_n(p) = (p-1)pΣ[i=1,n] i・p^(i-1)
  = p・{n・p^n - Σ[i=1,n] p^(i-1)}
  = p・{n・p^n - (p^n - 1)/(p-1)}
  = p・{n・p^(n+1) - (n+1)・p^n + 1)}/(p-1),
∴ f_n(p) = p・{n・p^(n+1) - (n+1)・p^n + 1)}/(p-1)^2,

(2)
 k=p-1 のときは j^(p-1)≡1 (mod p) より
 Σ[j=1,p-1] j^(p-1) ≡ p-1 ≡ -1 (mod p)
よって
 p|b(p-1) ⇒ p|a(p-1)
しかし 1≦k≦p-2 のときは
 Σ[j=1,p-1] j^k ≡ 0 (mod p)
の場合もあるので……
236132人目の素数さん:2010/12/26(日) 04:58:20
>>234
(2) あきらかにいえない
>>235 に正しい解答があるが、
敢えてはっきりというならば、
たとえば a(p-1)=p, a(i)=1 (1≦i<p) としても
問題の条件を満たすことがいえる。
237132人目の素数さん:2010/12/26(日) 22:13:08
>>216
C
F(n;x) の次数は [ ((n-1)/2)^2 ]

>>217
I
平方数の定義により
 a=A^2, b=B^2, c=C^2 (A,B,Cは正整数)
よって
 p^n = a^2 + b^2 + c^2 -2(ab+bc+ca) = -(A+B+C)(-A+B+C)(A-B+C)(A+B-C),
右辺の4つの因子はすべて偶 または すべて奇。
238132人目の素数さん:2010/12/26(日) 23:41:05
>>237
C,Iのいずれも解答の途中だね
そこまでなら皆わかっているハズ(`・ω・´)
239132人目の素数さん:2010/12/27(月) 01:00:00
0<a≦b≦c。
c−a−b<a+c−b≦b+c−a<a+b+c。
(c−a−b)+(a+b+c)=(a+c−b)+(b+c−a)。
240御手洗景子:2010/12/27(月) 02:48:02
>>217
I 先に結論を書くならば、
「そのような組はギリギリ存在しない」
以下にそのことの証明を述べることにする。

a≧b≧c と仮定する。(そう仮定しても一般性を失わない)
a=x^2, b=y^2, c=z^2 を満たす正整数x,y,zの組を取る。
d=gcd(x,y,z)として、x=ds,y=dt,z=du を満たす正整数s,t,uの組を取る。
a^2+b^2+c^2-2(ab+bc+ca)=(x+y+z)(x+y-z)(x-y+z)(x-y-z) と分解できて、
右辺=d^4(s+t+u)(s+t-u)(s-t+z)(s-t-u) となるので、とくに、
(s+t+u)(s+t-u)(s-t+z)(s-t-u) はpの巾であることがいえる。
ここで、s+t+u>s+t-u≧s-t+z>s-t-u>0 に注意しておく。

(@) p≧3 であるときを考える。
p|s+t+u,s+t-u,s-t+z であることから、p|2s,2t,2u がいえる。
pが奇素数であることとあわせて、p|s,t,u がいえる。矛盾。

(A) p=2 であるときを考える。
s+t+u≡s-t-u (mod 2) であるので、2|s-t-u となる。(s-t-u>0に注意)
よって、4|s+t+u,s+t-u,s-t+u であることがいえる。
そこから、4|2s,2t,2u が得られるので、2|s,t,u がいえる。矛盾。

解答のポイントを箇条書きすると...
1) 簡単な因数分解に気づく
2) 3つの数のGCDを取る
3) 大小関係に気をつける
4) pが奇素数かどうかでわける
241132人目の素数さん:2010/12/29(水) 00:46:35
a^2+b^2=c^3+1
242132人目の素数さん:2010/12/29(水) 02:59:33
>>241
 (|a|,|b|,c) = (1, n^3, n^2)
 (|a|,|b|,c) = (8m^6 -1, 4m^3, 4m^4)
243132人目の素数さん:2010/12/29(水) 03:12:12
残った問題は難しいのしかないな
244132人目の素数さん:2010/12/29(水) 14:00:00
p^d=c−a−b。
p^e=a+b+c。
p^f=a+c−b。
p^g=b+c−a。

p^d+p^e=p^f+p^g。
p^e,p^f,p^gはp^(d+1)の倍数でp^dはp^(d+1)の倍数でないので矛盾。
245132人目の素数さん:2010/12/29(水) 17:18:03
>>244
GJ
246132人目の素数さん:2010/12/30(木) 04:36:31
御手洗景子嬢晒し上げ

□投稿者/ 御手洗景子 @ 一般人(4回)-(2010/06/17(Thu) 21:05:50)
http://mixi.jp/show_friend.pl?id

(1)1/(1-x-x^2)=Σ(n=0〜∞)a_n(x^n)に対して、a_0,・・・,a_10を求め、その規則性を見つけよ。そして、どうしてその規則性が成り立つのか説明せよ。
(2)(2-x)/(1-x-x^2)Σ(n=0〜∞)a_n(x^n)に対して、a_0,・・・,a_10を求め、その規則性を見つけよ。そして、どうしてその規則性が成り立つのか説明せよ。
(3)(x^2)/(1-x-x^2-x^3)Σ(n=0〜∞)a_n(x^n)に対して、a_0,・・・,a_10を求め、その規則性を見つけよ。そして、どうしてその規則性が成り立つのか説明せよ。

できるだけ、詳しく教えてください。お願いします。
247132人目の素数さん:2010/12/30(木) 05:02:11
>>246
別人だろjk
偶然名前が一致したか、恨みがあるかの2択しかありえない。
248132人目の素数さん:2010/12/30(木) 10:02:12
相当マルチしまくりだったからなw
249132人目の素数さん:2010/12/30(木) 20:40:24
>>231
もしよろしければ解答おねがいできますか?
過疎ってきているから、加速燃料になるはz...
250132人目の素数さん:2010/12/30(木) 22:42:08
>>246

両辺に 分母を掛けて x^k の係数を比較する。
(1) a_0 =1, a_1 -a_0 =0, a_k -a_(k-1) -a_(k-2) =0, (k≧2)
  ∴ a_n = F_(n+1),  Fibonacci数列。
(2) a_0 =2, a_1 -a_0 =-1, a_k -a_(k-1) -a_(k-2) =0, (k≧2)
  ∴ a_n = F_n +2F_(n-1),
(3) a_0 =0, a_1 -a_0 =0, a_2 -a_1 -a_0 =1, a_k-a_(k-1)-a_(k-2)-a_(k-3) =0, (k≧3)
  ∴ a_n = T_(n-3) Tribonacci数列。
251250:2010/12/30(木) 22:45:40
>>246 訂正

(3) a_n = T_(n-1) 
252132人目の素数さん:2010/12/31(金) 02:05:23
>>221

K の例
(x,y,z) =
 ( 5, 7,  55)
 (13, 13, 239)
 (27, 37, 1551)

 ( 8, 17, 296)
 (32, 17, 1064)
 (28, 47, 2344)
 (76, 47, 6184)
 (44, 63, 4416)
 (64,111,12984)

 (22, 31, 1076)
 (46, 31, 2324)
 (46, 97, 9644)

さて、無限にあるかどうか…
253132人目の素数さん:2010/12/31(金) 02:45:34
そんだけ解あるならば無限にありそうだね。
それでも証明はかなり厳しそうだけど。

それよりFのほうが真っ暗だとおもう。1つすらみつけられない。
問題文から察するに解は1つはありそうだけど。。。もしくは罠?
254132人目の素数さん:2010/12/31(金) 02:47:39
素因数
255132人目の素数さん:2010/12/31(金) 03:16:05
まあその2題に関しては解答は用意されているわけだし問題なかろう。
256132人目の素数さん:2010/12/31(金) 03:17:27
C
E
は削除しろ
257132人目の素数さん:2010/12/31(金) 03:34:59
>>256
Cはなぜ? かなり難しい問題であるのはわかるけど...
258132人目の素数さん:2010/12/31(金) 06:28:09
>>253

Fの類題
a^a*b^b*c^c=d^d を満たす整数a,b,c,d>1の組は存在するか?


259132人目の素数さん:2010/12/31(金) 07:13:05
>>258
存在する。 a=12,b=12,c=8,d=24
しかしながら、F自体は1つも解がみつけられない。
ということで 解がそもそも存在するかどうか教えてくれ
>>1
260132人目の素数さん:2010/12/31(金) 15:00:00
a=2^(2^(2n+1)−(n+1)2^(n+1))(2^n−1)^(2^(n+1))。
b=2^(2^(2n+1)−(n+1)2^(n+1)+2n)(2^n−1)^(2^(n+1)−2)。
c=2^(2^(2n+1)−(n+1)2^(n+1)+n+1)(2^n−1)^(2^(n+1)−1)。
261132人目の素数さん:2010/12/31(金) 15:18:51
>>257
死ね屑
262132人目の素数さん:2010/12/31(金) 17:42:40
>>258
 存在する。 a=b=c=4, d=8
263132人目の素数さん:2010/12/31(金) 18:47:09
Fの解の見つけ方。

d=gcd(a,b,c)とおく。a=ds,b=dt,c=du とおく。
すると そのもとで a^a*b^b=c^c ⇔ d^(s+t)*s^s*t^t=d^u*u^u
ここで たとえば u=s+t-1 とすれば、d*s^s*t^t=u^u となるので、
s^s*t^t|u^u=(s+t-1)^(s+t-1) を満たす正整数s,tを見つければ十分である。
s+t-1 が s,tで割り切れてくれたら十分であるのだがそれは不可能。
そこで、s=a^2,t=b^2 なる置き換えを行う。a-b=1 のときを考えれば、
aが2の累乗にならざるをえないことがいえる。それから >>260 を得る。
(共通項を抹消すると 2^(2n), (2^n-1), 2^(n+1)*(2^n-1) が残る)
264132人目の素数さん:2010/12/31(金) 18:48:12
あ、ちなみに わたしは 260ではないので、彼がどうやって解を得たのかは知らんよw
265132人目の素数さん:2010/12/31(金) 18:48:50
C
E
は削除しろ
266132人目の素数さん:2010/12/31(金) 19:22:12
>>265
提案者?
267132人目の素数さん:2010/12/31(金) 20:16:32
>>241
(|a|,|b|,c) = (4, 7, 4)
       (17, 21, 9)
       (6, 36, 11)
       (12, 51, 14)
       (21, 48, 14)
268 【凶】 【1175円】 株価【30】 u:2011/01/01(土) 00:45:18
今までで解かれてなくて^^が削除したのは、
出題者が取り下げたの以外で1つあった。(旧スレ@)
長期間だれも解かなかったから、という理由だったが
今やリストの大半が長期間放置されているね。
個人的にはCEはまだ考えているから残しておいてほしいけど。
269 【小吉】 【1002円】 株価【30】 :2011/01/01(土) 00:47:28
つまらん
270132人目の素数さん:2011/01/05(水) 22:30:15
暇つぶし用の易問だから各5分以内に解いてね

@2より大きい自然数nを考える。n進数で2011とあらわされる自然数が2011の倍数となるnを全て求めよ
Akを正整数とする。(a^2+k)/2011が整数になる正整数aが無限に存在するようなkを全て求めよ。
Bnを正整数とする。(a^n+1)/2011が整数となる正整数aが無限に存在するnを全て求めよ。
271132人目の素数さん:2011/01/06(木) 03:44:42
手計算で5分以内は厳しい。2011は素数。p=2011とおく。

@2X^3+X+1∈F_p[X] がX-10を因子にもつことは明らか。
残り2つの因子を決定するのは少々計算が面倒。

A (-k/p)を計算すればよいがこれもやはり面倒なだけである。

B (-1/2011)=-1より、nが偶数のときは条件を満たす正整数aは存在しない。
nが奇数のときは 2011|a+1 なる正整数aを取ればよい。(無数に取れる)
272132人目の素数さん:2011/01/06(木) 18:30:56
1/x1 + 1/x2 + 1/x3 + … + 1/xn <1
(但し、x1,x2,x3,…,xnは正の整数)
を満たす左辺の最大値を与えるx1〜xnは、
x1=2 ,x(n+1)=Π(k=1〜n)xk +1

という定理、結果はよく見るが、
証明は知らない。(名前すら知らないので調べようがない)

これを認めればEは解けるか?
273132人目の素数さん:2011/01/06(木) 19:05:46
>>272
あのさあ、その問題文で検索すると
ttp://okwave.jp/qa/q4770106.html
がヒットするんだけど
なめてんの?
274132人目の素数さん:2011/01/06(木) 19:12:12
ぺろぺろ
275132人目の素数さん:2011/01/06(木) 19:35:24
>>272
それは誰でも知っているとおもうけど、
それ使ったところでなにか発展しますか。
276132人目の素数さん:2011/01/06(木) 19:45:33
C
E
は削除しろ
277132人目の素数さん:2011/01/06(木) 23:07:56
>>276
Cはなぜ? かなり難しい問題であるのはわかるけど...

提案者?
278132人目の素数さん:2011/01/06(木) 23:26:56
まあまあ
残った問題で解けそうなものから潰していこうぜ。
279132人目の素数さん:2011/01/06(木) 23:40:56
>>277
お前それしかいえんのか?
280132人目の素数さん:2011/01/07(金) 00:10:54
■■■■■■■■■■■■■■■■■■■■■■■■■■■■■■■■■■■■■■■
■■■■■■■■■■■■■■■■■■■■■■■■■■■■■■■■■■■■■■■
■■■■■■■ このスレは他板・他スレ運営妨害の非常に悪質糞スレの為に ■■■■■■
■■■■■■■反感を買って終了しました。 皆様のご愛顧有難う御座いました■■■■■■
■■■■■■■■■■■■■■■■■■■■■■■■■■■■■■■■■■■■■■■
■■■■■■■■■■■■■■■■■■■■■■■■■■■■■■■■■■■■■■■


281132人目の素数さん:2011/01/07(金) 13:17:07
>>271
面倒でも全部書いて
282132人目の素数さん:2011/01/09(日) 08:12:56
>>270 >>281   p=2011 とおく。

@ 2X^3 +X +1 = 2(X-10)(X-109)(X+119) + (13X-129)p
       ≡ 2(X-10)(X-109)(X+119),   (mod p)

・n ≡ 10 (mod p) のとき、
 2n^3 +n +1 = 2(n-10)^3 + 60(n-10)^2 + 601(n-10) + p ≡ 0, (mod p)

・n ≡ 109 (mod p) のとき、
 2n^3 +n +1 = 2(n-109)^3 + 654(n-109)^2 + 71287(n-109) + 1288p ≡ 0, (mod p)

・n ≡ -119 (mod p) のとき、
 2n^3 +n +1 = 2(n+119)^3 - 714(n+119)^2 + 84967(n+119) - 1676p ≡ 0, (mod p)
283132人目の素数さん:2011/01/09(日) 15:50:51
>>282
具体的に F_p多項式 分解したのは
存在の 証明が 他に無いから
284132人目の素数さん:2011/01/10(月) 19:17:49
↓数オリスレから拾ってきたw
↓おまえらなら瞬殺だよな?w

1.1以上9以下の整数の組(a,b,c,d)であって、
  0<b-a<c-b<d-c をみたすものはいくつあるか。

2.2011以下の正の整数のうち3で割って1余るものの総和をA、
  3で割って2余るものの総和をBとする。
  A-Bを求めよ。

3.相異なる7以下の正の整数a,b,c,d,e,f,gを用いて
  a*b*c*d+e*f*gと表せる素数を全て求めよ。

5.2011以下の正の整数のうち、一の位が3または7であるもの
  すべての積をXとする。Xの十の位を求めよ。

8.2桁の正の整数x、yがあり、xの十の位はyの一の位と等しく、
  yの十の位はxの一の位と等しい。また、xとyの積をPとすると、
  Pは4桁の整数になり、Pの下2桁を2桁の整数とみなしたものは
  上2桁を2桁の整数とみなしたものより23大きくなった。
  このとき、Pの値を求めよ。

10.正の整数に対して定義され、正の整数値を取る関数fであって、任意の正の整数x、yに対して
     (x+y)f(x) <= x^2 + f(xy) + 110
   をみたすものを考える。このとき、f(23)+f(2011)としてありうる最小の値と最大の値を求めよ。
285132人目の素数さん:2011/01/10(月) 23:27:48
12.nを2以上の整数とする。非負実数a1,…,anが a1+…+an=1 をみたすとき、
   
    {Σ(i=1〜n)(i*ai)}*{Σ(i=1〜n)(ai/i)}^2

   としてありうる最大の値を求めよ。


数オリスレでは12が誰も解けないみたいだから、おまえら助けてやれや
286132人目の素数さん:2011/01/10(月) 23:49:03
現役離れたおっさんには無理だにゃ〜
287132人目の素数さん:2011/01/11(火) 00:19:54
^^の人に期待か
288132人目の素数さん:2011/01/11(火) 00:37:16
>>285
1
289132人目の素数さん:2011/01/11(火) 00:40:56
a1=1
a2=…=an=0
290132人目の素数さん:2011/01/11(火) 00:48:11
>12番が1とかいう噂が漂っているが証明でもないくせにそんな自明解だったら財団に切れるぞ
291132人目の素数さん:2011/01/11(火) 01:29:57
>>290
勝手に切れてろ
292132人目の素数さん:2011/01/11(火) 06:26:45
>>288
an = (1−1/n)*(1/3)
a1 = 1−an
a2 = … = a_{n-1} = 0

とすると、nが十分大きいとき与式は1を超える。
293132人目の素数さん:2011/01/11(火) 09:07:16
>>285
全展開後の相加→相乗平均から、
a1= …= anのとき最大値 n {Σ(i=1〜n)(i)}*{Σ(i=1〜n)(1/i)}^2 を取る?
294132人目の素数さん:2011/01/11(火) 09:43:44
368 名前:132人目の素数さん[sage] 投稿日:2011/01/11(火) 07:11:40
<さらに転載>

12
4(n+1)^3/27n^2
a_2~a_n-1を0として3次関数の増減調べただけ。
たしかa_1=(2n-1)/3(n-1) a_n=(n-2)/3(n-1)。
でも(僕のあてにならない感覚ですが)感覚的にこれは正しそう。証明が無理・・・
これ以上の式が答えであることは確実です。
295132人目の素数さん:2011/01/12(水) 09:19:53
上から目線の例の人もお手上げか
296132人目の素数さん:2011/01/12(水) 09:23:59
というかまだ今年はあらわれてないな
297132人目の素数さん:2011/01/12(水) 14:18:53
>>271が例の人だろw
298132人目の素数さん:2011/01/12(水) 18:33:40
最大値が存在することは簡単に証明できるけど、(ほぼ明らかだが)
具体的にそれがなにかはあの問題の場合は難しいきがする。

スレチ-不等式スレ池
299132人目の素数さん:2011/01/12(水) 20:19:25
>>297
^^じゃないだろ
300132人目の素数さん:2011/01/12(水) 23:12:20
^^の人と上から目線の例の人は違うよ
301132人目の素数さん:2011/01/15(土) 08:51:26
>>284

1. 1≦b-a, 2≦c-b, 3≦d-c,
 辺々たすと、 6≦d-a,
 d-a=6: (1,2,4,7) (2,3,5,8) (3,4,6,9)
 d-a=7: (1,2,4,8) (2,3,5,9)
 d-a=8: (1,2,4,9) (1,2,5,9)

2.
 A = 1 + 4 + 7 + ・・・・・ + 2011,
 A =   1 + 4 + ・・・・・ + 2008 + 2011,
 A =   1 + 4 + ・・・・・ + 2008 + 2011,
相加平均して
A = 1/3 + 2 + 5 + ・・・・・ + 2009 + 2011*(2/3)
 = 1/3 + B + 2011*(2/3)
 = B + 1341,

3.
 題意により、abcd と efg は互いに素。
 ∴ 6を含む項はまた2,3,4も含むはずだから、
 {a,b,c,d} = {2,3,4,6}, {e,f,g} = {1,5,7} p=144+35=179,

5.
 (10a+3)(10a+7) = 100a(a+1) + 21 ≡ 21 (mod 200)
 (100b+21)(100c+21)(100d+21)(100e+21)(100f+21) ≡ 1 (mod 100),
 ∴ 50ごとに1に戻るので 2000以上の因数を考えればよく、
 2003x2007 = 4020021 ≡ 21 (mod 100) より 2

8.
 P = 3154 = 38 * 83
302132人目の素数さん:2011/01/15(土) 11:55:02
>>301
5の無駄を省いたバージョン

φ(100)=40, 任意の非負整数nに対して、(10n+3)(10n+7)≡21 (mod 100)
したがって、問題の積≡21^(201)≡21^(40*5+1)≡21 (mod 100)
303132人目の素数さん:2011/01/15(土) 12:28:01
任意に a+b=10^k を満たす正整数a,b,kの組を固定する。(a,bはk桁とする)
3k+1桁以下の正整数のうち下k桁がaまたはbであるような数の総積をPとする。
このとき、P-1は10^(2k)で割り切れることを証明せよ。
304訂正:2011/01/15(土) 12:30:15
(10,ab)=1 を条件に追加しておく。 これでOK
305132人目の素数さん:2011/01/15(土) 19:56:58
>>285
4(n+1)^3/27n^2
306132人目の素数さん:2011/01/15(土) 22:46:20
>>285

つまり、与式の最大に関しては、 (x,0,・・・・,0,1-x) を考えれば十分だから
 (与式) ≦ {x + n(1-x)}{x + (1-x)/n}^2
 = {x + n(1-x)}{[1 +(n-1)x]/2}^2・(2/n)^2
 ≦ {(n+1)/3}^3・(2/n)^2   (← 相乗・相加平均)
 = 4(n+1)^3/(27n^2),     >>305
等号成立は x = (2n-1)/(3(n-1)) のとき
307132人目の素数さん:2011/01/15(土) 22:48:51
>>305
>>306
どうしてa1, an以外は0なの?
308132人目の素数さん:2011/01/15(土) 23:04:27
>>306
『与式の最大に関しては、 (x,0,・・・・,0,1-x) を考えれば十分だから』

なぜ? それはかなり非自明な匂いがしますが・・・
309132人目の素数さん:2011/01/15(土) 23:14:31
そのレベルですか
まずはn=3の場合はやってみてはいかがですかな
310132人目の素数さん:2011/01/15(土) 23:28:20
>>305>>306
>>294で既に出てるよ。
311306:2011/01/15(土) 23:33:50
>>285 >>294 >>307-308

〔補題〕与式の最大に関しては、 (x,0,・・・・,0,1-x) を考えれば十分。

(略証)
 (与式) = F(a_1,a_2,・・・・,a_n)・G(a_1,a_2,・・・・,a_n)^2,
と書ける。ここに
 F(t_1,t_2,・・・・,t_n) = Σ(i=1〜n) i・t_i,
 G(t_1,t_2,・・・・,t_n) = Σ(i=1〜n) t_i/i,
いま
 x = {1/(n-1)}Σ(j=1〜n-1) (n-j)・a_j,
 1-x = {1/(n-1)}Σ(j=2〜n) (j-1)・a_j,
とおけば
 僥 = F(x,0,・・・・,0,1-x) - F(a_1,a_2,・・・・,a_n) = 0,
 僭 = G(x,0,・・・・,0,1-x) - G(a_1,a_2,・・・・,a_n)
   = (1/n)Σ(j=2〜n-1) {(n-j)(j-1)/j}・a_j > 0,
また
 y = {1/(n-1)}Σ(j=1〜n-1) (n-j)・a_j/j,
 1-y = {n/(n-1)}Σ(j=2〜n) (j-1)・a_j/j,
とおけば
 僥 = F(y,0,・・・・,0,1-y) - F(a_1,a_2,・・・・,a_n)
   = Σ(j=2〜n-1) (n-j)(j-1)・a_j > 0,
 僭 = G(y,0,・・・・,0,1-y) - G(a_1,a_2,・・・・,a_n) = 0,
いずれの場合も F・G^2 は増加する。
よって頭書の結論を得る。(終)
312132人目の素数さん:2011/01/15(土) 23:34:00
証明が重要なわけで それが書かれていない時点でお察し。
313132人目の素数さん:2011/01/15(土) 23:34:59
>>312
晒しあげ
314132人目の素数さん:2011/01/15(土) 23:37:21
恥ずかしいヤツがいると聞いて飛んできたwwwwww
315132人目の素数さん:2011/01/15(土) 23:38:10
>>311
やるね今年の1年
316132人目の素数さん:2011/01/15(土) 23:42:31
遅い時間で良かったな
317132人目の素数さん:2011/01/15(土) 23:43:56
ほらはやく初等数論の問題うpれやゴラっ
318132人目の素数さん:2011/01/15(土) 23:53:16
>>312
自分はできてないくせになんで態度でかいん?
319132人目の素数さん:2011/01/15(土) 23:58:20
わからなくてイライラしてるから
320132人目の素数さん:2011/01/15(土) 23:58:25
不等式への招待 第5章
http://kamome.2ch.net/test/read.cgi/math/1287932216/

57 名前:132人目の素数さん[sage] 投稿日:2011/01/12(水) 21:00:00
1<m<n。
0≦a。
b=(n−m)a/(n−1)。
c=(m−1)a/(n−1)。
a=b+c。
ma=b+nc。
a/m≦b+c/n。

 (x+n(1−x))(x+(1−x)/n)^2
=(2n−2(n−1)x)(1+(n−1)x)^2/2n^2
≦((2n+2)/3)^3/2n^2
=4(n+1)^3/27n^2。

59 名前:132人目の素数さん[sage] 投稿日:2011/01/14(金) 05:00:00
1≦i≦n。
(i−1)(i−n)≦0。
n≦i(n+1−i)。
1/i≦(n+1−i)/n。

 Σ(ia(i))Σ(a(i)/i)^2
≦Σ(ia(i))Σ(((n+1−i)/n)a(i))^2
=Σ(ia(i))(((n+1)/n)Σ(a(i))−(1/n)Σ(ia(i)))^2
≦(4(n+1)^3/27n^2)Σ(a(i))^3。
321132人目の素数さん:2011/01/15(土) 23:58:29
だれができたとかできないとかどうでもよい。
ただそこに証明が書かれたかどうかが重要だ。
322132人目の素数さん:2011/01/16(日) 00:00:27
自分で書き足したら
323132人目の素数さん:2011/01/16(日) 00:00:28
さすが不等式の住人達。
59の回答はとても明快。
324132人目の素数さん:2011/01/16(日) 00:01:24
>>321
必死すなあ
325132人目の素数さん:2011/01/16(日) 00:02:03
つか常識的に考えて
>>312>>311 のレスをみていないよ。
だって わずか10秒の差だぜ?
326132人目の素数さん:2011/01/16(日) 00:02:36
>>323
示すべき式が違うわけだが
327132人目の素数さん:2011/01/16(日) 00:03:18
>>325
それがどうした?
328132人目の素数さん:2011/01/16(日) 00:04:34
答えだけでいいよ
329132人目の素数さん:2011/01/16(日) 00:04:41
>>326
ほんとうだ 57のアイデアでFAか
330132人目の素数さん:2011/01/16(日) 00:09:56
>>323
何がすごいって投稿時間
331132人目の素数さん:2011/01/16(日) 00:10:56
568 もしもの為の名無しさん[sage]:2011/01/13(木) 00:13:05
計算みすtった

a1=(2n-1)/3(n-1)
an=(n-2)/3(n-1)
(Σi(ai))(Σ(ai)/i)^2 = 4(n+1)^3/27n^2

こうかな。まぁどうでもいいや。たぶんこれであってる
332132人目の素数さん:2011/01/16(日) 00:32:11
321 132人目の素数さん[]:2011/01/15(土) 23:58:29
だれができたとかできないとかどうでもよい。
ただそこに証明が書かれたかどうかが重要だ。
333132人目の素数さん:2011/01/16(日) 00:37:00
>>306>>320 (不等式5-057) は相乗・相加平均を使っている。

>>294 (368) は与式をxの3次関数として、増減を調べている。おそらく、

 4{(n+1)/3}^3 - {x + n(1-x)}{nx + (1-x)}^2
= 4{(n+1)/3}^3 - {n - (n-1)x}{1 + (n-1)x}^2
= {(n+4)/3 + (n-1)x}{(2n-1)/3 -(n-1)x}^2 ≧ 0,

等号成立は x=(2n-1)/(3(n-1)) のとき。
334132人目の素数さん:2011/01/16(日) 01:36:20
よくやった
不等式スレに戻ってよいぞよ
335132人目の素数さん:2011/01/16(日) 17:58:26
ついでに・・・

〔系〕
 L>0, m>0 のとき
 F(a_1,・・・・,a_n)^L・G(a_1,・・・・,a_n)^m ≦ λ^L・μ^m,
 λ = (n+1)L/(L+m),
 μ = (n+1)m/(L+m)n,

(略証)
 >>311 より
 (左辺) ≦ F(x,0,・・・,0,1-x)^L・G(x,0,・・・・,0,1-x)^m
  = {x + n(1-x)}^L・{x + (1-x)/n}^m
  = {n - (n-1)x}^L・{[1+(n-1)x]/n}^m
  = (L^L)・{[n - (n-1)x]/L}^L・{[1+(n-1)x]/m}^m・(m/n)^m
  ≦ (L^L)・{(n+1)/(L+m)}^(L+m)・(m/n)^m     (← 相乗・相加平均)
  = λ^L・μ^m,
336132人目の素数さん:2011/01/17(月) 06:31:57
>>311 の訂正
 最後から4-5行目

 僥 = F(y,0,・・・・,0,1-y) - F(a_1,a_2,・・・・,a_n)
   = Σ(j=2〜n-1) {(n-j)(j-1)/j}・a_j > 0,
337132人目の素数さん:2011/01/18(火) 02:18:47
>>216@
x+y+z=0, xy+yz+zx=a, xyz=b と置き、整数nに対しS、[n]をS[n]=x^n+y^n+z^n で定める。
S[1]=x+y+z=0 , S[2]=x^2+y^2+z^2=-2a , S[3]=3b であり、
S[n+2](x+y+z)={x^(n+2)+y^(n+2)+z^(n+2)}(x+y+z) を整理することで
S[n]は漸化式 S[n+3]=-aS[n+1]+bS[n] を満たすことが示せる。これより
nを自然数とした場合、S[n]はa,bの多項式で、S[8]までは次のようになる。
S[4]=2a^2 , S[5]=-5ab , S[6]=-2a^3+3b^2 , S[7]=7a^2b , S[8]=2a^4-8ab^2
S[m+n]/(m+n)=S[m]S[n]/(mn)がa,bによらず成り立つ(m,n)を全て列挙すればよい。

aの多項式f(a)に対し、aについて係数が0でない最高次の項を取る操作を<f,a>と書くことにする。
(<S[6],a>=-2a^3 , <S[6],b>=3b^2 , <S[7],a>=<S[7],b>=7a^2b 等 f≡0の場合は定義しない事とする。)
<S[n],a> , <S[n],b> について次が成り立つ。 (以下いずれもm≧1とする)
<S[2m],a>={(-1)^m}2*a^m , <S[2m+1],a>={(-1)^(m-1)}(2m+1)a^(m-1)b ,
<S[3m],b>=3b^m , <S[3m+1],b>={m(3m+1)/2}a^2b^(m-1) , <S[3m-1],b>=-(3m-1)ab^(m-1)
証明は帰納法による。m=1のときは上記S[2]〜S[4]により確認できる。
以下m≦kで成り立つと仮定する。
<S[2(k+1)+1],a>=<-aS[2k+1]+bS[2k],a>=<-a<S[2k+1],a>+b<S[2k],a>,a>
仮定より<S[2k],a>={(-1)^k}2*a^k , <S[2k+1],a>={(-1)^(k-1)}(2k+1)a^(k-1)b なので
<{(-1)^k}(2k+1)(a^k)b+{(-1)^k}2*(a^k)b,a>={(-1)^((k+1)-1)}(2(k+1)+1)a^((k+1)-1)b
となり、m=k+1でも成り立つ。<S[2(k+1)],a>についても同様に示せる。
<S[3(k+1)-1],b>=<-aS[3k]+bS[3k-1],b>=<-a<S[3k],b>+b<S[3k-1],b>,b>
仮定より<S[3k],b>=3b^k , <S[3k-1],b>=-(3k-1)ab^(k-1) なので
<S[3(k+1)-1],b>=<-3ab^k-(3k-1)ab^k,b>=<-(3k+2)ab^k,b>=-{3(k+1)-1}ab^k
となり、m=k+1でも成り立つ。<S[3(k+1)],b> , <S[3(k+1)],b> についても同様に示せる。
338132人目の素数さん:2011/01/18(火) 02:19:46
>>216@
まずm,nがともに自然数の場合を考える
m,nがともに奇数のとき、m',n'を自然数として、m=2m'+1 , n=2n'+1 とおくと
<S[m]S[n],a>/(mn)=<S[m],a><S[n],a>/(mn)={(-1)^(m'+n')}a^(m'+n'-2)b^2
<S[m+n],a>={(-1)^(m'+n'+1)2/(m+n)}a^(m'+n'+1)
より、aの最高次の項が異なるのでS[m+n]/(m+n)≠S[m]S[n]/(mn)
m,nがともに偶数のとき、m=2m' , n=2n' とおくと
<S[m]S[n],a>/(mn)=<S[m],a><S[n],a>/(mn)={(-1)^(m'+n')4/(mn)}a^(m'+n')
<S[m+n],a>={(-1)^(m'+n')2/(m+n)}a^(m'+n')
より4/(mn)=2/(m+n)となる(m,n)=(4,4)でのみaの最高次の項が一致する。
S[4]S[4]/(16)=a^4/4 , S[8]/8=a^4/4-ab^2 であるので
m,nがともに偶数のときも S[m+n]/(m+n)≠S[m]S[n]/(mn) となる。
m,nの偶奇がそれぞれ異なるときm=2m'+1 , n=2n' とすると
<S[m]S[n],a>/(mn)=<S[m],a><S[n],a>/(mn)=(-1)^(m'+n'-1)(2/n)a^(m'+n'-1)b
<S[m+n],a>={(-1)^(m'+n'-1)}a^(m'+n'-1)b
であるので mが奇数でn=2のときのみaの最高次の項が一致する。

m=6m'+1の場合
<S[m],b><S[2],b>/(2m)=m'/2a^3b^(2m'-1) , <S[m+2],b>/(m+2)=-6ab^(2m'+2)/(m+2)
bの最高次の項が異なるのでS[m]S[2]/(2m)≠S[m+2]/(m+2)
m=6m'+3の場合
<S[m],b><S[2],b>/(2m)=(-3/m)ab^(2m'+1) , S[m+2,b]/(m+2)=-ab^(2m'+1)
これはm=3のときのみ一致する。m=3,n=2,のときS[3+2]/5=-ab=S[3]S[2]/(3*2) より成り立つ。
m=6m'+5の場合
<S[m],b><S[2],b>/(2m)=a^2b^(2m'+1) , <S[m+2],b>/(m+2)=(m'+1)a^2b^(2m'+1)
これはm'=0、つまりm=5のときのみ一致する。m=5,n=2のとき
S[7]/7=a^2b=S[5]S[2]/(5*2) より成り立つ。
以上から、m,nが自然数の場合、(m,n)=(2,3),(2,5),(3,2),(5,2)の時のみ
成り立つことが示された。
339132人目の素数さん:2011/01/18(火) 02:20:36
>>216@
以下m,nが負の数を含む場合を考える。
x^{-m}+y^{-m}+z^{-m} ={(xy)^m+(yz)^m+(zx)^m}/(xyz)^m={(xy)^m+(yz)^m+(zx)^m}/b^m
となるので T[m]=(xy)^m+(yz)^m+(zx)^m と置くと
T[0]=3 , T[1]=xy+yz+zx=a , T[2]=(xy)^2+(yz)^2+(zx)^2=a^2
T[n+3]=aT[n+2]+b^2T[n] が成り立つ。n≧0でT[n]はa,bの多項式となる。
上記の等式から S[-m]=T[m]/b^m が常に成り立つ。また<T[n],a>=a^nが成り立つ。(n≧1)

m,nがともに負の場合、
S[m]S[n]/(mn)=T[-m]T[-n]/{b^(m+n)mn} , S[m+n]/(mn)=T[-(m+n)]/{b^(m+n)mn}
であるので T[-(m+n)]/(m+n)=T[-m]T[-n]/(mn) となるものを探せばよい。
-m=m'>0 , -n=n'>0 とし、T[m']T[n']/(m'n')とT[m'+n']/(m'+n')を考える。
<T[m'],a><T[n'],a]>/(m'n')=a^(m'+n')/(m'n') , <T[m'+n'],a>/(m'+n')=a^(m'+n')/(m'+n')
となるので、T[-(m+n)]/(m+n)=T[-m]T[-n]/(mn) が成り立つのならば
m'+n'=m'n' が成り立たなければならない。よってm'=n'=2の時のみ成り立ちうるが、
T[2]/2=a^2/2 , T[4]/4=a^4/4+ab^2 なので、常にT[-(m+n)]/(m+n)≠T[-m]T[-n]/(mn) となる。

m<0,1<nの場合(n=1のときは明らかに成り立たないので考えなくてよい。)
S[m]S[n]/(mn)=T[-m]S[n]/{b^m(mn)} について考える。最高次の項についての結果から
これはaについて -m+[n/2] 次の多項式となることがわかる。([ ] はガウス記号)
一方 S[m+n]/(mn) について、m+n≧1の場合、S[m+n]/(mn)は[(m+n)/2]次の多項式となる。
[(m+n)/2]<-m+[n/2 ]なので、 S[m]S[n]/(mn)≠S[m+n]/(mn) が常に成り立つ。
m+n<0の場合 S[m+n]/(mn)=T[-(m+n)]/{b^(m+n)mn} は -(m+n) 次の多項式となり
-(m+n)<-m+[n/2] なので、このときも明らかにS[m]S[n]/(mn)≠S[m+n]/(mn)

以上からm,nを整数とした場合も、解は(m,n)=(2,3),(2,5),(3,2),(5,2)のみである。
340sage:2011/02/02(水) 16:21:18
同じ3桁の数を2つ並べた数値は、7で割り切れる
例)
256256 ÷ 7 = 36608
954954 ÷ 7 = 136422

341132人目の素数さん:2011/02/02(水) 16:29:00
abc(1000+1)=abcx143x7
342132人目の素数さん:2011/02/02(水) 18:17:50
>>341
340です。
なるほど、簡単なんですね。
343132人目の素数さん:2011/02/02(水) 18:58:05
>>342
1001=10^3+1 が7で割り切れることは1秒でわかるから〇×クイズれべる
344132人目の素数さん:2011/02/02(水) 21:48:19
>>343
よー、おまえ復活したか、上から目線野郎
345132人目の素数さん:2011/02/04(金) 06:47:45
mod 2 の整数の世界で(つまり0か1)

xyz + xy + z

を因数分解するか、因数分解できないことを証明してっちょ
346132人目の素数さん:2011/02/04(金) 15:17:06
>>345
問題のリステート
xyz+xy+z∈F_2[x,y,z]は既約であるか。
そうでないならばこれの分解を与えよ。

347132人目の素数さん:2011/02/04(金) 16:53:13
>>345
>>346

R=F_2[X,Y,Z]とおく。
p:=XYZ+XY+Z∈R が既約であることを示したい。
pが可約だと仮定し、p=fg を満たすf,g∈R\F_2を取る。
ここで、pにZ=1を代入したものを考えると、
p(X,Y,1)=XY1+XY+1=1 となるが、p=fg だったから、
1=f(X,Y,1)g(X,Y,1) の成立がいえる。
ここから、f(X,Y,1)=g(X,Y,1)=1 がいえる。
よって、f-1=jZ, g-1=kZ を満たすj,k∈Rが取れる。
p=fg=(jZ+1)(kZ+1)=kjZ^2+(k+j)Z+1
Z^2の項に着目することで、jk=0 がいえる。
これは明らかに矛盾である。
(∵jk=0ならばjかkのいずれかは0であるが そのときは
f-1=jZ, g-1=kZ より、f,gのいずれかはF_2に属する)

348132人目の素数さん:2011/02/04(金) 17:20:25
>>Z^2に着目
の部分がわからないのだが、Z^2≡Zじゃないか?
349132人目の素数さん:2011/02/04(金) 17:32:55
xyz + xy + z = (xz + x + z) * (yz + y + z)

でも、2次*2次になっちゃってるし。これならいくらでも作れそう
なんか制限をかけないとmodの世界だと一意に因数分解できないのかも
350132人目の素数さん:2011/02/04(金) 17:43:24
>>349
うそかくなよ。
xyz^2=xyz は成立せんぞ。
zも不定元だろ。
351132人目の素数さん:2011/02/04(金) 17:45:26
>>349
君、もしかして、一般に体係数の多変数多項式環がUFDであることを知らないの?
それとも問題を勘違いしてる?
>>345>>346 の意味で書いているのだから これは分解できないよ。
352345=349:2011/02/04(金) 17:58:23
xyz^2 = xyzじゃないか?
1 * 1 = 1, 0 * 0 = 0だし。
でも、俺群とか環とかイデアルとかあんまり知らないから、変なことを言ってるのかもしれないけど
とりあえずF_2の記号がわからん
353132人目の素数さん:2011/02/04(金) 18:04:27
>>352
多項式の概念では不定元と係数は区別される。
354132人目の素数さん:2011/02/04(金) 18:21:55
>>353
分かってきた。
多項式の掛け算で指数に対してもmodが効くっていう定義を付け加えておけばよかったんだな
でも、初等整数論のスレなのにー
355132人目の素数さん:2011/02/04(金) 18:24:05
1変数のXに関する整数係数多項式の例

X^2+1 はそういうものの1つである。
ここでXは不定元であり、複素数ですらない。
Xに値を(形式的)"代入する"(X=z∈C)という操作によって
はじめてz^2+1は何らかの良く知られた数の集合に属する。

さて、F_2というのは2元体を意味していることがおおい。
これは0と1だけからなる集合であり、演算は次のように定義されている。
0+0=0, 0+1=1+0=1, 0*1=1*0=0*0=0
この演算(+,*)により、F_2は可換体をなす。

1変数のF_2係数多項式環F_2[X]というのは
"係数"がF_2に属している多項式の集合で、
自然な足し算と掛け算により 可換環をなしている。
(掛け算は注意が必要。Σa_iX^iΣb_iX^i=Σa_ib_jX^(i+j))
ちなみに係数が0の項は書かないのが慣習である。
(こうすることで有限項で打ち切ることができる)

計算例: (X+1)(X+1)=X^2+(1+1)X+1*1=X^2+0X+1=X^2+1
356354:2011/02/04(金) 18:25:33
間違えた、指数にmodを効かすだけじゃだめだったけど、
とりあえず不定元が整数の剰余環っぽくふるまうように定義すればよかったということで
357132人目の素数さん:2011/02/04(金) 18:38:32
1105 = a^2 + b^2
を満たす、自然数a, bをもとめてっちょ
358132人目の素数さん:2011/02/04(金) 18:45:21
>>357
君は高校生かい?
Z[i]の中では 1105=(2+i)(2-i)(3+2i)(3-2i)(4+i)(4-i)
あとはa+biの取り方を考えれば 解が全て得られる。
359132人目の素数さん:2011/02/04(金) 18:52:01
(2+i)(3+2i)(4+i)=9+32i
(2+i)(3+2i)(4-i)=23+24i
(2+i)(3-2i)(4+i)=33+4i
(2+i)(3-2i)(4-i)=31-12i

この計算結果から求める正整数a,b(a≧b)の組が
(a,b)=(33,4),(32,9),(31,12),(23,24)の4個で全てといえる。
360357:2011/02/04(金) 18:58:01
頭いいな、俺には数学向いてないや
俺はなんとなくa^2 + b^2の集合が掛け算に関して閉じてるなっと思ったから出したが、そういうことだったか
361132人目の素数さん:2011/02/04(金) 19:14:08

[問題]
X^9+X+1∈F_2[X] は既約かどうか判定せよ。
(なるべく手際の良い方法で)

362132人目の素数さん:2011/02/05(土) 02:48:37
>>361
F_2[X]の2次の既約多項式は X^2+X+1
3次の既約多項式は X^3+X+1, X^3+X^2+1
4次の既約多項式は X^4+X+1, X^4+X^3+1, X^4+X^3+X^2+X+1

f:=X^9+X+1が可約だと仮定すると、
1次以上4次以下の既約因子を持つ。

fは明らかにF_2の中に根をもたない。

任意に原始3乗根w,原始5乗根sをとるとき、
f(w)=w^9+w+1=(w^3)^3+w+1=1+w+1=w≠0
f(s)=s^9+s+1=s^4+s+1≠0 (∵4次既約多項式のリスト)

X^3+X+1の根の1つをαとすると、
f(α)=α^9+α+1=(α+1)^3+(α+1)=(α+1)((α+1)^2+1)≠0
(∵f(α)=0ならば第2因子=0となるが それはαの次数に矛盾する)
X^3+X^2+1の根の1つをαとすると、
f(α)=α^9+α+1=(α^2+1)^3+(α+1)=(α+1)^6+(α+1)
=(α+1)((α+1)^5+1)=(α+1)(α^5+α^4+α)=α(α+1)(α^4+α^3+1)≠0
(∵f(α)=0ならば第3因子=0となるが それはαの次数に矛盾する)

X^4+X+1の根の1つをαとすると、
f(α)=α^9+α+1=α(α^4)^2+α+1=α(α+1)^2+(α+1)≠0 (∵αの次数)
X^4+X^3+1の根の1つをαとすると、
f(α)=α^9+α+1=α(α^3+1)^2+(α+1)=α^7-1≠0
(∵X^(2^3-1)-1の根はGF(2^3)の元とみれるから次数は3以下)

以上より、fが既約であることが示された。
363132人目の素数さん:2011/02/05(土) 17:00:50
初等代数の問題まで出題していいってことにすれば、
出題可能なものが少しだけ増える悪寒がスr
364132人目の素数さん:2011/02/07(月) 20:24:03
nを自然数とする
n,n+1,n+2,n+3,n+4,n+5,
これらの自然数を2組に分けたとき
それぞれの組に含まれる自然数の積は必ず異なることを示せ
365132人目の素数さん:2011/02/07(月) 21:51:08
>>364
以下、Z/7Zの中で考える。

n≠1のとき、
6つの自然数の中に0が唯一つ存在する。
0を含む組の積は0になり、
そうでない組の積は0でない。

n=1のとき、
6つの自然数の集合は{1,2,3,4,5,6}に等しい。
任意に2組にわけ、2つの積をA,Bとおくと、
6つの自然数の総積は 6!= -1 であるから、
当然、A*B= -1 の成立がいえる。
ここで、A=Bと仮定するならば、A^2 = -1 となる。
しかし、(-1/7)= -1 であるから これは矛盾。
366132人目の素数さん:2011/02/07(月) 21:52:35
>>364
nが十分大きいとき。 2つの組は同数の要素を含まなければ、要素の多い組のほうが積は大きくなる。

5で割った余りが双方で一致するためには n〜n+5に5の倍数を2つ含まなくてはならない。
つまり、nは5の倍数でなくてはならない。 またnとn+5はおなじ組に入れられない。

3で割った余りが双方で一致するためには
(n,n+3),(n+1,n+4),(n+2,n+5) その3ペアからひとつずつの数をえらばなくてはならない。
nとn+5は同じ組に入れないのだから、nとn+2、n+3とn+5は同じ組になくてはならない。

残った組わけのうち n,n+2,n+4 と n+3,n+1,n+5 では どちらかが奇数ばかり他方は偶数ばかりになり
両者の積は一致しない。

最後の組み合わせ、 n,n+1,n+2 と n+3,n+4,n+5 では 前者が必ず小さいので不適

結局どのように組わけをしても両者の積は一致しない。

nが小さいときは… まだわからん。
367132人目の素数さん:2011/02/07(月) 22:02:47
>>366
mod 7で考えればいい。

>>365 のいっている通りだけど、
より広範囲にわかるように説明すると
nを7で割ったときの余りを考えればよい。
余りが1でないかぎり、6つのうち、1つだけ余りが0になる。
ということは その場合は考えなくて良い。
余りが1のときはウィルソンの定理と平方剰余の第一補充法則が使える。
この手法は明らかに自然な一般化ができる。
368132人目の素数さん:2011/02/10(木) 00:31:12
>>364

大昔の数オリの問題だな
まだ日本が数オリに参加してなかった時代
369福田和也:2011/02/22(火) 22:51:31.58
自然数上を動くn個の変数x1,x2・・・,xnが
x1 + x2 + ・・・ + xn = M である(和が一定、無論 M ≧ n )。
この条件の下で変数の積 x1 * x2 * ・・・ * xn を最大にする
値の組み合わせ (x1 , x2 , ・・・ , xn) は | xi - xj | ≦ 1 (∀i,j)
であることを示せ。

(意訳: x1 * x2 * ・・・ * xn を最大にするには手持ちのM個の資源を
 n個の変数に出来るだけ均等に配分する必要があることを示せ。 )
370>>1:2011/02/23(水) 20:57:19.22
皆さんすみません^^;
数学に取り組んでいて、算数パズルを作っている時間がありませんでした^^;
昔の問題はかなりありますがそろそろここに投稿しますね。

>>369
(私が問題に返答するのは稀でしたが...単に燃料ということで^^;)
a-b>1 ⇔ (a-1)(b+1)>ab からダイレクトに従うと思います^^
371整数の基本性質:2011/03/07(月) 20:21:08.57
(定理1-1)

a.b∈Z、b≧0とするとき
a=bq+r.0≦r<b…(*)
を満たすq.rが唯一組定まる
372132人目の素数さん:2011/03/07(月) 20:42:02.27
>>371ノ証明

どのようなbに対しても
qb≦a< (q+1)b
を満たすq∈Z が必ず唯一つ存在する
すなわち、
0≦a−bq <b
よって、r=a−bqとおくと
(*)が成り立つ
373132人目の素数さん:2011/03/21(月) 14:52:52.26
y^2=x^n-x+2 を満たす自然数n,x,yの組をすべて求めよ
大学への数学の宿題の問題だけど
過疎ってるし投下
374:2011/03/28(月) 14:07:00.59
>>373
y^2≡-1 (mod 3)になりえないので、
mod 3 でみることで x≡-1 (mod 3)がいえる。
これから nが偶数であることもいえる。n=2mとおく。
x>2であるとき (x^m-1)^2<x^n-x+2<(x^m)^2 が成立する。
なのでこのときは問題の等式は成立しえない。
x=2であるときは nが偶数であることが
問題の等式が成立することの必要十分条件である。
375132人目の素数さん:2011/03/28(月) 16:48:15.25
>>374
死ね
376132人目の素数さん:2011/04/05(火) 18:42:24.88
>>374
ネタバレはあかん
377132人目の素数さん:2011/04/07(木) 05:02:26.82
x^3 +y^3 +z^3 = (x+y+z)^3
をみたす(x,y,z)を求めよ
378132人目の素数さん:2011/04/07(木) 07:21:58.12
>>377
(0,0,0)
379132人目の素数さん:2011/04/07(木) 13:20:13.18
(x,y,z)=(t,s,-s),(-s,t,s),(s,-s,t)
380:2011/04/07(木) 16:30:41.87
>>377
0 = 左辺-右辺 = 3(x+y)(y+z)(z+x)

(問題)
3n^2-n-1 が2つの互いに素な合成数の和で表現できるような整数nを全て求めよ。
381132人目の素数さん:2011/04/07(木) 21:18:59.54
>>380
3n^2 -n-1=n^2 +(2n+1)(n-1)だから、検討すべき場合は有限個に絞れるね。
382132人目の素数さん:2011/04/07(木) 21:54:34.16
381のは綺麗な方法だけど こんなアホみたいな方法もあるぞ。

2^2, 2^3, 2^4, 2^5 のどれかを
3n^2-n-1 から引き算すれば 5の倍数になる
なぜなら 3n^2-n-1は常に5で割り切れなくて
しかも 2^2,2^3,2^4,2^5を5で割った余りは全て異なるから。
一方、3n^2-n-1は常に奇数であるから あわせて解答がつくれる。
383132人目の素数さん:2011/04/07(木) 22:03:02.04
-1,0,1,2.
210.
384132人目の素数さん:2011/04/08(金) 14:48:55.81
156156や598598や713713713713みたいな3桁で繰り返し
かつ6の倍数の桁数である数は必ず143で割り切れる
123123のような3桁を繰り返した数字は
まずもって7で割り切れる
3桁を繰り返せば1001=7×11×13で割り切れるからね

↑有名なコピペだから知らなかったヤツは暗記しとけ^^
暇な奴はこの性質を使った難問でも作ってろ^^
385132人目の素数さん:2011/04/08(金) 16:34:04.18
問題を作るのが一番難しいんだよ!
386132人目の素数さん:2011/04/08(金) 21:32:32.49
任意に整数m>2と整数aを固定する。
a^n≡-1 (mod m)を満たす正整数nが存在すると仮定し、
そのようなもので最小のものをsとおき、さらに、
aのmod mでの位数をuとおくとき、u=2sが成立する。
これを証明せよ。

sは位数uの半分くらいな感じがしますよね^^
sの言葉を使うと解答を少しだけ簡略化できる問題が存在します^^
387132人目の素数さん:2011/04/08(金) 23:42:48.56
>>386
問題整理汁
388132人目の素数さん:2011/04/09(土) 02:17:41.24
>>386
この問題のポイントとしては、sがuで割り切れることと、2sがuで割り切れることに尽きる。
そして、m>2だからu=sはあり得ないから、結局u=2sがいえるんだね。
389388:2011/04/09(土) 02:38:46.64
これだけじゃ何なんで、「sがuで割り切れることと、2sがuで割り切れること」を軽く説明
2sがuで割り切れる事の説明
2sをuで割った時の余りをrとする。1≡a^(2s)≡a^r (mod m)でuの最小性より、r=0がいえる。

uがsで割り切れる事の説明
uをsで割った時の賞をq、余りをtとおくと、a^t≡(-1)^(-q)*a^u≡±1 (mod m)
sの最小性よりa^t≡-1 (mod m)にはなりえず、 a^t≡1 (mod m)の場合もa^(s-t)≡a^(s-t)*a^t≡-1 (mod m)
より、sの最小性を考慮するとt=0とならざるを得ない。
390132人目の素数さん:2011/04/09(土) 04:21:25.09
まあまあな問題だな
利用価値のある小道具っていう感じだ
391132人目の素数さん:2011/04/12(火) 00:57:31.48
x,yが整数で、y^2 + 3x^2y^2 = 30x^2 + 517 のとき、3x^2y^2 の値をもとめよ
392≫≫301:2011/04/12(火) 00:59:33.30
3x^2y^2 は、3かけるx^2かけるy^2 ね
393132人目の素数さん:2011/04/12(火) 01:06:02.43
括弧を使え
394132人目の素数さん:2011/04/12(火) 01:20:49.65
395132人目の素数さん:2011/04/12(火) 01:31:47.17
>>391
なぜ2乗にしているの? べつに y+3xy=30x+517 でもいいでしょ。
無駄に指数を上げることで 問題の価値が下がっていくよ。

y+3xy=30x+517 ・・・(*) を満たす整数x,yを全て求めることにする。
LHS=y(3x+1), RHS=10(3x+1)+507
そして 常に3x+1≠0であるから、
(*)が成立する ⇔ 3x+1|507 かつ y=10+507/(3x+1)
507=3*13^2であるから、3x+1|507 ⇔ 3x+1|13^2
⇔ 3x+1∈{1,13,169} ⇔ x∈{0,4,56}

以上より(*)を満たす全ての整数x,yの組は
(x,y)=(0,517),(4,49),(56,13) の3つである。

とくに >>391 の問題の答えは (x,y)=(4,49)
396132人目の素数さん:2011/04/12(火) 01:34:15.29
ごめん。>>391 の問題の答え自体は 3*4*49=588 だね。
3ポストのうち、2つも無駄な投稿をしてしまった。すまんね。
397132人目の素数さん:2011/04/19(火) 00:00:30.34
>>395
3x+1|13^2 から 3x+1∈{1,13,169} は "フツウ"はいえないね
mod 3での対称性をつかっているのかね?
ふつうは負の約数まで考慮する必要があるからね。
ただ、mod 3で0でない数にマイナスをつけると、
mod 3での符号も反転するから、その議論は特別に通用する。
3x+1はmod 3で1で、1,13,169もすべてmod 3で1だからね。
-1,-13,-169はすべてmod 3で-1になり、3x+1に一致しえない。
398132人目の素数さん:2011/04/19(火) 00:20:26.11
>>391
>>395
の問題を数ランクうpしたものをつくってみた

3xy+6x+y-1が平方数になるような
整数x,y(y<1)の組を全て求めよ。

大きなヒントは既に出ているから簡単すぎかな
399132人目の素数さん:2011/04/19(火) 03:09:33.86
予想
398はあほな勘違いをしている
400132人目の素数さん:2011/04/19(火) 11:21:35.61
3xy+6x+y-1=n^2
(3x+1)(y+2)=n^2+3
y+2=(n^2+3)/(3x+1)
y=(n^2+3)/(3x+1)-2
401:2011/04/19(火) 15:44:29.82
4|yになる場合を除外すれば締まりのある問題になりそうですね。
402132人目の素数さん:2011/04/21(木) 00:36:21.10
-1
403132人目の素数さん:2011/04/21(木) 19:09:18.18
p^2=(1+a/b)(1+b/a)(a+b-1)を満たす自然数abの組み合わせを求めよ
pは素数
404132人目の素数さん:2011/04/21(木) 22:49:08.33
>>403
d=(a,b),a=ds,b=dt(s≧t)とすると、与方程式は
stp^2=(s+t)^2*(d(s+t)-1) ・・・(*) と変形される。
(s+t,st)=1 であるから、(*)より、(s+t)^2|p^2 がいえる。
pは素数であるから、これから、p=s+t がいえる。
(*)とあわせて、(s+1)(s-1)=p(s-d) ・・・(!) が得られる。
pは素数であるから、(!)より、p|s+1 または p|s-1 である。
s-d<s+1 であるから、(!)より、p|s-1 になりえず、p|s+1 となる。
s-1≧s-d であるから、p|s+1 と (!) より、d=1 かつ t=1 が得られる。
よって、求める自然数a,bの組は
a=p-1, b=1(p:任意の素数)で全てである。
405132人目の素数さん:2011/04/22(金) 23:24:38.65
http://www.casphy.com/bbs/test/read.cgi/highmath/1174966415/

85 だるまにおん [2009/01/28(水) 22:38:52]

x,y,zが自然数のとき
  (xy+1)(yz+1)(zx+1)が平方数
    ⇔ xy+1,yz+1,zx+1全てが平方数
406132人目の素数さん:2011/04/23(土) 05:54:18.90
>>405
やや有名な問題。
以下、補題からはじめて解答を書くことにする。

正整数全体の有限な部分集合で、どの2元の積に1を足しても
平方数となるようなものをD-setと呼ぶことにする。
たとえば、{1,3,8,120} はD-setになっている。

[補題]
{p,q,r}がD-setであるとき、
s=p+q+r+2pqr±2√{(pq+1)(qr+1)(rp+1)} ・・・(*)
とおけば、(符号はsが正となるように選ぶことにする)
{p,q,r,s}もD-setとなる。

(証明)
次の3つの等式の成立が確認できることから明らか
(p+q-r-s)^2=4(pq+1)(rs+1)
(p+r-q-s)^2=4(rp+1)(qs+1)
(p+s-q-r)^2=4(qr+1)(ps+1)

407132人目の素数さん:2011/04/23(土) 06:03:50.31

[解答]
(pq+1)(qr+1)(rp+1)が平方数であり、かつ、
{p,q,r}がD-setとならないような正整数p,q,r(p≦q≦r)
が存在していたと仮定する。(ここから矛盾を導きたい)
そのようなp,q,rの組のうち、とくに、
p+q+rが最小となるようなp,q,rを取ってきたとする。
ここで sを (*)においてマイナスの符号の方で定める。
このとき、0<s<rであること、および、
(ps+1)(qs+1)(pq+1)が平方数であることが示せる。
さらに、{p,q,r}がD-setでないことも示せる。
もし、それが示されれば、p+q+rの最小性に矛盾となる。
ということで、それらの主張が正しいことを示したい。

sの定め方から次の等式の成立を確認することができる。
16(pr+1)(qs+1)(qr+1)(ps+1)(pq+1)^2={(pq+1)(p+r-q-s)(p+s-q-r)}^2
これから、(ps+1)(qs+1)(pq+1)が平方数であること、
および、{p,q,s}がD-setでないことは ほとんど明らかである。
408132人目の素数さん:2011/04/23(土) 06:06:05.34

最後に 0<s<rの成立を示すことにする。
まず、0<s であることを示す。
(p+q-r-s)^2=4(pq+1)(rs+1) であるから
(これは補題の証明で用いた3つの等式のうちの1つである)
とくに rs+1≧0 であることがいえる。
r=1であるならば、p≦q≦r より、p=q=r=1 となり、
このとき、(pq+1)(qr+1)(rp+1)は平方数とならない(矛盾)
また、s=0であるならば、補題で用いた3つの等式から、
{p,q,r}がD-setとなってしまい、これも矛盾である。
よって、s>0 であることがいえた。
つぎに (*)においてプラスの符号をとったものをtとすると、
st=p^2+q^2+r^2-2pq-2qr-2rp-4<r^2
さらに、s<t であるから、あわせて、s<r がいえた。
                                ■
409132人目の素数さん:2011/04/28(木) 22:13:32.70
問題投下
正の整数kを任意に取る。
n!がn^2 +kで割り切れるような正整数nは無数に存在することを証明せよ。
410132人目の素数さん:2011/04/28(木) 22:52:56.04
n = k^2 * a^3 (aは任意の整数)
みたいな
411132人目の素数さん:2011/04/28(木) 22:54:31.12
ミスったorz
412:2011/04/29(金) 01:47:09.03
>>409
rをkとパリティの異なる3以上の任意の整数とする。
このとき、n=r^2+r+k とおけば、n^2+k|n! がいえる。

(pf)
上記のようにr,nを取るならば、
n^2+k=(r^2+k)(r^2+2r+k+1) となるが、
r≧3 であることから、
2<(r^2+2r+k+1)/2<r^2+k<r^2+r+k=n
がいえて、さらに rのパリティから、
(r^2+2r+k+1)/2 は整数であることがいえる。
よって、2*{(r^2+2r+k+1)/2}*(r^2+k)|n! がいえた。
すなわち、n^2+k|n! がいえた。
413132人目の素数さん:2011/04/29(金) 17:28:23.96
>>412
回答ありがとう、正解。
俺はk+1の倍数rに対して、n=r^2 +r+kとおく方法を考えていた。
414132人目の素数さん:2011/04/30(土) 02:27:17.46
問題
12で割ると7余る素数が無数に存在することを証明せよ。
(算術級数の素数定理を使わないで示すこと)
415:2011/04/30(土) 02:55:50.07
>>414
そのような素数が無限に存在しないと仮定し、
そのような素数の全ての積(有限)をPとおく。

ここで、Q:=(2P)^2+3 という自然数を考える。
Q≡-1(mod 4)であるから、q≡-1(mod 4)なるQの素因数qが存在する。
Pは3で割り切れないから、Qは3で割り切れないので、q≠3
(-3/q)=1 であるから、これから q≡1(mod 3)がいえる。
あわせて、q≡7(mod 12)であることがいえる。
よって、Pの定め方から、q|P であるといえるが、
Q=(2P)^2+3 であったから、あわせて、q|3 となる。
しかし、これは明らかに q≠3 に矛盾している。
416132人目の素数さん:2011/04/30(土) 21:49:31.13
>>415
正解。

それではまた、問題投下
a とbをa>bをみたす正の整数とする。
このとき、n|a^(n-1)-b^(n-1)をみたす合成数nが無数に存在することを証明せよ。
417:2011/04/30(土) 23:00:01.49
>>416
aとbが互いに素であるときを考えれば十分である。
以下、aとbは互いに素であると仮定しておく。

(補題)
6より大きい整数mに対して、
ord_q(a/b)=mなる素数qが存在する。

有名なので(証明はどこにでもあるので)証明は省く。

[解答]
任意にord_r(a/b)<r-1 なる素数r>7を取る。
補題より、とくにord_q(a/b)=r-1なる素数qが存在する。
これから r-1|q-1 がいえるので r-1|qr-1 もいえる。
また、rの取り方から、q≠rがいえるので、あわせて、
qr|a^(qr-1)-b^(qr-1) の成立がいえる。
418:2011/04/30(土) 23:08:15.86
問題投下します。
既にスレの中にあるようですが、
再浮上ということで・・・

[問題)
各整数n≧6に対して τ(n!)|n! であることを示せ。
(各正整数xに対して、τ(x)はxの正の約数の個数を表すとする)
419132人目の素数さん:2011/05/01(日) 00:21:26.79
>>417
補題はZsigmondyの定理からすぐいえるね、俺の回答の方針はこう。
pをab(a^2-b^2)より大きな素数とする。
n={a^(2p) -b^(2p)}/(a^2 -b^2)とおくと、nが合成数でかつn|a^(n-1) -b^(n-1)であることがいえる。
実は意外と簡単に解けたのですw
420:2011/05/01(日) 02:33:11.03
>>419
良い回答ですね!
補題はまさにZsgimondyの定理(の枝葉を切った)のつもりでした。
421あんでぃは単細胞 ◆AdkZFxa49I :2011/06/16(木) 21:26:20.67
あんでぃ
422132人目の素数さん:2011/06/27(月) 16:37:37.79
質問です。
1以上の自然数a,b(a≠b)で、
a^3+b^3
は、必ず合成数になりますか?それって当たり前の事ですか?
教えてください。よろしくお願いいたします。
423132人目の素数さん:2011/06/27(月) 19:41:15.13
>>422
a^3+b^3=(a+b)(a^2-ab+b^2) で a+b>1, a^2-ab+b^2>1 なので,合成数
424132人目の素数さん:2011/06/27(月) 20:06:32.69
>>423
ありがとうございます。素晴らしいです。
私も自力で解けるように、学生時代の数学の偏差値は30台〜40台でしたが、
地道に数論の勉強を続けます。
425132人目の素数さん:2011/06/27(月) 20:40:51.76
これが数論といわれるとすごい違和感
426132人目の素数さん:2011/06/27(月) 20:48:01.99
ただの文字の問題ですね
427132人目の素数さん:2011/06/27(月) 20:49:12.01
2つの整数a、bがあって、
両者が等しい時1、
両者が異なる時0を返す関数は整数論で定義できますか?
428じゅー:2011/06/27(月) 21:15:23.42
[[a/b]-a/b]+1とか??
429132人目の素数さん:2011/06/27(月) 21:16:34.98
[1/(1+|a-b|)]とか?
430132人目の素数さん:2011/06/27(月) 21:20:57.23
Thxなるほど
ガウス記号か...
431132人目の素数さん:2011/06/27(月) 21:21:03.83
>>428
b=0の時に使えない。
a=k*b(kは整数)が成り立つときに、1を返す。
432132人目の素数さん:2011/06/27(月) 21:24:49.80
ガウス記号が整数論の関数に入るんなら、
クロネッカーデルタ自体も整数論の関数に入るってことでいいんじゃねーの?
433じゅー:2011/06/27(月) 22:56:14.72
>>431
スマソ。わすれてました。
しかもa≠bでも1になれるし…orz
434132人目の素数さん:2011/07/04(月) 18:32:24.75
n→∞で|1-(1/(|a-b|+1)^n)|
435132人目の素数さん:2011/07/05(火) 13:38:37.21


【政治】菅首相の資金管理団体、北の拉致容疑者親族所属政治団体から派生した政治団体「政権交代をめざす市民の会」に6250万円献金★14

http://raicho.2ch.net/test/read.cgi/newsplus/1309573084/

現実のほうがものすごいことが起きている件について、キミはどう思う?


特捜1「献金されています!五千万です!」
特捜2「献金元はどこだ・・・!?」

特捜1「・・・これは・・・ウソだろ?総理です!総理が五千万献金しています!」

110:名無しさん@12周年 07/02(土) 08:36 GAZzjy8T0 [sage]
オバマがビンラディンの親族が属する政治団体に大口寄付してたようなものw

909:名無しさん@12周年 07/02(土) 09:55 oEGy+UI/0 [sage]
テロのスポンサーが総理大臣って…。

>25 名前:名無しさん@12周年[] 投稿日:2011/07/02(土) 08:22:15.91 ID:8a/xyVGw0 [1/9]
>一瞬拉致被害者団体に献金ならまぁいいんじゃないかと思ったんだが
>よく読んだら容疑者団体ってwwwwww
>有り得ない文字に目がおかしくなったのか俺wwwww

俺もwwwwww


総合演出責任者 ばぐ太  ←関係者は苦情はこちらまで

436132人目の素数さん:2011/07/22(金) 10:18:03.57
虚数は整数論には入らないのかも知れませんが、質問があります。
i^2 = -1 になるiが虚数ということですが、
X^2 = i とか、X^3 = iになるXっていうのは、どのような値ですか?
437132人目の素数さん:2011/07/22(金) 11:00:19.83
>>436
その虚数iと実数を足し合わせたような数ならば
そのようになる数が作れる。 そういう数を複素数という。
438132人目の素数さん:2011/07/22(金) 11:04:42.67
>>437
ありがとうございます。
そうか。複素数なんですね。
439132人目の素数さん:2011/07/23(土) 08:30:50.07
>>436
二次体で検索!
440132人目の素数さん:2011/07/23(土) 18:26:53.21
>>439
ありがとうございます。
2次体ですね。調べて見ます。
ところで、整数論を勉強するのに虚数は必要ですか?
と言うか、整数論の範囲に虚数は含まれますか?

441132人目の素数さん:2011/07/23(土) 18:37:18.40
虚二次体を勉強せよ。
442132人目の素数さん:2011/07/23(土) 18:46:49.68
>>440
虚数どころかp進体とかめちゃめちゃ範囲広いぞ、整数論は。
443132人目の素数さん:2011/07/23(土) 19:58:28.85
解析的整数論なら、積分さえも使う。
代数的整数論なら、虚数はおろか"虚数ですらない新しい数の体系"も使う。
444132人目の素数さん:2011/07/24(日) 12:50:09.95
数論でZ[i]やZ[ω](複素数)は基本中の基本中
平方剰余相互規則や二次体や類数公式を見てみよう!
445132人目の素数さん:2011/07/24(日) 13:11:45.59
素数の逆数の和は発散する。
446132人目の素数さん:2011/07/24(日) 13:16:04.22
双子素数の逆数の和は発散する。
447132人目の素数さん:2011/07/24(日) 13:16:32.48
メルセンヌ数の逆数の和は発散するか。
448132人目の素数さん:2011/07/24(日) 15:13:38.28
今年の数オリ問題くらいサクっと解けよお前らw
449132人目の素数さん:2011/07/24(日) 15:33:06.64
みなさん、ありがとうございます。
整数論だから整数だけの世界であって、実数とか複素数とか、
微分・積分とか、そんな難しいことは知らなくても良いと考えていました。
甘かったです。でも、あきらめずに少しずつ勉強します。
中学生の数学から挫折したので、昔使った教科書の、
啓林館の新訂数学復刻版中学教科書を購入しました。
1年生の1章は「数と集合」です。ここから勉強しなおします。
450132人目の素数さん:2011/07/25(月) 21:37:21.79
あまりに酷すぎるのだけど釣り?
451132人目の素数さん:2011/07/26(火) 22:43:09.99
>>450
釣りではなく事実です。
私はセンター試験の前の共通1次世代なのですが、模試でも本番でも、
数学は200点満点中いつも30点でした。
なぜ30点なのかというと、y=3x+2とy=3x^2-5の交点を求めよというような
問題は得意(?)だったので解けたのと、
赤玉5個と白玉3個から赤玉3つを取り出す確率を求めよというような問題は、
全てのパターンを
赤1赤2赤3
赤1赤2赤4
赤1赤2赤5
赤1赤2白1
...
白1白2白3
のように書き出して求めるので正解できたからです。
それ以外の問題はさっぱりわかりませんでした。
三角関数、微分・積分、数列、幾何などは、問題の意味すら
わかりませんでした。
452132人目の素数さん:2011/07/26(火) 22:47:29.55
別に今生きてるんだからそれで良いじゃん。

貴方の書き込み見ると単なる勉強不足だと思うよ。
453132人目の素数さん:2011/07/26(火) 23:12:06.64
>>445

http://www.junko-k.com/cthema/17sosuu.htm
http://ja.wikipedia.org/wiki/素数

>>446

B ≒ 1.902160583104
http://mathworld.wolfram.com/BrunsConstant.html

>>447
 収束する
 Σ[n=1,∞) 1/(2^n - 1) < Σ[n=1,∞) 1/2^(n-1) = 2,
454132人目の素数さん:2011/07/27(水) 00:04:45.76
>>449
あきらめたほうが賢いと思うよ
その歳でそのレベルから数学を初めても死ぬまでに大学初年級の
数学にたどり着けるかも疑問だ
世の中には数学と同じくらい楽しいことがたくさんある。
しかもより手軽に楽しめるものがだ
数学はこの世で縁がなかったと思うことだな
455132人目の素数さん:2011/07/27(水) 14:38:54.73
好きなコトよりも手軽に他にしめるものを楽しんだほうが賢いという立場からの意見ですね。
456132人目の素数さん:2011/07/28(木) 07:09:31.39
>>445

Σ_n 1/n = Π_p (1 + 1/p + 1/p^2 + …… ) = Π_p 1/(1-1/p),

log(Σ_n 1/n) = -Σ_p log(1-1/p)
 = Σ_p Σ[k=1,∞) (1/k)(1/p)^k
 = Σ[k=1,∞) (1/k)Σ_p (1/p)^k
 < Σ_p (1/p) + (1/2)Σ[k=2,∞) Σ_p (1/p)^k
 = Σ_p (1/p) + (1/2)Σ_p 1/{p(p-1)}
 < Σ_p (1/p) + (1/2)Σ[n=2,∞) 1/{n(n-1)}
 = Σ_p (1/p) + (1/2)Σ[n=2,∞) {1/(n-1) -1/n}
 = Σ_p (1/p) + (1/2),
より発散。
 (「大学への数学」によるらしい)
457 忍法帖【Lv=18,xxxPT】 :2011/08/21(日) 12:13:36.10
保守
458132人目の素数さん:2011/08/22(月) 16:55:17.59
数学以上に面白く素晴らしいものなんて存在しませんよ
459132人目の素数さん:2011/08/22(月) 19:27:56.75
面白いものを数学と名付けたんですよ
460:2011/08/30(火) 20:31:27.90
>>449
数学に王道あり。
数学雑誌など探して自分にあった近道を見つけよう。
461132人目の素数さん:2011/08/31(水) 12:56:12.80
>>449
ありがとうございます。
最近は「天才ガロアの発想力」小島寛之著が読み物として楽しくて、
繰り返し読んでいます。
さらに「共立講座21世紀の数学(9)代数と数論の基礎」中島匠一著が、
分かりやすそうなので精読しようと読み始めました。
整数論は、自然数や素数などの小学生でも分かる基本的な概念から、
どんどん発展していくのが楽しく感じます。それに、色々な解釈が存在する
他の学問とは異なってスパっと明快なのが気持ち良いです。
462132人目の素数さん:2011/08/31(水) 13:07:50.95
1988IMOの6がすごい。
シンプルなんだがテレンスタオも
解けなかった初等整数論の難問
463132人目の素数さん:2011/09/06(火) 18:30:09.00
大学への数学宿題より
x^p+y^p=p^zを満たすxyzpの組み合わせを求めろ
xyzは自然数pは素数
464検便のナウシカ ◆UVkh7uHFoI :2011/09/08(木) 01:23:48.26
n枚のコインの中に一つだけ偽物がある。
このとき天秤を用いて偽物を見分けるための最小ステップ数を計算するアルゴリズムってありますかね?
465検便のナウシカ ◆UVkh7uHFoI :2011/09/08(木) 01:35:44.28
連投失礼。
できるだけ計算量が少なくてエレガントなやつ
466132人目の素数さん:2011/09/08(木) 01:53:43.83
あるけど、ここに書くのはちとしんどいな
467検便のナウシカ ◆UVkh7uHFoI :2011/09/08(木) 01:58:05.37
すごいっすね、このスレの人たち
468132人目の素数さん:2011/09/08(木) 04:51:47.41
469132人目の素数さん:2011/09/08(木) 17:24:32.23
>>468
ありがとうございます。
参考にさせていただきます。
470132人目の素数さん:2011/09/14(水) 14:47:58.42
『天才ガロアの発想力』という書籍を読んで、
有理数体Qに√2を追加した拡大体をQ(√2)と書くと言うことがわかりました。
とても面白いです。
では、有理数体Qに何かを追加して、有理数体の拡大体として実数体Rを
表すことはできますか。
つまり、
R = Q(*)
の*に何を入れれば良いかです。
情報弱者ならぬ数学弱者なので、全く見当違いのことを尋ねているかも知れません。
どなたかよろしくお願いいたします。

471132人目の素数さん:2011/09/14(水) 15:15:31.67
R = Q(R) 

472132人目の素数さん:2011/09/14(水) 15:27:24.30
>>471
早速のご教示ありがとうございます。
でも、なんか、RはQを含んでいるので当たり前なので、
Rを使わないで表すことは不可能なのでしょうか。
R = Q(R)
だと、Qじゃなくても何でもよさそうなので、
これだと書き方が間違っているかも知れませんが、
R=Ø(R)
みたいに感じます。
私の考え方が見当違いなら申し訳ありません。
473132人目の素数さん:2011/09/14(水) 15:28:32.79
X= 無理数全部の集合

R = Q(X)
474132人目の素数さん:2011/09/19(月) 05:09:19.74
>>472
QからRへの拡大は超越拡大といって、有限個の元を追加する拡大では到達できない。
可算個の追加でも無理。
475132人目の素数さん:2011/09/19(月) 08:03:13.14
>>474
ありがとうございます。
わかりやすい回答ありがとうございます。
超越拡大で調べて見ます。
476132人目の素数さん:2011/09/19(月) 10:09:24.16
>>474みてふとおもたんやけど、
集合論とかではないいわゆる普通の数学でハメル基が活躍する場面て
あるんやろか。
477132人目の素数さん:2011/10/04(火) 14:14:22.64
>>476
AV
478132人目の素数さん:2011/10/06(木) 15:42:22.68
整数a,b,c…についてgcd(a,b,c…)=1とa,b,c…が対ごとに互いに素というのは全く異なるというが是非反例が欲しい
479132人目の素数さん:2011/10/06(木) 15:44:17.08
反例ではないな。例だ。訂正
480132人目の素数さん:2011/10/06(木) 15:48:17.54
因みに記述のソースは『ガウス整数論への道』(加藤明史)
481132人目の素数さん:2011/10/06(木) 18:27:06.13
n≧3とする。p_1, p_2, …, p_n は全て異なる素数として、

x_k = (p_1p_2…p_n)/p_k (k=1, 2, …, n)

と置く。gcd(x_1, x_2, …, x_n)=1 であることが言える。
また、n≧3に注意して、任意のi,j に対してgcd(x_i, x_j)≠1 であることが言える。
482132人目の素数さん:2011/10/06(木) 20:05:34.07
難しく考えなくても こんな程度でいい
a=2・3 、 b=3・5 、 c=2・5
483132人目の素数さん:2011/10/07(金) 22:26:11.17
>>481>>482
感謝
では「対毎に素」⇒「全部のgcdが1」は言えるだろうか
こちらも聞きたかった
484132人目の素数さん:2011/10/07(金) 22:31:03.94
すこしは自分の頭で考えろボケナス
485132人目の素数さん:2011/10/07(金) 22:47:46.85
>>484
いや、実は>>481>>482の例は件の本に載っていたんだ
だから本当は聞きたかったのは>>483のことなんだ
486132人目の素数さん:2011/10/07(金) 22:48:58.29
誤解を招く様な書き方についてはすまなかった
487132人目の素数さん:2011/10/07(金) 22:58:09.18
>>483
対偶を証明すればいいがな。

さすがに この程度は自力で証明できるだろう?(^ω^)
488132人目の素数さん:2011/10/08(土) 01:01:06.03
>>485
むしろ483の方は明らかだから,いちいち書いていないのでは?
互いに素ということは共通の素因数がないということだよね?
489132人目の素数さん:2011/10/08(土) 01:17:35.44
>>488
元々は>>478の「全く異なる」を>>483が成り立たないと勘違いしたんだ
だけどそれは同値性が成り立たないと言うだけであって>>483は成り立つんだよな
ということに後から気付いた
490132人目の素数さん:2011/10/08(土) 19:17:50.20
>>485>>483
「対毎に素」⇒「全部のgcdが1」かい?マジ?スゲー簡単だろ。

a(1),a(2)…,a(n)の最大公約数dをとるとき、dは当然a(1),a(2)を割り切る。
a(1),a(2)は仮定より互いに素だから、当然d=1となる。

これでいいんじゃないの?
491132人目の素数さん:2011/10/08(土) 23:11:45.55
>>464
>偽物を見分けるための最小ステップ数を計算するアルゴリズムってありますかね?
当然のことながら、そのアルゴリズムを自動生成するアルゴリズムもありますね。
>>468
>http://f59.aaa.livedoor.jp/~nadamath/bushi/2010_rc.pdf
筆者は今年のIMO金賞なんだね。
492132人目の素数さん:2011/10/09(日) 00:13:07.81
孫子の定理(中国剰余定理)

m1,m2,m3,…,mn∈Z∧gcd(ms,mt)=1(s,t= 1,2,3,…,n∧s≠t)とする時、
a1,a2,a3,…,an∈Z⇒∃x∈Z;x ≡ ak (mod mk) (k = 1,2,3,…,n)
で、解は(mod m1m2m3…mn)で一意的である.

について、結論としては
mΠ=m1m2m3…mnとする時
x=Σ((mΠ/mi)*(mΠ/mi)^(-1)*ai)(1≦i≦nで、逆数はmod miにおけるもの)
が解ということだよな

解がわかるんだから存在定理にしなくてもいい気がするんだが
493132人目の素数さん:2011/10/09(日) 00:54:05.68
一般に二項係数C[n.k]が整数となることは素因数を用いてどのように示せば良いのでしょうか
494132人目の素数さん:2011/10/09(日) 09:38:43.67
パスカルの公式
   C〔n,k〕=C〔n−1,k−1〕+C〔n−1,k〕
により、帰納的に明らかでしょう。
495132人目の素数さん:2011/10/09(日) 13:38:54.89
整数論的に示したいのです
496132人目の素数さん:2011/10/09(日) 14:30:14.30
パスカルの公式は、思いっきり整数論的ですが
497132人目の素数さん:2011/10/09(日) 17:55:59.65
素因数を用いてということです
498132人目の素数さん:2011/10/11(火) 18:06:04.68
C[n.r]=n!/r!(n-r)!=n(n-1)(n-2)…(n-r+1)/r!

ここで(n-r+1)からnまでの整数はr個あるので、これらは法rについて完全剰余系を成す。
故にn,(n-1),(n-2),…,(n-r+1)の内どれか一つはrと合同、即ちrの倍数である。
r!はrの倍数であるから分母分子ともrの倍数である。
同様にして分母分子とも(r-1),(r-2),…,2,1の倍数であることがわかる。
従って分母分子ともr!の倍数であることがわかる。

ここでn-r+1≧1で分母分子共に連続したr個の整数の積であるから(分母)≧(分子)で、分母分子ともr!の倍数であるから与式は自然数になる。 □

これなら厳密な証明かね
499132人目の素数さん:2011/10/11(火) 18:15:07.36
4は2と4の倍数だけど2×4=8の倍数じゃない。
500132人目の素数さん:2011/10/11(火) 20:39:22.03
パスカルの公式で十分簡単かつ初等的なのに、なぜ複雑かつ非初等的な七面倒臭い証明をしたのか、さっぱり分からん。
501132人目の素数さん:2011/10/11(火) 21:23:06.60
>>500
帰納法があまり好きではないんでね。
まあ好みの問題だ。
502132人目の素数さん:2011/10/11(火) 21:38:33.19
>>501
ていうか>>499じゃないが、>>498は完全に間違っている罠。
「整数Nがr,(r-1),(r-2),…,2,1の倍数ならば、Nはr!で割り切れる」なんてとても言えないので。

「整数Nがr,(r-1),(r-2),…,2,1の倍数」からいえるのは、「Nはr,(r-1),(r-2),…,2,1の最小公倍数で割り切れる」まで。
しかも残念ながら、r,(r-1),(r-2),…,2,1の最小公倍数はr!よりもかなり小さいことが言える。

悪いが、整数の勉強をやり直した方がいいと思うぜw
503132人目の素数さん:2011/10/11(火) 21:40:35.78
好み云々言う前に正しい証明が出来るようにならんといかんな
504132人目の素数さん:2011/10/11(火) 21:51:36.20
>>502
全くその通りだ。
恥ずかしい限り
505132人目の素数さん:2011/10/12(水) 04:55:58.02
所で次のライプニッツの定理を示すには帰納法しかないのだろうか。合同の法は素数pで、iは1からnまでをわたるとする。
(Σai)^p≡(Σ(ai)^p)
506132人目の素数さん:2011/10/12(水) 11:00:40.65
> 非初等的な
どのあたりが?
507132人目の素数さん:2011/10/13(木) 17:44:40.58
508132人目の素数さん:2011/10/15(土) 21:04:39.38
この問題解ける方いますか?
歯が立たなくて困ってます。

問題
a_{n+1}=a_{n}^a_{n}、初項a_1=A
(n≧1)
で表される漸化式がある。
このとき、a_nをCで割った余りを求めよ。漸化式の初項、Cは共に自然数とする。
509132人目の素数さん:2011/10/15(土) 23:32:17.00
スレ違い
初等整数論以外の質問は、他の質問スレへ
510132人目の素数さん:2011/10/17(月) 16:09:08.60
n!の素因数pの指数はΣ[n/p^i](iは1から∞まで)で与えられる。
ここでnCk=n!/k!(n-k)!について
Σ[n/p^i]-(Σ[k/p^i]+Σ[(n-k)/p^i])が素因数pの指数を与える。
そして、任意の非負実数x,yについて[x+y]≧[x]+[y]が成り立つことは明らかだから上の式を整理した次式についてΣ([n/p^i]-([k/p^i]+[(n-k)/p^i]))≧0
が成り立つことがいえる。
よって任意の素数についてnCkの分子の方が分母よりも多く含むことがわかる、即ちnCkは整数であることがいえる。

これなら問題ない筈
511132人目の素数さん:2011/10/17(月) 19:43:04.60
無限個の和の順番を入れ替えるには問題がある/注意が必要だから、Σの上限は適当な有限の数で抑えておく方が無難かな
512132人目の素数さん:2011/10/17(月) 19:58:18.54
階乗の定義が露骨に帰納的なのに、ここで帰納的な証明を忌避する感覚がわからん
513132人目の素数さん:2011/10/17(月) 21:28:25.03
背理法は避けたい、帰納法は厭だ、とは余りに馬鹿
そんなこと言ってたら論文書けないし
514132人目の素数さん:2011/10/18(火) 00:29:30.69
>>511
適当な有限の数って何だよ
実際には途中から0になるんだからいいんじゃないか?
それともiは[n/p^i]を正整数にする数jまでとかにするのか?
そうしたら他の二つのiも別に設定しなければいけなくなるぞ
心配なら実際には有限和に等しいことをいえば良い気がする
515132人目の素数さん:2011/10/18(火) 00:30:20.96
>>512
わかったから黙ってろ
516132人目の素数さん:2011/10/18(火) 00:50:44.05
>>514
[log_2(n)]とかで十分だと思うが
517132人目の素数さん:2011/10/18(火) 17:02:48.38
>>514
不当な指摘だと言っているのか?
些細な指摘なので、聞くに値しないと言っているのか?
意味が分からないから、詳しく説明しろって言っているのか?
518132人目の素数さん:2011/10/18(火) 17:15:57.69
絶対収束で問題なし&Σ毎に範囲が異なると面倒
519132人目の素数さん:2011/10/18(火) 22:34:17.22
こんなとこで絶対収束とか持ち出すのはばかばかしすぎる
520132人目の素数さん:2011/10/18(火) 22:38:20.06
絶対収束は特別なものじゃない
521132人目の素数さん:2011/10/24(月) 17:07:42.47
∞で良いな
絶対収束を付記すれば問題ないに同意
522132人目の素数さん:2011/10/24(月) 22:05:09.64
ちょっと工夫すりゃ済む話なんだがなぁ
523132人目の素数さん:2011/10/25(火) 00:04:34.09
有限で済む話を意味無く無限にして考えるだけ
「Σ毎に範囲が異なると面倒 」って書いているが、異なる値にする必要はない

いずれに於いてもしなくていいことをわざわざ行い、一方は余計な概念を要求し、
一方は面倒とのたまう。
この様な輩には、レコード大賞を受賞した「○か○」という曲を贈りたい。
524132人目の素数さん:2011/10/25(火) 00:18:20.74
n!のとき[log_2(n)]なら(m^2)!のとき[log_2(m^2)]になったりすると思うが
(m^2)!のときも[log_2(n)]でいいのか
525132人目の素数さん:2011/10/25(火) 13:35:48.62
>>523
悔しいのうwwwww悔しいのうwwwww
526132人目の素数さん:2011/10/26(水) 04:45:12.41
最優秀新人賞の 綾香 「三か月」ですね。 2006年でしたっけ?
527132人目の素数さん:2011/10/26(水) 13:36:01.91
>>526
自演ですか?
そうでなくともわざわざ書かなくていいですよwww
528132人目の素数さん:2011/10/26(水) 23:52:38.07
書かなくてもよい、ということは
書いてはいけない、ということではない。
529132人目の素数さん:2011/10/27(木) 00:33:43.81
書くなって意味だろ
読解しろ
530132人目の素数さん:2011/10/27(木) 06:09:31.45
歪曲表現
531132人目の素数さん:2011/10/27(木) 07:52:50.03
>>514
>それともiは[n/p^i]を正整数にする数jまでとかにするのか?
>そうしたら他の二つのiも別に設定しなければいけなくなるぞ

[n/p^i] が正整数になる最大の数 i を j_1 と置く。
[k/p^i] が正整数になる最大の数 i を j_2 と置く。
[(n-k)/p^i] が正整数になる最大の数 i を j_3 と置く。

このとき

Σ[i=1〜∞][n/p^i]=Σ[i=1〜j_1][n/p^i]
Σ[i=1〜∞][k/p^i]=Σ[i=1〜j_2][k/p^i]
Σ[i=1〜∞][(n-k)/p^i]=Σ[i=1〜j_3][(n-k)/p^i]

と表せて、それぞれのΣで範囲が異なる。
が、ちょっと工夫すれば範囲が統一できる。

上で定義したj_1, j_2, j_3のうち最大のものをj_4と置くと、
i>j_4のとき[n/p^i]=[k/p^i]=[(n-k)/p^i]=0 となる。特に

Σ[i=1〜∞][n/p^i]=Σ[i=1〜j_4][n/p^i]
Σ[i=1〜∞][k/p^i]=Σ[i=1〜j_4][k/p^i]
Σ[i=1〜∞][(n-k)/p^i]=Σ[i=1〜j_4][(n-k)/p^i]

と表せて、どのΣでも同じ範囲「i=1〜j_4」での和となる。
532132人目の素数さん:2011/11/05(土) 11:29:45.42
まだスレが残っていたんですね^^;
533132人目の素数さん:2011/11/13(日) 06:50:24.69
鳩の巣原理(ディリクレの原理)について詳しい本ありますか?
534132人目の素数さん:2011/11/13(日) 15:26:17.64
Wikipedia
535132人目の素数さん:2011/11/14(月) 22:46:03.86
1/x+5/y+8/z=1
こゆ問題好き(´・ω・`)
536132人目の素数さん:2011/11/15(火) 12:51:18.43
その問題は明らかにeffectiveに解けるが
537132人目の素数さん:2011/11/15(火) 20:39:27.63
こゆき(´・ω・`)
538検便のナウシカ ◆UVkh7uHFoI :2011/11/16(水) 18:27:19.68
数列a[n]を
a[n+1]=10*a[n]+3、a[1]=8
で定義します。この時、集合
{4*a[n]-1| nは自然数}
に素数は無数に含まれますか?
539132人目の素数さん:2011/11/16(水) 19:27:11.84
>>538
死ねアホ
540132人目の素数さん:2011/11/16(水) 20:37:21.41
>>538
要するに 31,331,3331,..の形の素数が無限にあるか?
ということだろうが それはよくある未解決問題だよ。
541検便のナウシカ ◆UVkh7uHFoI :2011/11/16(水) 20:41:14.96
>>540
そうなんですか。
そんな難しい問題だったんですか。
考えて損した。
教えてくれてありがとうございました。
542検便のナウシカ ◆UVkh7uHFoI :2011/11/16(水) 20:42:19.47
でもあまりいい問題とは言えないと思うけど、なんで未解決なんだろうね。
543132人目の素数さん:2011/11/16(水) 21:22:17.39
>>542
たとえば、111...11の形の素数が無限個あるかどうかも未解決問題。
あからさまな理由がみえないかぎり、この手のものは大抵未解決さ。
544検便のナウシカ ◆UVkh7uHFoI :2011/11/16(水) 21:39:44.85
>>543
へー、勉強になります。
パッと見高校生のちょっと難しめの演習問題みたいに見えるだけど全然違うんですね。
有名な未解決問題ってことは秀才が考えてもわからないってことだな。
素数が絡むと大抵難しくなるみたいですね。
問題からは見えない、どんな深い真理が眠ってるのか興味ありますね。
ありがとうございました。
545132人目の素数さん:2011/11/16(水) 22:10:53.83
>>543
んなわけねーだろタコスケ
546132人目の素数さん:2011/11/16(水) 22:13:11.35
>>545
うん? なんのつもりですか?^^
それらは未解決問題でないという意味でしょうか?^^
547132人目の素数さん:2011/11/17(木) 00:37:13.80
>>543を否定するとしたら
1) 11...11の形の素数が無限個あるかどうかは既に解決している。
2) あからさまな理由がなくても、この手のものは大抵未解決ではない。
3) 1)2)の両方。
548132人目の素数さん:2011/11/17(木) 01:59:17.74
1) まだ解決していない
http://ja.wikipedia.org/wiki/%E3%83%AC%E3%83%94%E3%83%A5%E3%83%8B%E3%83%83%E3%83%88

2) 大抵未解決である
たとえば、a>1,bをそれぞれ整数の定数とするとき、
b+a^nが素数となるような正整数nが無限に存在するかどうかは
一般には未解決である。
ただし、多項式に由来した分解ができる場合や、
いくつかのmodが機能する場合は未解決とは限らない。
333..31は(10^n-7)/3と表現できるので、この手の問題に属するといえる。
549132人目の素数さん:2011/11/18(金) 14:48:32.12
電波テロ装置の戦争(始)
エンジニアと参加願います公安はサリンオウム信者の子供を40歳まで社会から隔離している
オウム信者が地方で現在も潜伏している
それは新興宗教を配下としている公安の仕事だ
発案で盗聴器を開発したら霊魂が寄って呼ぶ来た
<電波憑依>
スピリチャル全否定なら江原三輪氏、高橋佳子大川隆法氏は、幻聴で強制入院矛盾する日本宗教と精神科
<コードレス盗聴>
2004既に国民20%被害250〜700台数中国工作員3〜7000万円2005ソウルコピー2010ソウルイン医者アカギ絡む<盗聴証拠>
今年5月に日本の警視庁防課は被害者SDカード15分を保持した有る国民に出せ!!<創価幹部>
キタオカ1962年東北生は二十代で2人の女性をレイプ殺害して入信した創価本尊はこれだけで潰せる<<<韓国工作員鸛<<<創価公明党 <テロ装置>>東芝部品)>>ヤクザ<宗教<同和<<公安<<魂複<<官憲>日本終Googl検索
550132人目の素数さん:2011/11/18(金) 14:48:55.61
魂は幾何学


誰かがやった
ソウルコピー機器
551551蓬莱:2011/12/06(火) 00:35:08.37
魂は あんまん、肉まん、アイスキャンデー

http://www.551horai.co.jp/
552132人目の素数さん:2011/12/09(金) 20:31:58.99
九月に出た代数幾何学事始という本、斬新だな
553132人目の素数さん:2011/12/19(月) 00:54:03.29


ワイは日本人やけど聞いた情報によると、もうじき中国はバブルがはじけて昔の貧乏な元の中国に戻るそうやで
みんなも知っての通りでもう経済は破綻してて、取り戻すには無理なんだそうや


その世界ではとても有名な政府関係者筋から聞いた確かな情報やで

君らほど頭の良い連中には、今さらなくらいのネタやな、かえって失礼なくらいな
お前らからすればもう常識的なくらいの知識やろ!!


554132人目の素数さん:2011/12/20(火) 15:01:10.28
f_k(n)を1〜nの中で約数がk個となる自然数の個数とすると
例えばf_2(n)はn以下の素数の個数になりますが
3以上の任意のn,kに対してf_k(n)<f_2(n)が成り立つでしょうか
555:2011/12/21(水) 14:14:12.94
>>554
n,k=3 で f_2(3)=f_3(3)=0 じゃないか。
556:2011/12/21(水) 14:15:22.06
あ、まちがえた!
557:2011/12/21(水) 15:57:06.29
>>554
k が素数なら f_k(n) 個に入る自然数は p^(k−1):p は素数
しかないから f_k(n)<f_2(n) だが、合成数で k=ab・・ (a,bは素数でなくて良い) だと
k の任意の分解に対して p^(a−1) q^(b−1)・・が全部 f_k(n) 個に入るから、あやしくなるな。
k=4=2×2 でも f_2(27)=9, f_4(27)=8 まで迫ってるし。
558 ◆BhMath2chk :2011/12/21(水) 20:00:00.76
n=35,k=4。
12>11。

n=670,k=8。
122>121。

n=57239,k=16。
5805>5804。

n=58121,k=12。
5886>5885。

n=3266950,k=24。
234714>234713。
559:2011/12/22(木) 03:52:24.97
>>558
35 まで見れば f_4(35)=12, f_2(35)=11 だったのか!
しかし、みんな 1 つ違いってのが謎?
560エトス:2011/12/28(水) 02:29:07.53
f_4(39)=14, f_2(39)=12

[問題]
任意の正整数nに対して
Σ[k=1,n]C[2gcd(n,k),gcd(n,k)]は2nで割り切れることを示せ
561132人目の素数さん:2012/01/01(日) 18:50:34.75
>>560
http://www.artofproblemsolving.com/Forum/resources.php?c=200&cid=115&year=2007&sid=5fd55718b10dbc3bdef9a54088183806
に回答があるね(一般化されているけど)。

それでは、以前あったサイトの問題を投下
n-1|2^n +1とn|2^n +2が同時に成り立つ正の整数nが無数にあることを証明せよ。
(ただし、a|bとはaがbを割り切ることを意味する)


562エトス:2012/01/02(月) 02:30:35.74
>>561
ほんとだ。(70)にありますね。
ただ、私の用意していた回答は本質的に異なります。

その問題の回答を考えてみました。
基本的なアイデアは
d|s かつ v_2(d)=v_2(s) ⇒ 2^d+1|2^s+1 だけです。
(v_2(n)は2^e|n を満たす最大の整数eを表すとします)

[回答]
n-1|2^n+1 かつ n|2^n+2 を満たす整数n≡2(mod 4)を任意に取る。
(たとえば、n=2 がある)
m=2^n+2>n とおくとき、m≡2(mod 4)であり、
(このとき、n|m かつ n-1|m-1 が成立しているが)
m-1|2^m+1 かつ m|2^m+2 が成立している。

実際、m-1=2^n+1 であるから、n|m より、m-1|2^m+1 であり、
(n≡m≡2 (mod 4)に注意)

また、m/2=2^(n-1)+1 であるから、n-1|m-1 より、
m/2|2^(m-1)+1 なので、m|2^m+2 となる。
(n-1≡m-1≡1 (mod 2)に注意)
563エトス:2012/01/02(月) 02:34:40.52
実はn>1ならば2nの部分は4nに修正できます。

[問題]
任意の整数n>1に対して
Σ[k=1,n]C[2gcd(n,k),gcd(n,k)]は4nで割り切れることを示せ

これで一応被っていないとおもいます・・・w
564132人目の素数さん:2012/01/11(水) 18:05:42.01
では数論の問題を。前半は難しくない。

i) p を p ≡ 3 (mod.4) なる素数とする時、
 ((p - 1)/2)! = 1・2・3・・ ... ・・・(p-1)/2 ≡ ±1 (mod.p)
を示せ。

後半は熊さんでもすぐ解けるかどうか。猫には当然分かりっこない。

ii) 上記の時、何時 (p - 1)/2! ≡ 1 (mod.p) になり、いつ (p - 1)/2! ≡ -1 (mod.p) になるか決定せよ。
565訂正:2012/01/11(水) 18:09:09.48
ii) 上記の時、何時 ((p - 1)/2)! ≡ 1 (mod.p) になり、いつ ((p - 1)/2)! ≡ -1 (mod.p) になるか決定せよ。
566132人目の素数さん:2012/01/11(水) 18:28:29.11
>>564
オリジナルティがない
2つとも有名問題

(A)についてはたとえば次のようにいえる
問題の符号は 0<x<p/2 なる整数xのうち、
mod pでみたときに平方数でないものの個数のパリティに対応する。
偶数ならばプラスであり、奇数ならばマイナスである。

(@)は単にp/2を境にして(p-1)!をわければいいだけ
567検便のナウシカ ◆UVkh7uHFoI :2012/01/11(水) 18:30:53.36
このスレの人たちってほんとすごいよね。
初等整数論の問題は何でも知ってる。
568132人目の素数さん:2012/01/11(水) 20:14:44.30
>>566
もっと簡明に答えろ

i) だって何処に仮定を使っているか分からん
569132人目の素数さん:2012/01/11(水) 20:16:35.64
うるせえ!
570132人目の素数さん:2012/01/11(水) 20:18:01.36
>>566
ii) この解答の表現の意味が良く分からんが多分間違っている。
571あのこうちやんは始皇帝だった:2012/01/11(水) 20:18:28.61
>>566

 コイツ、60歳の、無職の、頭デッカチの虚弱児・ひ弱な、ゴミ・クズ・カスの、
クソガキ!!!!!!!!!!!!!!!!!!!!!!!!!!!!!!!!!!!!!!!!
572132人目の素数さん:2012/01/11(水) 20:20:19.40
発作マグナ
573132人目の素数さん:2012/01/12(木) 02:40:48.45
>>564
>>566さんが解いてるけど、わかってない人がいるから詳しくw
(1)
ウィルソンの定理(p-1)!≡-1 (mod p)を使う。
({(p-1)/2}!)^2≡(-1)^{(p-1)/2}・1・2・...・{(p-3)/2}・{(p-1)/2}・{p-(p-1)/2}・{p-(p-3)/2}・...・(p-2)・(p-1)
≡(-1)^{(p-1)/2}・(p-1)!≡(-1)^{(p+1)/2}≡1 (mod p)
したがって、{(p-1)/2}!≡±1 (mod p)がいえる。
(p≡3 (mod 4)だから、(p+1)/2が偶数になるのがポイント!)

余談だが、もしp≡1 (mod 4)であれば、上と同様にして({(p-1)/2}!)^2≡-1 (mod p)がいえるから
p≡1 (mod 4)のとき、x^2≡-1 (mod p)の整数解が求められる。

(2)
(1)より、({(p-1)/2}!)^2≡1がいえている。また(p-1)/2は奇数だから
{(p-1)/2}!≡({(p-1)/2}!)^{(p-1)/2}≡(1/p)・(2/p)・...・({(p-1)/2}/p)
ちなみに(1/p),(2/p),…,({(p-1)/2}/p)はルジャンドルの記号
よって>>566さんの言うとおり、1、2、…、(p-1)/2のうちで平方剰余の個数が
偶数であれば{(p-1)/2}!≡1 (mod p)、奇数であれば{(p-1)/2}!≡-1 (mod p)となる。
574573:2012/01/12(木) 15:55:06.90
>>573
あらら、(2)で(mod p)書き忘れたw
({(p-1)/2}!)^2≡1 (mod p)がいえている。また(p-1)/2は奇数だから
{(p-1)/2}!≡({(p-1)/2}!)^{(p-1)/2}≡(1/p)・(2/p)・...・({(p-1)/2}/p) (mod p)
だなw
575132人目の素数さん:2012/01/12(木) 18:19:17.56
さらにいうと平方剰余の個数じゃなくて、
非平方剰余の個数だとおもわ
576573:2012/01/13(金) 00:14:58.91
>>575
そうだなw
577エトス:2012/01/13(金) 01:34:35.57
もう1題追加☆

[問題]
f,g∈Z[X]が互いに素であるならば,
max[x∈Z]gcd(f(x),g(x))が存在する.
(これを証明せよ)
578132人目の素数さん:2012/01/13(金) 01:52:13.13
>>566>>573-576
そんな事は最初から分かっている。だから例えば
p = 100003
の時に簡単に分かる表式を与えよと云っているのだ。
579エトス:2012/01/15(日) 19:07:22.77
(問題)
「f(X):=X^5-10X^3+5X^2+10X+1∈F_p[X]
f(X)はF_pの中に根を持たないなら可約」
というのは正しいか.証明または反証せよ.
580132人目の素数さん:2012/01/15(日) 20:10:00.25
p=2のとき既約なので正しくない。
581エトス:2012/01/15(日) 20:37:00.97
すみません!
タイプミスでした.

(問題)
「f(X):=X^5-10X^3+5X^2+10X+1∈F_p[X]
f(X)はF_pの中に根を持たないなら既約」
というのは正しいか.証明または反証せよ.
582132人目の素数さん:2012/01/15(日) 20:57:00.55
可約なら全て一次にまで分解できるようだ。
583エトス:2012/01/15(日) 21:05:02.78
>>582
その結論は正しいです.

当然,問題とは同値ではないですが.
それも加えて問題としておきますミ☆
584エトス:2012/01/15(日) 21:10:15.45
可約なら1次の積に分解できるの主張のほうが強いですね.失礼.

(問題)
「f(X):=X^5-10X^3+5X^2+10X+1∈F_p[X]
f(X)はF_pの中に根を持つなら,F_p[X]で完全分解される」
この命題が正しいことを証明せよ.
(ネタバレがあったのでこちらにしておきます)
585132人目の素数さん:2012/01/15(日) 21:35:05.19
5,4+1,3+2,3+1+1,2+2+1,2+1+1+1,1+1+1+1+1

>>581 3+2がない
>>582 4+1,3+2,3+1+1,2+2+1,2+1+1+1がない
>>584 4+1,3+1+1,2+2+1,2+1+1+1がない
586エトス:2012/01/15(日) 21:40:17.38
それらの分解が起こりえないことを示すのが問題になりますね.
587132人目の素数さん:2012/01/16(月) 22:19:50.65
[>>582]=[>>581]and[>>584]
588132人目の素数さん:2012/01/16(月) 23:00:00.40
a。
(a^4+3a^3−8a^2−19a+9)/7。
(−2a^4+a^3+23a^2−18a−25)/7。
(−3a^4−2a^3+24a^2−6a−6)/7。
(4a^4−2a^3−39a^2+36a+22)/7。
589エトス:2012/01/16(月) 23:20:44.39
>>588
正解です.

[略解]
αを1の原始25乗根として,β=α+1/α+α^7+1/α^7 とします.
また,L=Q(α),E=Q(β) とします.
L/Qは巡回拡大だから,E/Qも巡回拡大となります.
βのQ上の最小多項式が問題のfに相当するわけです.
ですからF_pへのreductionを考えれば結論が得られます.
590エトス:2012/01/17(火) 03:24:45.02
一応,補足しておきます.(F_pへのreductionの詳細です)
以下,p>11とします.
このとき,fが重根を持たないことがいえます.
fのF_p上での最小分解体をKとします.
Gal(K/F_p)はGal(E/Q)の部分群とみなせます.
よって,Gal(K/F_p)は{id}またはZ/5Zと同型です.
前者の場合は明らかにfはF_p[X]で完全分解され,
後者の場合はfはF_p[X]で既約であるといえます.
(∵もし可約ならばfはあるh次(1<h<5)の既約因子で
割り切れるとなるが,それは h|[K:F_p]=5 に矛盾する)
591132人目の素数さん:2012/01/17(火) 12:17:50.23
自然数 n に対して次の条件を考える。

∀p : prime [ p|n ⇔ (p - 1)|n ]

この条件を満たす自然数は高々二つの例外を除いて
必ず素数の平方で割り切れる。
(事を示せ。)
592エトス:2012/01/17(火) 15:49:02.95
>>591
[証明]
n=1806=2*3*7*43 は条件を満たします.
整数n>1が条件を満たしていると仮定します.
このとき,1806|n が成立が確認できます.
n≠1806かつnは平方因子を含まないとします.
gcd(1806,q)=1を満たすnの素因数で最小のものをqとします.
(nが平方因子を含まないことからそういうものが取れます)
このとき,条件より,q-1|n がいえますが,
qの定め方より,q-1の素因数は1806を割り切ります,
nが平方因子を含まないこととあわせて,q-1|1806 がいえます.
再び条件より,q|1806 がいえますがこれは明らかに矛盾です.
(n=1,1806 の2つが例外となります)               ■ 
593エトス:2012/01/17(火) 16:00:48.74
失礼.証明の一部分に微妙な飛躍がありました.
最後の下から2行目は次のように訂正します.
(q-1|1806 まではO.K.なのでその続きから)

ここで,n=1806が条件を満たすことを用いることで,
q|1806 がいえますが,これは明らかに矛盾です.
594132人目の素数さん:2012/01/17(火) 16:30:00.21
n=1は条件を満たさない。
595132人目の素数さん:2012/01/17(火) 16:37:13.39
haa
596エトス:2012/01/17(火) 16:37:38.54
>>594
そうですね!
でも安心してください解答自体に欠陥は無いとおもいます!

[問題]
原始n乗根の1つをsとし、Q(s)の整数環をAとします.
このとき,(1-s)^2|n がAの中で成立している.(n>2)
597エトス:2012/01/17(火) 16:40:15.35
あ,1の原始n乗根です.
598132人目の素数さん:2012/01/17(火) 17:24:12.12
二つの例外ってなんだったんだろう
599エトス:2012/01/17(火) 17:46:55.02
あえて言うならば0だとおもいます
0を自然数に含めることもありますので
600132人目の素数さん:2012/01/17(火) 17:58:58.42
2^2|0
601エトス:2012/01/17(火) 18:16:12.84
そうですね.0は平方因子をもちます.
問題文の真意ははかりかねますが,
用心深く"高々"2つとつけられたのでしょうかw
602132人目の素数さん:2012/01/18(水) 08:38:45.90
∀p : prime [ p|n ⇔ (p - 1)|n ]
を満たす自然数 n を少なくとも四つ求めよ。
電卓・パソコンの使用可
603132人目の素数さん:2012/01/18(水) 10:00:00.83
0,2・3・7・43,2・3・7^2・43,2・3・7^3・43。
604132人目の素数さん:2012/01/18(水) 11:00:00.40
2・3・7・43^2・77659。
605132人目の素数さん:2012/01/18(水) 12:09:49.15
みんな暇だな〜
では、少し骨のある問題。

f (x), g (x) を定数でない整係数多項式とする。この時、条件:

∃m ∈ Z ∃n ∈ Z [ p|f (m), p|g (n) ]

を満たす素数 p は無限に存在することを示せ。
606132人目の素数さん:2012/01/18(水) 14:00:00.35
2・3・7・43^3・77659。
2・3・7・43^4・77659。
2・3・7・43^5・77659。
2・3・7・43^6・77659。
2・3・7・43^7・77659。
2・3・7・43^8・77659。
2・3・7・43^9・77659・21108889701347407
・5474088843701260097485589623。
2・3・7・43^10・77659・21108889701347407
・5474088843701260097485589623。
2・3・7・43^11・77659・21108889701347407
・5474088843701260097485589623。
2・3・7・43^12・77659・21108889701347407
・5474088843701260097485589623。
2・3・7・43^13・77659・21108889701347407
・5474088843701260097485589623。
607132人目の素数さん:2012/01/18(水) 19:26:23.03
608132人目の素数さん:2012/01/18(水) 19:36:52.41
この様な数は無限に多くあるのだろうか?
609605:2012/01/18(水) 20:43:07.90
これは少し難しすぎたかな?では易しいのを一題。

a, b を整数とし、数列 {F_n} を次の様に定義する:
F_0 = 0, F_1 = 1, F_(n+2) = a*F_(n+1) + b*F_n.
この時任意の k, n に対して
F_1*F_2* ..... *F_k | F_(n+1)*F_(n+2)* ..... *F_(n+k)
(整除関係) を示せ。
610エトス:2012/01/18(水) 22:28:27.64
>>605
Chebotarev's Density Theorem
を使える形にもっていけばおわりそうです
611132人目の素数さん:2012/01/18(水) 23:13:47.40
F(n,k)=Π_[i=1,n+k}F(i)/{Π_[i=1,n}F(i)・Π_[i=1,k}F(i)}とおく。
(便宜上、Π_[i=1,0}F(i)}=1とおく、0!=1みたいなものです)
するとF(0,0)=1、F(n,0)=F(n,n)=1、F(n+1,k+1)=F(n,k)・F(h+1) +F(n,k+1)・{bF(k)}
から帰納的に、F(n,k)が整数であることが分かる。

僕からも問題
a,bを互いに素な、a^2-4b≠0をみたす整数とする。
F(0)=0、F(1)=1、F(n+2)=aF(n+1)+bF(n)をみたす数列F(n)とG(0)=2、G(1)=1、G(n+2)=aG(n+1)+bG(n)をみたす数列G(n)をとる。
d=gcd(m,n)とするとき、gcd{F(n),G(m)}が下記のようになることを証明せよ。
(1) gcd{F(n),G(m)}=G(d) (m/dが偶数のとき)
(2) gcd{F(n),G(m)}=1あるいは2 (m/dが奇数のとき)
612エトス:2012/01/18(水) 23:16:02.08
f,gは互いに素な既約多項式として一般性を失いません.
f,gを分解するようなQ上のGalois拡大をEとします.
f,gの根(の1つ)をそれぞれα,βとします.
density theoremより,Eにおいて完全分解される
有理素数の密度は1/[E:Q]>0 ですから,
Q(α),Q(β)のそれぞれの中で完全分解されるような
有理素数が無限に取れます.
F_pへの還元を考えることで結果が従うといえます.
613エトス:2012/01/18(水) 23:26:27.34
[問題]
次の条件を満たすようなf:N→Z は存在するか
(ここで,Nは正整数全体の集合としておきます)

[条件]
|Im(f)|=18 であり,任意の正整数nに対して,
f(n+6)=Σ[i=0,5]a_i*f(n+i)を満たすような
整数定数a_0,a_1,..,a_5 の組が取れる。
614611:2012/01/18(水) 23:51:44.44
>>611>>609の回答です・
615132人目の素数さん:2012/01/19(木) 01:12:03.70
a=3, b=1
F(0)=0, F(1)=1, F(2)=3, F(3)=10
G(0)=2, G(1)=1, G(2)=5
gcd{F(3),G(2)}=5≠G(1)
616611:2012/01/19(木) 01:37:20.12
>>615
ごめん >>611の問題は以下のように訂正するわw
問題:a,bを互いに素な、a^2-4b≠0をみたす整数とする。
F(0)=0、F(1)=1、F(n+2)=aF(n+1)+bF(n)をみたす数列F(n)とG(0)=2、G(1)=1、G(n+2)=aG(n+1)+bG(n)をみたす数列G(n)をとる。
d=gcd(m,n)とするとき、gcd{F(m),G(n)}が下記のようになることを証明せよ。
(1) gcd{F(m),G(n)}=G(d) (m/dが偶数のとき)
(2) gcd{F(m),G(n)}=1あるいは2 (m/dが奇数のとき)
617132人目の素数さん:2012/01/19(木) 01:42:14.82
???
618132人目の素数さん:2012/01/19(木) 01:44:13.67
[問題]
 u=5sin(ωt-π/3)を正弦と余弦の成分に分解せよ
619132人目の素数さん:2012/01/19(木) 01:45:19.26
>>617
mとnが逆になっている部分があるから修正されている
620132人目の素数さん:2012/01/19(木) 01:52:19.20
それでも615が反例になっとるな
621611:2012/01/19(木) 02:20:28.39
ごめん、またやっちゃったなw

「a,bを互いに素な、a^2-4b≠0をみたす整数とする。
F(0)=0、F(1)=1、F(n+2)=aF(n+1)+bF(n)をみたす数列F(n)とG(0)=2、G(1)=a、G(n+2)=aG(n+1)+bG(n)をみたす数列G(n)をとる。
d=gcd(m,n)とするとき、gcd{F(n),G(m)}が下記のようになることを証明せよ。
(1) gcd{F(n),G(m)}=G(d) (m/dが偶数のとき)
(2) gcd{F(n),G(m)}=1あるいは2 (m/dが奇数のとき) 」

G(1)の値が違ってた、これで問題ないと思うw
622132人目の素数さん:2012/01/19(木) 02:56:02.61
もういい、さがれ
623132人目の素数さん:2012/01/19(木) 22:36:41.15
>>621は、http://note.chiebukuro.yahoo.co.jp/detail/n5829
の定理9の内容そのものだね。
624132人目の素数さん:2012/01/19(木) 22:38:55.60
いや、漸化式が間違ってる
625611:2012/01/19(木) 22:40:35.72
>>621も間違いかw
たびたび間違えたな。正確にはこうだ。
「a,bを互いに素な、a^2-4b≠0をみたす整数とする。
F(0)=0、F(1)=1、F(n+2)=aF(n+1)+bF(n)をみたす数列F(n)とG(0)=2、G(1)=a、G(n+2)=aG(n+1)+bG(n)をみたす数列G(n)をとる。
d=gcd(m,n)とするとき、gcd{F(m),G(n)}が下記のようになることを証明せよ。
(1) gcd{F(m),G(n)}=G(d) (m/dが偶数のとき)
(2) gcd{F(m),G(n)}=1あるいは2 (m/dが奇数のとき) 」
上記のとおり、回答はhttp://note.chiebukuro.yahoo.co.jp/detail/n5829
の定理9の内容そのものだから、関心がある人は見てくれw
626132人目の素数さん:2012/01/19(木) 22:44:04.56
どうでもよいけどその問題をここに挙げた人と
知恵袋puit578さんは中の人が同じだとおもわれ
適当な2次の数体の整数環の中で議論すれば
ノートの証明はいくらか縮約できるとおもうがどうでもよい
627611:2012/01/19(木) 22:52:22.18
>>611の問題は却下します。間違いまくってすみませんでした。新たに問題を投下します。
問題:a、b、nを正の整数(ただし、a>bとする)、φ(n)をオイラーのトーティエント関数とする。
φ(a^(n-1) +a^(n-2)・b+…+b^(n-1) )はnで割り切れることを証明せよ。
628132人目の素数さん:2012/01/19(木) 23:05:37.33
>>627
gcd(a,b)=1のときに正しいことを示せば十分.
m=(a^n-b^n)/(a-b) とおく.
すぐわかるように(m,ab)=1である.
以下,G:=U(Z/mZ)の中で考える.
c=a/b とおくと,<c>は位数nのGの部分群である.
証明ここまで.

629エトス:2012/01/19(木) 23:12:20.38
さらにもう一題追加☆

(問題)
f∈Q[X]を既約とし,n:=deg(f)>2とする.fの最小分解体をLとするとき,
n=[L:Q]ならばf(X)|f(g(X))かつ1<deg(g)<nを満たすg∈Q[X]が存在する.

630エトス:2012/01/19(木) 23:19:05.02
上のはちょっと微妙なので.
(次数が1より大きいという部分以外はほとんど明らかに思えるから)

(問題)
F_p(p>2)上の方程式:ax^4+by^4=cz^2(abc≠0)は非自明な解を持つ

これなら少しマシだとおもいます.
631エトス:2012/01/19(木) 23:33:51.20
>>627

(別解)
gcd(a,b)=1としておく.
Zsigmondy Theoremより,
いくつかの特殊な場合を除いて,
http://mathworld.wolfram.com/ZsigmondyTheorem.html
a^n-b^nはord_p(a/b)=nを満たす素因数pを持つ.
証明ここまで.(いくつかの特殊な場合は別に検証する)
632エトス:2012/01/20(金) 00:08:21.63
[改題]
n^{τ(n)/2}|φ((a^n-b^n)/(a-b))

n^{τ(n)/2}はnの正の約数の全ての積だから,
問題はZsigmondyの定理からほとんど明らかです.
(いくつかの特殊なケースを注意深く処理してください)
たとえば,n=12,a=2,b=1とすると,
n^{τ(n)/2}=12^3
φ((a^n-b^n)/(a-b))=φ(2^12-1)=12^3
偶然にも等号まで成立しています.
633132人目の素数さん:2012/01/20(金) 14:27:28.12
では、あのダージも解けなかった問題。君たちには解けるか?

f (x) を整係数多項式とする。その n 回 iteration f^n (x) を
f^1 (x) = f (x), f^(n+1) (x) = f (f^n (x)) で帰納的に定義する。
m, n を互いに素な自然数とするとき、整係数多項式環 Z [x] における次の整除関係を示せ。

(f^m (x) - x)*(f^n (x) - x) | (f^(mn) (x) - x)*(f (x) - x)
634132人目の素数さん:2012/01/20(金) 14:44:24.48
ついでにもう一題

f_i (x), i = 1, 2, ... , n を複素係数多項式、α_i, i = 1, 2, ... , n を複素数とする。
d = Σ[i = 1 → n] dim f_i (x), 更に自然数 m に対し複素数 F (m) を
F (m) = Σ[i = 1 → n] f_i (m)*α_i^m
で定義する。もしこの時 F (m), m = 1, 2, ... , d + 1 が全て(有理)整数なら、
任意の m に対して F (m) は(有理)整数。
635132人目の素数さん:2012/01/20(金) 16:07:47.84
>>633
明らかに任意の正の整数s,t(s>t)に対して次がいえる
f^s(x)-x=f^(s-t)(f^t(x))-f^(s-t)(x)+f^(s-t)(x)-x

f^t(x)-x|f^(s-t)(f^t(x))-f^(s-t)(x) だから
f^(s-t)(x)-x∈(f^s(x)-x),f^t(x)-x) (2元で生成されるZ[x]のイデアル)

m,nは互いに素であったので 上記のことより
f(x)-x∈(f^m(x)-x),f^n(x)-x) がいえる
またf^m(x)-x,f^n(x)-xはともにf(x)-xで割り切れるので
gcd((f^m(x)-x)/(f(x)-x),(f^n(x)-x)/(f(x)-x))=1
ここで (f^m(x)-x)/(f(x)-x)|(f^mn(x)-x)/(f(x)-x)
と (f^n(x)-x)/(f(x)-x)|(f^mn(x)-x)/(f(x)-x)
の成立はほとんど明らかだが(分母を払えばすぐ確認できる)
2つの因子は互いに素であることは先ほど示したので
したがって証明が完了したといえる
636635:2012/01/20(金) 16:12:49.38
>>633
問題を引用するときは出典をかきましょう(オリジナルでない限り)

上の問題はここにある
http://www.artofproblemsolving.com/Forum/viewtopic.php?f=61&t=412797

自分の書いた解答はほぼGreenKeeperさんに因んでいる
(彼のつまらないミスを修正したバージョンに過ぎない)
637エトス:2012/01/20(金) 21:13:27.69
>>625
定理5の証明は一般項の形をそのまま利用すれば早いです.
u(m)=v(m-n)*u(n)±b^s*u(±(2n-m))
(s=min(m-n,n)で,s=m-nのときプラスを取り,それ以外はマイナス)
|2n-m|<mですから,定理1とm+nに関する帰納法で簡単に示せます.
とくに|b|=1のときは定理1に相当するものすら不要となります.
(たとえばfibonacciとかlucasとか)
638611:2012/01/20(金) 23:16:21.93
>>637
アドバイスありがとうございます。

>>632の回答ですが、 特殊なケースというのは
(1)a+bが4以上の2のベキ乗かつ2|nのとき、(2)a=2、b=1かつ6|nのとき
ですね。
(1) Zsigmondyの定理から、1,2以外のnの約数dに対してord_p(a/b)=dをみたす素数p(≠2)の存在がいえるから
(n^{τ(n)/2})/2|φ((a^n-b^n)/(a-b))がいえる。 ここで(a^n-b^n)/(a-b)は明らかに4で割り切れるから、
n^{τ(n)/2}|φ((a^n-b^n)/(a-b)) がいえた。

(2)
Zsigmondyの定理から、1,6以外のnの約数dに対してord_p(2)=dをみたす素数pの存在がいえるから、
(n^{τ(n)/2})/6|φ(2^n -1)がいえる。
ここでord_7(2)=3であるから、(n^{τ(n)/2})/3|φ(2^n -1)がいえる。
また、同様に2,6以外のnの約数dに対してord_p(2)=dをみたす素数p(≠3)の存在がいえるから、
(n^{τ(n)/2})/12|φ(2^n -1)がいえる。ここで2^n -1は9で割り切れるから、(n^{τ(n)/2})/4|φ(2^n -1)がいえる。
以上より、(2)の場合もn^{τ(n)/2}|φ((a^n-b^n)/(a-b))がいえた。
639132人目の素数さん:2012/01/21(土) 14:08:49.63
>>636
ダージの一言で分かれば問題ないじゃん
640132人目の素数さん:2012/01/22(日) 00:32:30.36
test
641132人目の素数さん:2012/01/22(日) 02:55:25.86
>>638
36|φ(2^6-1)
2|φ(a^n-b^2) (2|a+bのとき)
の2通りを確かめるだけでも十分

>>577
Q[X]はPIDだからとくに
pf+qg=1 を満たすp,q∈Q[X]の組が取れる
np,nq∈Z[X]を満たすn∈Z(n≠0)を取ると
npf+nqg=n となるので
各整数kに対して gcd(f(k),g(k))|n がいえる
642132人目の素数さん:2012/01/22(日) 13:49:00.15
Chebotarev's Density Theorem
とか
Zsigmondy Theorem
難しすぎ
643132人目の素数さん:2012/01/22(日) 16:59:53.75
Z_p を p - 進整数環、 Z_p (×) Z_p を Z 上のテンソル積、P : Z_p (×) Z_p → Z_p を積写像とする。
この時 P は環準同型となるが、 Ker P は 0 では無く、 Abel 群として divisible であることを示せ。
644132人目の素数さん:2012/01/22(日) 17:18:34.98
>>643
KerPがゼロでないのはZ_pの構造からほとんど明らか
Abel群としてinjective groupであることも少し考えればわかる

スレチの内容が増えてきたのはいかん
解法はなにをつかってもいいのおもうのだが
せめて問題文は初等整数論の言葉で書かれるべきだろ
645132人目の素数さん:2012/01/22(日) 17:35:10.53
>Abel群としてinjective groupであることも少し考えればわかる
だったら理由を一言書けよ。
646132人目の素数さん:2012/01/22(日) 17:50:14.35
付け足し。
∩_n (Ker P)^n を求めよ。
簡単に求まるかな?
647132人目の素数さん:2012/01/22(日) 18:05:27.07
一般lucas数列(u(n))において、
n>30ならばu(n)はprimitiveな素因数を持つ
これを証明せよ
(u(n)のprimitiveな素因数とはとくにnより小さい全ての自然数kに対して
u(k)を割り切らないu(n)の持つ素因数ことである)

これは問題文は誰だってわかるが知られている解法は高度で
代数的整数論を使いこなす必要性がある
このような問題も >>1 からスレチとみなすべきだろう
重要なのは初等的な問題文とある程度のオリジナル性と
なるべく初等整数論的解法が事前に用意されていること
初等的な言葉で書かれている>オリジナル性>初等的な解法
648132人目の素数さん:2012/01/22(日) 18:29:40.40
フィボナッチ数列類似の多項式列{F(n)}を次のように定義する:
F(0) = 0, F(1) = 1, F(n+2) = F(n+1) + x^n F(n).
このとき、任意の自然数nに対して、
F(5^n) は 1 + x + x^2 + … + x^(5^n - 1) で割り切れることを示せ。
649132人目の素数さん:2012/01/23(月) 18:40:50.14
一度高い山まで登ってしまったら
わざわざ降りていき麓を観察するなどはしないが
たとえばフェルマの小定理は改めてみてもとても美しい
たとえばこれがほとんど明らかに感じられるようになっていたとしても
650132人目の素数さん:2012/01/24(火) 19:06:30.85
>>596
n=Π_[i=1,n-1](1- s^i)かつ1-s|1- s^iより、(1-s)^(n-1)|nがいえる。とくに,(1-s)^2|nがいえる。
651132人目の素数さん:2012/01/24(火) 23:49:52.91
(2^n+1)/(n^2)が自然数となる自然数nをすべて挙げろ
652132人目の素数さん:2012/01/25(水) 00:05:20.36
>>651
>>1 を1回か3回読みなされ
IMOの問題は有名すぎるよ
653132人目の素数さん:2012/01/25(水) 00:07:36.03
一応解法の流れを書くと

(@)
n|2^n+1 かつ n>1 ならば 3|n がいえる

(A)
n^2|2^n+1 ならば nは9で割り切れない

(B)
n^2|2^n+1 かつ n>3 ならば nは3以外の素因数をもつ

(W)
矛盾が導かれる
654132人目の素数さん:2012/01/25(水) 02:12:42.93
655132人目の素数さん:2012/01/25(水) 18:00:02.20
(n^2)|(a^n+1)となる1より大きい整数nが存在しない整数aを求めよ。
656132人目の素数さん:2012/01/25(水) 18:42:29.85
>>655
a>2であるならば、Zsigmondyの定理を用いることで、
n^2|a^n+1 を満たす整数n>1が存在するといえる。
(無限に存在することも当然いえる)
これだけで終わってもいいが、以下そのことの詳細。
(別にこの定理を用いなくても説明がつくが
つかったほうが説明が早いのでそういう意味で賢いし、
そもそもこの定理の証明は初等数論の範囲で説明がつく
ただし計算が少々面倒な証明方法しか私は知らないが)
657132人目の素数さん:2012/01/25(水) 18:50:07.01
>>656
おっと、誤った情報を吐いていた。
脳がバグっていたようだ大幅に訂正しておく。

(claim)
a+1が2のべき乗であるとき、問題の整数n>1は存在しない。
a>2かつa+1が2のべき乗でないとき、問題の整数n>1は存在する。
とくに無限に存在することがZsigmondyの定理からすぐに示せる。
658132人目の素数さん:2012/01/25(水) 18:56:02.59
(@)a+1が2のべき乗のときを考える。
a=1のときは明らかである。a>1とする。
n^2|a^n+1 を満たす整数n>1が存在したと仮定する。
nを割り切る最小の素因数をpとする。
p|a^k+1を満たす最小の正の整数kを考えると、
すぐわかるように k|(p-1)/2 かつ k|n がいえる。
pの最小性から、k=1がいえるので、p|a+1 がいえた。
a+1は2のべき乗だったので、p=2かつ2|nがいえた。
しかし、4|a^n+1 は明らかに成立しない。
claimの前半が正しいことが示された。
659β:2012/01/25(水) 19:00:57.31
claim(笑)
いちいち英語使ってドヤ顔せんでええから
660132人目の素数さん:2012/01/25(水) 19:03:57.33
あ、ちょっと変だな。
nが奇素因数を持っていたと仮定して矛盾を導くほうがよかったな。
そうすれば、5〜8行目はきちんと機能する。

次はa+1が2のべき乗でない場合を。
661132人目の素数さん:2012/01/25(水) 19:06:39.26
>>656
それは間違い、a+1が2のベキ乗のとき、つまりa=2^m -1のときは、
n^2|a^n +1をみたすnはn=1のみであることがいえる。

証明の流れとしては
nが偶数のとき
4|n^2、a^n +1≡2 (mod 4)だからn^2|a^n +1とはなりえない。

nが奇数のとき
n>1の最小の素因数pをとる。
a^n +1≡0 (mod p)がいえる。
a^(p-1)≡1 (mod p)とa^(2n)≡1 (mod p)、gcd(p-1,2n)=2よりa^2≡1 (mod p)がいえる。
よって、a≡±1 (mod p)がいえるが、いずれにせよpが2の約数となり、矛盾する。

また、a>2でかつa+1が奇素数pを素因数にもてば
pがn^2|a^n +1をみたすことは容易に示せるね。

ついでにいえば、このとき、n^2|a^n +1をみたすnが無数にあることも以下のように言える。

a^p +1がp以外の奇素数の素因数qを持つことが言える
(Zsigmondyの定理を使うと自明だけど、使わなくても出来る)
したがって、pqもn^2|a^n +1をみたすことがいえる。

上記のような議論を繰り返せば、n^2|a^n +1をみたす整数が無数にあることが言える。
662β:2012/01/25(水) 19:09:24.37
ほらな
ドヤ顔するやつは間違えるんだ
663132人目の素数さん:2012/01/25(水) 19:13:10.36
そっか。じゃあカタカナにすればよかったかな。クレイム。

a+1が2のべき乗でないときを考える。
a+1を割り切る奇素因数pが取れる。
このとき、p^2|a^p+1 が成立している。
(これを確認するにはp|(a^p+1)/(a+1)を確認すればよいが
それはa≡-1(mod p)からすぐに確認できる)

存在はいえたから問題はこれでお終いとしていいけれど、
さらにa>2を仮定すれば、Zsigmondyの定理を使うことで
ここから無限に存在することがすぐいえる。

ある奇数の整数d>1に対して、
d^2|a^d+1 が成立していたとする。
Zsigmondyの定理より、a^d+1にはprimitiveな奇素因数pが存在する。
それは明らかにdと互いに素である。(なぜなら、2d|p-1 だから)
p^2|a^(pd)+1 は簡単に確認できる。(前とほぼ一緒の方法で)
ここで、d^2|a^d+1 だったから、
gcd(p,d)=1 かつ a^d+1|a^(pd)+1 (p,dはともに奇数だから)
まとめて(pd)^2|a^(pd)+1 がいえた。
664132人目の素数さん:2012/01/25(水) 19:17:07.66
>>661
すまんね内容被らせることを書かせてしまって!
665132人目の素数さん:2012/01/25(水) 19:23:59.85
Zsigmondyの定理は使って早いなら使ったほうがいい
その程度で終わる問題があったとしたら、
たとえば使わないで解いたとしても所詮その程度の問題なんだから!
666132人目の素数さん:2012/01/25(水) 19:31:36.04
○たとえ使わないで
×たとえば使わないで

L_1=1, L_2=3, L_(n+2)=L_(n+1)+L_(n)
によって数列(L_n)を定めるとき、
n^2|L_n を満たす正の整数nは無限に存在するか。

×無数に存在する
○無限に存在する

む‐すう【無数】
[名・形動]数えきれないほど多いこと。
667132人目の素数さん:2012/01/25(水) 22:01:11.53
aが負のときは?
668132人目の素数さん:2012/01/25(水) 22:02:05.54
Zsigmondyって何て読むの?シグモンディ?ジグモンディ?ツィグモンディ?
669132人目の素数さん:2012/01/25(水) 22:12:08.07
nが偶数のときは考える必要がないので
nが奇数だけを動くとしてよい

だから結局aが負のときは n^2|a^n-1 (a>1)
をかわりに考えればよい
やはりこれも同様の議論で解けるので無問題
670132人目の素数さん:2012/01/25(水) 22:40:37.71
>>669
n^2|a^n-1 (a>1) をみたすnだね。

結論だけ書くと
a=2のとき、解なしになるんだよね。
そして、a≧3のときのとき、正の整数nは無数にあることがいえる。
671エトス:2012/01/25(水) 22:47:03.90
>>650
(X^n-1)/(X-1) = Π[1≦i<n](X-ζ^i)
そういわれればそうですね...
相応しくない回答しか用意していませんでした.

>>666
R=Z[φ],A=Z[φ]/pZ[φ]とします.(p:素数)
φ=(1+√5)/2 のAでの位数をみればいいでしょう.
(AはF_p(√5)に自然に同型で,√5∈F_pでないなら,
さらにGal(p^2)と同型になります.)
つまりはn^2|2^n+1の問題と本質的に同じ解法が取れるでしょう.
672エトス:2012/01/25(水) 22:48:42.04
失礼.Gal(p^2)じゃなくてGF(p^2)でした...
673エトス:2012/01/25(水) 23:45:52.55
>>666
n|L_n を満たす奇数n>1が存在しないことを示してみます.
n|L_n を満たす奇数n>1が存在したと仮定します.
このときのnの最小の素因数をqとします.

X^2-X-1∈Q[X]の根のうち,正のものをαとし,もう片方をβとします.
(上のほうでφの記号を使いましたが,以降は代わりにα,βとします)

[準備]
任意に奇素数pを取ります.F_p=Z/pZとします.(同型を固定)
以下,主にA=R/(p)の中で考えているとします.(R=Z[α])

(@)√5がF_pに入っていないとき
このとき,F_p(√5)/F_pは2次のGalois拡大で,
(このガロア群はFrobenius射で単生成されています)
αに対してノルムを取れば,αα^p=αβ=-1
よって,α^(p+1) = -1 を得ました.

(A)√5がF_pに入っているとき
このとき,明らかに,α^(p-1)=1 です.
674エトス:2012/01/26(木) 00:08:48.50
以下,主に B=R/(q)の中で考えているとします.(R=Z[α])
αのBでの位数をo(q)と書くことにします.

q|L_nより,α^n+β^n=0
ここから,α^(2n)=(-1)^(n+1)=1

√5がF_qに入っていないとすると,
s=o(q)/2とおくとき,s|q+1 かつ s|n
(>>386 を参照にするとよいです)
よって,qの最小性から,s=1がいえます.
s=1のとき,α+1=0 ですから,
α^2-α-1=0 とあわせて,1=0 となり矛盾です.

√5がF_qに入っているとすると,
o(q)|q-1 かつ o(q)|2n なので,
qの最小性より,o(q)=1,2 となります.
o(q)=1ならば,α=1ですから,α^2-α-1=0とあわせて,
-1=0となり矛盾です,
また,o(q)=2とすれば,α^2=1 ですから,
0=α^2-α-1=-α となるので,α=0 となります.
これは明らかに矛盾です.

以上より,n|L_nを満たす奇数n>1が存在しないといえました.
675エトス:2012/01/26(木) 00:23:33.00
n^2|L_n を満たす偶数n>1が存在しないことはかなり簡単です.
そういうnが存在したとすると,4|L_n がいえますが,
一般に4|L_nを満たす整数nは奇数に限ります.矛盾です.

2つの結果をあわせて, >>666 の問題は解けたとおもいます.
676エトス:2012/01/26(木) 00:41:20.49
おまけで,n|L_n を満たす整数n>1はどんなものがあるでしょうか.
一般に,3^k|L_n ならば 3^(k+1)|L_(3n) がいえますから,
これを用いれば,6|L_6=18 から無限に構成できるでしょう.
18|L_18=5778=2*3^3*107, 54|L_54=2*3^4*m (m:ある正整数)
677132人目の素数さん:2012/01/26(木) 00:52:53.69
ryou
678エトス:2012/01/26(木) 00:54:41.59
[問題]
n(n+1)(n+2)(n+3) が2つの平方数の和で
表現できるような整数nは無限に存在するか
679132人目の素数さん:2012/01/26(木) 01:21:55.33
ねーよアホwwwwwwwwwwwwwwwwwwww
680132人目の素数さん:2012/01/26(木) 02:06:15.21
3*4*5*6 = 18^2+6^2
681132人目の素数さん:2012/01/26(木) 11:08:24.48
フェルマーの二平方数和
682132人目の素数さん:2012/01/26(木) 18:12:43.41
>>678
m^2-3k^2=1を満たす正整数m,kの組はいくらでも存在します^^;
そのときに n=m^2-1 とおけば 与式=(m^2-1)m^2(m^2+1)(m^2+2)
ここで肝心なのは m^2-1=3k^2、m^2+2=3(k^2+1) となっていること^^;
結局、与式=9k^2(k^2+1)m^2(m^2+1) となりますね^^;
素分解したときのどの4s-1の形の素因数の指数も偶数だから終わりですね^^;
683132人目の素数さん:2012/01/26(木) 19:30:34.89
リスト更新しました^^
いくつかの問題は排除しました^^

@ フィボナッチ数列類似の多項式列{F(n)}を次のように定義する:
F(0) = 0, F(1) = 1, F(n+2) = F(n+1) + x^n F(n).
このとき、任意の自然数nに対して、
F(5^n) は 1 + x + x^2 + … + x^(5^n - 1) で割り切れることを示せ。

A自然数x_1, x_2, ..., x_nが
1/x_1+1/x_2+...+1/x_n=1 を満たすとき
x_1+x_2+...+x_n の最大値を求めよ。

B4桁の自然数Mをp進法において反転させたものをNとする。
M=2N が成立するような正整数p,M(p>1)の組を全て求めよ。

C各整数n≧6に対して、τ(n!)|n! が成立することを示せ。

D|2^x-3^y|=p を満たす整数x,yの組が存在するような
p≡-7(mod 24)なる素数pはp=17以外に存在するか。
684132人目の素数さん:2012/01/26(木) 19:31:51.98
Ex^4+y^4=z^2+1 を満たす整数x,y,z>1の組は無限に存在するか。

F 次の条件を満たすようなf:N→Z は存在するか
(ここで,Nは正整数全体の集合としておきます)

[条件]
|Im(f)|=18 であり,任意の正整数nに対して,
f(n+6)=Σ[i=0,5]a_i*f(n+i)を満たすような
整数定数a_0,a_1,..,a_5 の組が取れる。

GF_p(p>2)上の方程式:ax^4+by^4=cz^2(abc≠0)は非自明な解を持つ

Hf_i(x) (i=1,2,..,n)を複素係数多項式、
α_i (i=1,2,..,n)を複素数とする。
d=Σ[i=1,n]degf_i(x)とおく。
更に各自然数mに対し複素数F(m)を
F(m)=Σ[i=1,n]f_i(m)*(α_i)^m により定義する。
もしこの時、F(m)(m=1,2,.., d+1) が全て(有理)整数なら、
任意のmに対してF(m)は(有理)整数。

I次の素数の連立方程式を解け(ゴミ問題追加 mew!)
ps=q+1
sq+1=r
685132人目の素数さん:2012/01/26(木) 20:38:22.03
誰か@Aはやく解けよカス共
686132人目の素数さん:2012/01/26(木) 20:41:01.48
いやだよ
687132人目の素数さん:2012/01/26(木) 21:40:00.16
F(m)=2(1/2)^m.
F(1)=1.
F(2)=1/2.
688132人目の素数さん:2012/01/26(木) 23:04:00.60
非自明って何。
689132人目の素数さん:2012/01/26(木) 23:08:46.74
>>688
x=y=z=0 が自明な解だと思われ
それ以外が非自明だと思われ

有理整数のmodの言葉で書くと
ax^4+by^4≡cz^2 (mod p)を満たす
整数x,y,z(ただし,x,y,z全てがmod pで0ではない)
の組が存在する
690132人目の素数さん:2012/01/26(木) 23:23:23.60
>>669
同様の議論で解ける?
691132人目の素数さん:2012/01/26(木) 23:28:29.56
>>690
ああできるぞ
思考停止でOK
692132人目の素数さん:2012/01/27(金) 02:07:12.22
>>690
こういう風になる(ほとんど同様だね)。

n^2|2^n-1をみたすnはn=1のみであること。

明らかにnは奇数
n>1の最小の素因数pをとる。
2^n -1≡0 (mod p)がいえる。
2^(p-1)≡1 (mod p)と2^n≡1 (mod p)、gcd(p-1,n)=1より2≡1 (mod p)となり矛盾する。

また、a>3のとき、a-1の素因数pを取れば
pがn^2|a^n -1をみたすことは容易に示せる。
a^p -1がp以外の奇素数の素因数qを持つことが言える。
(Zsigmondyの定理を使うと自明だけど、使わなくても出来る)
したがって、pqもn^2|a^n -1をみたすことがいえる。
上記のような議論を繰り返せば、n^2|a^n -1をみたす整数が無限にあることが言える。

a=3のときは4^2|3^4 -1=16・5であることから、(4・5)^2|3^20 -1がいえる。
Zsigmondyの定理より、3^20 -1には3^4 -1はない素因数p'が存在する。
よって、(20・p')^2| 3^(4・p') -1がいえる。
上記のような議論を繰り返せば、n^2|3^n -1をみたす整数が無限にあることが言える。
693132人目の素数さん:2012/01/27(金) 02:42:32.01
Zsigmondy使わないなら円分多項式の結果かそれに対応するものを使うだけ
そんなこと書いたら解答が長くなる(=醜くなる)のでZsigmondyを使うのみ
もっというとlucas sequence(いわゆるlucas数じゃなくて一般のlucas数列)
にはprimitive divisorが小数の例外を除いて必ず存在することが知られている。
なので、もっと上から目線でいうと、
もはやZsigmondyの定理といちいち断る意味すら皆無となってしまう。
ちなみに小数の例外が具体的になにかも完全に知られている。
(Zsigmondyの定理とかいちいちいうとなんかかっこ悪いし
きもいから、かっこよくprimitive divisorが存在するときめておこう)

Zsigmondyの定理? きんもー☆
なにその定理? primitive divisorが取れるでよくね?
なにそのマイナー定理? きもいんだけど。
694132人目の素数さん:2012/01/27(金) 02:49:27.16
確かにブロンズソードとレーザー兵器の差ぐらいあるが
同様に証明の難易度にも差がありすぎるだろw
ま、Zsigmondyの響きがよくないのは同意だがw
つかどうでもいいぞまじで!w
695132人目の素数さん:2012/01/27(金) 03:00:00.24
a,b,b−a,−a,−b,a−b,a,b。
696132人目の素数さん:2012/01/27(金) 03:28:43.49
国というものがよくわからない
697132人目の素数さん:2012/01/27(金) 05:10:39.96
x^3+y^3≡3 (mod n)を満たす整数(x,y)の
組が存在するような整数n>1を全て求めよ
698エトス:2012/01/27(金) 20:09:42.79
>>697
答えは gcd(n,63)|3 を満たす全ての整数n>1です.

(pf)
gcd(n,63)|3 が必要であることはすぐ確認できます.
ここでは十分であることを示したいとおもいます.

素数p≠3に対して,x^3+y^3≡3 (mod p)を満たす
整数x,yの組が存在するならば,(このこと自体はあとで示す)
各自然数kに対して,x^3+y^3≡3 (mod p^k) を満たす
整数x,yの組が存在することが次のようにして確認できる.

実際,x^3+y^3≡3 (mod p)が成立しているならば,
xとyのうち,少なくとも片方がmod pで0ではないので,
0でないものをy=aとし,もう片方をx=bとする.

f(X)=X^3+b^3-3∈F_p[X] とおくと,
その微分は f'(X)=3X^2 となるので,
とくにX=aはfのF_pの中の単根であるといえる.
よって,Henselの補題より,fはZ/p^kZの中でも根をもつ.
699エトス:2012/01/27(金) 20:11:19.38
次に,任意の素数p≠7に対して,x^3+y^3≡3(mod p)
を満たす整数x,yの組が存在することを証明する.

p=2,3,5のとき正しいことはすぐ確認できるのでp≧11とする.

F_p上の楕円曲線E:y^2=x^3-3888 を考える.
Eの判別式がゼロでないことはすぐに確認できる.
有限体上の楕円曲線に関するHasseの定理より,
とくに #E(F_p)≧p+1-2√p>3 (ここで p≧11を用いた)
これより,#E(F_p)≧4 がいえるので,
y^2=x^3-3888 を満たすx,y∈F_p(x≠0)の組の存在がいえる
このとき, s=(108+y)/(6x), t=(108-y)/(6x) とおけば,
s^3+t^3=3 in F_p の成立が確認できる.

以上の結果を中国剰余定理でまとめればよい.          ■
700132人目の素数さん:2012/01/28(土) 02:34:22.84
(n^2+5n+5)(5n^2+5n+1)
が2つの有理数の5乗の和となる自然数nは無限に存在するか
701132人目の素数さん:2012/01/28(土) 02:37:10.22
>>700
まじでいってんのか?
702132人目の素数さん:2012/01/28(土) 02:38:39.64
>>701
それはどういう意味か教えてくれないか?
703132人目の素数さん:2012/01/28(土) 02:48:31.37
たぶん答え用意せずにデタラメに生み出した問題かということだけど
それは違うよきちんと答えを用意してきてるよただし簡単かは知らない
704132人目の素数さん:2012/01/28(土) 10:58:36.12
x^3 + y^3 + z^3 = 1 の整数解は無限に多くあることを示せ。
答えは用意してある。
705132人目の素数さん:2012/01/28(土) 12:39:51.52
>704
x=3a^2+5ab-5b^2, y=4a^2-4ab+6b^2, z=5a^2-5ab-3b^2, w=6a^2-4ab+4b^2
706132人目の素数さん:2012/01/28(土) 18:00:00.26
>>655
a+1=0のとき。
nが1より大きい奇数のとき(n^2)|(a^n+1)。

a+1=±2^k・m(kは非負整数,mは1より大きい奇数)のとき。
a^m+1=(±2^k・m−1)^m+1≡0(mod.m^2)。

a+1=±2^k(kは非負整数)のとき。
nが偶数のときa^n+1=(a^(n/2))^2+1は4の倍数にならないから
(n^2)|(a^n+1)を満たさない。
nが奇数のときnの最小の素因数をpとする。
a^i≡1(mod.p)となる正の整数iの最小値をhとすると
h|(p−1),h|(2n)で,h|nでないからh=2。
p|(a−1)でなくp|(a^2−1)なのでp|(a+1)。
pは奇素数でa+1=±2^kなので条件を満たすnはない。

(n^2)|(a^n+1)となる1より大きい整数nが存在しない整数aは
a=±2^k−1(kは非負整数)。
707132人目の素数さん:2012/01/28(土) 18:00:11.43
>>683のD
答えは「存在しない」

x,y>0としても問題ないので、以後そうする。
2^x-3^y≡-7 (mod 24)となるのは、xが3以上の偶数かつyが偶数であることがすぐにわかる。
x=2u、y=2vとおくと2^x -3^y=(2^u +3^v)(2^u -3^v)

2^u -3^v=±1 をみたすu,vを求める。
2^u -3^v=1の解は(mod 8)で考えるとu=1、v=1以外にないことがわかる。
このとき|2^x -3^y|=5だから、問題の条件をみたさない。

2^u =3^v -1は(mod 8)で考えると、vが偶数であることがわかる。
3^(v/2) -1と 3^(v/2) +1が2のベキ乗になるのは3^(v/2) -1=2、 3^(v/2) +1=4
に限られるから、v=2、u=3と定まる。
このとき|2^x -3^y|=17だから、問題の条件をみたす。

u,vが上記の値以外のときは、|2^x -3^y|は合成数となるから
問題の条件をみたす素数pはp=17に限られる。
708132人目の素数さん:2012/01/28(土) 18:18:12.27
>>705
答えになって居ない
709132人目の素数さん:2012/01/28(土) 18:22:25.99
wを分母として、

x=(3a^2+5ab-5b^2)/(6a^2-4ab+4b^2)
y=(4a^2-4ab+6b^2)/(6a^2-4ab+4b^2)
z=(5a^2-5ab-3b^2)/(6a^2-4ab+4b^2)

ということかと
710132人目の素数さん:2012/01/28(土) 18:31:40.00
x^3+y^3+z^3=1 みたいな答えがどこにでもおいてある問題はどうでもいい
ウリジナルは要らない。オリジナルが必要だ。
711132人目の素数さん:2012/01/28(土) 18:36:49.04
>>707
上から3行目
○ |2^x-3^y|≡7 となるのは
× 2^x-3^y≡-7 となるのは

>>706
既に同様の解答がほぼ直前にあるぞよ
異なるアプローチなら意味がよりあったかもな
712132人目の素数さん:2012/01/28(土) 18:41:01.41
|2^x-3^y|≡-7 (mod 24) だった
x≧3ならば mod 8でみれば 絶対は負で外れることがわかり 2|y がでる
つぎに mod 3でみれば 2|x がでる
713132人目の素数さん:2012/01/28(土) 18:50:29.96
まるで白痴だな…
x^3+(-x)^3+1^3=1 でイイジャナイ

ちなみに x^3+y^3+z^3=3 を満たすような
整数x,y,zが無限に存在するかどうかは未解決問題
714132人目の素数さん:2012/01/28(土) 19:04:42.64
ゴミ問題をやってみた

ps=q+1
sq+1=r
(p,q,r,sは素数)

4=2*2≦ps=q+1 ⇒ q≧3

5=2*2+1≦sq+1=r だから
rは奇素数なのでsかqどちらかは2になる
でもqは奇素数だったので s=2 となる

2p=q+1 (イ)
2q+1=r (ロ)

q≠3とすると(ロ)よりr>3からq≡-1(mod 3)がいえる
ここで (イ)をつかえば p=3 がいえる
このとき q=5,r=11

q=3のとき p=2,r=7

以上より (p,q,r,s)=(2,3,7,2),(3,5,11,2)
715エトス:2012/01/28(土) 19:56:58.98
>>641
正解です.
>>682
正解です.

[問題]
600万より大きい任意の整数は2つの互いに素な合成数の和で表現できる
(600万をなるべく小さな数に置き換えてくれたらポイント可算です!)
716132人目の素数さん:2012/01/28(土) 20:12:01.56
>>711
>>706より前に正解がないが
717704:2012/01/28(土) 20:22:09.29
訂正

x^3 + y^3 + z^3 = 1 の自明でない整数解 (x, y, z のどれも 1 でないもの) が無限に存在することを示せ。
718132人目の素数さん:2012/01/28(土) 20:33:49.03
>>716
657-661,692 を見なされ
ちなみに君の解答は方は間違っている
方針は正しいから気にしなくていいと思うが

>>717
(1+9m^3)^3+(9m^4)^3+(-9m^4-3m)^3 = 1

下記をほとんどそのまま劣化引用しただけ
http://www.mathpages.com/home/kmath071.htm
719132人目の素数さん:2012/01/28(土) 20:35:03.27
自力でこのパラメータをみつけてオリジナル問題として投稿したのか
ただのパクリなのかはわからないがおそらくただのウリジナルだろう
720132人目の素数さん:2012/01/28(土) 20:37:46.81
よく考えたら716の解答のほうが正しいな
すまない正しいのは君のほうだった
721132人目の素数さん:2012/01/28(土) 20:38:55.46
もうディオファントス方程式はうんざりだよ!
722132人目の素数さん:2012/01/28(土) 20:39:47.74
>>718
(n^2)|((-3)^n+1)となる1より大きい整数nを一つ求めよ。
723132人目の素数さん:2012/01/28(土) 20:45:30.71
どこで間違いがおこったかというと
n^2|a^n+1 と n^2|a^n-1 (a>1)の問題に分解したとき
後者にはnが奇数という制約がつくというところだな
692はだからその制約がない場合を解いたということか
その場合はa≧3である限り無限に解があるということ
724132人目の素数さん:2012/01/28(土) 20:46:24.03
>>722
>>720
このどこにでもある問題はこれで終わりにしよう君ので正しい
725132人目の素数さん:2012/01/28(土) 20:48:39.74
ディオファントス方程式でもパクリでないかつある程度整った数式なら歓迎だ
しかしパクリは最悪だ。とくに有名なものは論外。死んだほうが良い。
726132人目の素数さん:2012/01/28(土) 20:49:08.71
>>724
終わりたきゃ一人で終われ
727132人目の素数さん:2012/01/28(土) 20:52:20.12
>>726
君も巻きぞいさ♪
728132人目の素数さん:2012/01/28(土) 20:52:55.94
巻きぞい?
729132人目の素数さん:2012/01/28(土) 20:53:51.07
巻き添え
おまえら全員巻き添えだ
730132人目の素数さん:2012/01/28(土) 20:54:59.65
>>725
じゃあお前の問題はどこにあるんだ?
731132人目の素数さん:2012/01/28(土) 20:56:35.93
>>724
どこにでもあるとは思わないけど...
少なくとも n^2|2^n+1 そのままよりは
732132人目の素数さん:2012/01/28(土) 21:00:08.64
>>730
無い
それがどうした?

>>731
冷静に考えたらそうだな
しかしx^3+y^3+z^3=1は論外だとおもう
まあ別の人だと思うしそうであってもなくても
匿名掲示板においてはそれは本質ではないがw
733132人目の素数さん:2012/01/28(土) 21:17:44.65
綺麗な素数の連立方程式を自作せよ
734132人目の素数さん:2012/01/28(土) 21:36:30.55
つか書き込みえらい多いな
地震があってからめっちゃ過疎ってたのに
735132人目の素数さん:2012/01/29(日) 00:30:41.89
地震があってからって何十年前の話だ?
736132人目の素数さん:2012/01/29(日) 01:33:17.71
>>718>>706の何行目が間違ってると思って間違っていると言ったのだろう?
737132人目の素数さん:2012/01/29(日) 01:46:52.42
位数の半分を使えば解答をわずかに短くできるぞ
738132人目の素数さん:2012/01/29(日) 01:52:59.28
2つほどつくってみたぞ

・x^6+y^6=1010 を満たす有理数x,yの組は存在するか?

・σ(n+1)<σ(n+2)<σ(n+3) を満たす自然数nは無限に存在するか?
(σ(n): nの正の約数の総和)
σが別の関数になった有名な問題が既にあるがこの問題のほうが難しい
739132人目の素数さん:2012/01/29(日) 01:58:52.35
1010の問題は簡単すぎだから見なかったことにしてくれw
740132人目の素数さん:2012/01/29(日) 03:00:36.58
両方とも簡単すぎるとおもうがなw
741132人目の素数さん:2012/01/29(日) 10:00:00.16
67.
742エトス:2012/01/29(日) 14:13:13.66
>>738
(回答)
n+2=a^pとして,素数pと整数aをうまく調整すればよいです.

a^p+1の素因数でa+1を割り切らないものは2p+1以上ですから,
lim[p→∞]f(a^p+1)=f(a+1) であることが容易に示せます.
ここで,f:Z+→Rはf(x)=σ(x)/x により定める乗法的関数です,
一方,lim[p→∞]f(a^p)=Π[q|a]q/(q-1) (qはaの素因数全体を動く)

たとえば, f(14)<Π[q|15]q/(q-1)<f(16) が確認できますので,
十分大きい素数pに対して,f(15^p-1)<f(15^p)<f(15^p+1)     ■
743エトス:2012/01/29(日) 14:24:01.42
最後の1行が抜けていました;;

[追加]
したがって,十分大きい素数pに対して,
σ(15^p-1)<σ(15^p)<σ(15^p+1) が成立します.

(問題)
f,g∈Z[X]に対して,f(X)|f(h(X)),g(X)|g(h(X))
を満たすdeg(h)>1かつ既約なh∈Z[X]が取れる.
744エトス:2012/01/29(日) 15:14:36.65
742の回答の上から2,3行目は次のように修正します(符号が漏れていました)

a^p±1の素因数でa±1を割り切らないものは2p+1以上ですから,
lim[p→∞]f(a^p±1)=f(a±1) であることが容易に示せます.(複号同順)
745132人目の素数さん:2012/01/29(日) 18:00:00.16
655=706。
746132人目の素数さん:2012/01/29(日) 19:00:00.29
f(X)=X。
747エトス:2012/01/29(日) 19:08:00.75
>>746
deg(f),deg(g)>1としておきます(もれていました;)
748132人目の素数さん:2012/01/29(日) 20:00:00.26
f(X)=X^2。
749エトス:2012/01/29(日) 20:28:40.94
>>748
たしかに! どうもです;

(問題)
f,g∈Z[X]に対して,f(X)|f(h(X)),g(X)|g(h(X))
を満たすdeg(h)>1かつ既約なh∈Z[X]が取れる.
ただし,f,gはともにxで割り切れないとする.

今度は大丈夫だとおもいます...
750132人目の素数さん:2012/01/30(月) 00:34:21.39
x^2+y^4の形の素数は無数にあることを示せ
751132人目の素数さん:2012/01/30(月) 01:05:09.34
                         ______
 >>1                      |  ,.へ、__,.ヘ/
                        | / \  ∠ヽ
                         |i^|「::::::ノ=l:::::ィ   / ̄ ̄ ̄ ̄
   ,. -‐- 、               |ヽ|   r_ \l   | 静粛に……!
  _/       \     ____/| ∧. (二二7!   < この男は今 このスレで
∠   ハヾミニ.r-、\∠L:r‐-‐-、:::::::::|/ ヽ_‐__.」`ー- | 初等的な解法が知られてない難問を挙げた
. /ィ ,L V∠ \l \\.)j j j j`二i\    /:|:::::::::::: | 最初に言ったはずだ
  W、ゞi ,、~ __ 「 ̄∧ ヾ´´´   |.  \ / |:::::::::::: | そういう行為は一切認めていないと・・・・・・!
    ,ゝし'/ ,ノ.|  / i  l.     l    \、.|::::::::::::   繰り返す! スレを荒らす奴は
   l 、`ヾニンl\./\|l、_」     ヽ、 /  ヾ::::::::::::   無条件で別室行きだっ・・・・!
.    | l    | _l\ト、  | \r──‐┐ト/ / r‐┴-、:::
.   |. |    7 l  ヽ | /☆☆☆.| | ∨ {ニニヾヽ
752132人目の素数さん:2012/01/30(月) 01:07:57.27
すまん1じゃなくて>>750だwしかもズレてるしw

In 1997, Iwaniec and John Friedlander proved that
there are infinitely many prime numbers of the form a2 + b4.[2]
Results of this strength had previously been seen as completely
out of reach: sieve theory—used by Iwaniec and Friedlander
in combination with other techniques—cannot usually distinguish
between primes and products of two primes, say.

以上はhttp://en.wikipedia.org/wiki/Henryk_Iwaniecから引用したw
753名無しさん:2012/01/30(月) 18:20:31.53 ID:5nj60JPR
          __ノ)-'´ ̄ ̄`ー- 、_
        , '´  _. -‐'''"二ニニ=-`ヽ、
      /   /:::::; -‐''"        `ーノ
     /   /:::::/           \
     /    /::::::/          | | |  |
     |   |:::::/ /     |  | | | |  |
      |   |::/ / / |  | ||  | | ,ハ .| ,ハ|
      |   |/ / / /| ,ハノ| /|ノレ,ニ|ル' 
     |   |  | / / レ',二、レ′ ,ィイ|゙/   私は只の数ヲタなんかとは付き合わないわ。
.     |   \ ∠イ  ,イイ|    ,`-' |      頭が良くて数学が出来てかっこいい人。それが必要条件よ。
     |     l^,人|  ` `-'     ゝ  |        さらに Ann.of Math に論文書けば十分条件にもなるわよ。
      |      ` -'\       ー'  人          一番嫌いなのは論文数を増やすためにくだらない論文を書いて
    |        /(l     __/  ヽ、           良い論文の出版を遅らせるお馬鹿な人。
     |       (:::::`‐-、__  |::::`、     ヒニニヽ、         あなたの論文が Ann of Math に accept される確率は?
    |      / `‐-、::::::::::`‐-、::::\   /,ニニ、\            それとも最近は Inv. Math. の方が上かしら?
   |      |::::::::::::::::::|` -、:::::::,ヘ ̄|'、  ヒニ二、 \
.   |      /::::::::::::::::::|::::::::\/:::O`、::\   | '、   \
   |      /:::::::::::::::::::/:::::::::::::::::::::::::::::'、::::\ノ  ヽ、  |
  |      |:::::/:::::::::/:::::::::::::::::::::::::::::::::::'、',::::'、  /:\__/‐、
  |      |/:::::::::::/::::::::::::::::::::::::::::::::::O::| '、::| く::::::::::::: ̄|
   |     /_..-'´ ̄`ー-、:::::::::::::::::::::::::::::::::::|/:/`‐'::\;;;;;;;_|
   |    |/::::::::::::::::::::::\:::::::::::::::::::::::::::::|::/::::|::::/:::::::::::/
    |   /:::::::::::::::::::::::::::::::::|:::::::::::::::::::::O::|::|::::::|:::::::::::::::/
754名無しさん:2012/01/30(月) 20:11:36.72 ID:???
くだらない論文?
それすらも生み出さずに公的資金をやりくりしているやつよりマシかも
755名無しさん:2012/01/31(火) 00:45:42.92 ID:???
フルレンジ、フルモーションで私をメタファライズして
756名無しさん:2012/01/31(火) 23:50:20.04 ID:???
>>707
正解です^^

>>714
正解です^^

>>738
前問は748にあるようにmod 67です^^;
757名無しさん:2012/02/01(水) 02:42:48.09 ID:???
bakerは神だったな
bakerの結果のapplyにしか過ぎないものが氾濫しすぎている
まずはbakerからだな
758132人目の素数さん:2012/02/06(月) 17:34:27.39
Π(X±√2±√3±√5) (符号の選び方は8通りで,8個の因子の積)
これがXを不定元とする整数係数多項式となることを証明せよ
759エトス:2012/02/06(月) 18:05:19.40
>>758
ガロア群で不変だからQ係数多項式となるのはほとんど明らか.
I∩Q=Z(I:ZのCにおける整閉包)からZ係数であることが従う.
760132人目の素数さん:2012/02/08(水) 11:10:48.03
係数が±1の多項式全体の集合をAとおき、
B={f(x)/g(x)|f(x),g(x)∈A} とするとき、
deg(p(x))≦3を満たすp(x)∈B∩Z[x]を全て求めよ。
761132人目の素数さん:2012/02/08(水) 11:20:00.47
x^3+x in A?
762132人目の素数さん:2012/02/08(水) 11:24:20.44
>>761
はい
763132人目の素数さん:2012/02/08(水) 11:28:52.94
あ、よくみたらx^2の係数と定数項がゼロですね。
じゃあ x^3+xはAに入っていないが正しいです。
764132人目の素数さん:2012/02/08(水) 11:39:39.69
これだとしょっぱいので問題を少し変えます。

係数が±1の多項式全体の集合をAとする。
f(x)g(x)∈Aを満たすg(x)∈Z[x]が存在するような
deg(f(x))=3なるf(x)∈Z[x]を全て求めよ。
765132人目の素数さん:2012/02/08(水) 11:40:00.20
0x+1 in A?
766132人目の素数さん:2012/02/08(水) 11:45:36.35
次数をmとするとき、0次(定数部)からm次の係数が全て±1ということです。
767132人目の素数さん:2012/02/08(水) 12:24:44.98
なんとなく、だけど...面白いから、とか、お金の為じゃない気がする...
768132人目の素数さん:2012/02/08(水) 20:00:00.13
x^3-x^2-x-1=x^3-x^2-x-1.
x^3-x^2-x+1=(x+1)(x-1)^2.
x^3-x^2+x-1=(x-1)(x^2+1).
x^3-x^2+x+1=x^3-x^2+x+1.
x^3+x^2-x-1=(x+1)^2(x-1).
x^3+x^2-x+1=x^3+x^2-x+1.
x^3+x^2+x-1=x^3+x^2+x-1.
x^3+x^2+x+1=(x+1)(x^2+1).
x^3-2x^2+1=(x-1)(x^2-x-1).
x^3-2x^2+2x-1=(x-1)(x^2-x+1).
x^3-2x+1=(x-1)(x^2+x-1).
x^3-1=(x-1)(x^2+x+1).
x^3-2x-1=(x+1)(x^2-x-1).
x^3+1=(x+1)(x^2-x+1).
x^3+2x^2-1=(x+1)(x^2+x-1).
x^3+2x^2+2x+1=(x+1)(x^2+x+1).
x^3-x-1=x^3-x-1.
x^3-x+1=x^3-x+1.
x^3+x-1=x^3+x-1.
x^3+x+1=x^3+x+1.
x^3-x^2-1=x^3-x^2-1.
x^3-x^2+1=x^3-x^2+1.
x^3+x^2-1=x^3+x^2-1.
x^3+x^2+1=x^3+x^2+1.
x^3-x^2+2x-1=x^3-x^2+2x-1.
x^3+x^2+2x+1=x^3+x^2+2x+1.
x^3-2x^2+x-1=x^3-2x^2+x-1.
x^3+2x^2+x+1=x^3+2x^2+x+1.
x^3-3x^2+3x-1=(x-1)^3.
x^3+3x^2+3x+1=(x+1)^3.
769132人目の素数さん:2012/02/09(木) 00:53:02.44
>>715
Cを十分大きな正の整数とする。nを正の整数、pを素数とする。
1からnまでの素数の個数をπ(n)とすると
n≧Cをみたす正の整数nに対して、π(n)<1.10556・n/log nがいえる。
http://archive.numdam.org/ARCHIVE/SDPP/SDPP_1975-1976__17_2/SDPP_1975-1976__17_2_A9_0/SDPP_1975-1976__17_2_A9_0.pdf
に上記不等式の証明があります。

メルテンスの定理より
Π_[p≦n](1- 1/p)={e^(-γ)/log n}・e^{f(n)}

ここで、f(n)とg(n)が以下のような関数とする。
任意の整数nに対して、正の実数αが存在して、|g(n)|≦α
f(n)=g(n)/log n -∫_[n,∞] g(x)/{x(log x)^2}dx
M=1 -log(log2) +∫_[2,∞] g(x)/{x(log x)^2}dx
M=0.26149…、γ=0.57721…

このとき、1 -log(log2) -M≦α・∫_[2,∞] 1/{x(log x)^2}dx=α/log 2
だから、α≧(1 -log(log2) -M)・log 2=0.76593…をみたす任意の実数α
に対して、|g(n)|≦αが成り立つ。

以後、α=1 -log(log2) -Mとおくことにする。
このとき。|f(n)|≦α/log nがいえる。
770132人目の素数さん:2012/02/09(木) 00:53:52.74
>>769の続き

tを正の整数とする。
nと互いに素な整数の個数φ(n)をとる
nの素因数pをn^(1/t)より大きなものとn^(1/t)以下のものに分ける。
φ(n)=n・Π_[p|n] (1- 1/p)=n・Π_[p|n,p≦n^(1/t)] (1- 1/p)・Π_[p|n,n^(1/t)<p] (1- 1/p)
nのn^(1/t)より大きな素因数は高々t-1個だから
Π_[p|n,p≦n^(1/t)] (1- 1/p)・Π_[p|n,n^(1/t)<p] (1- 1/p)
=Π_[p|n,p≦n^(1/t)] (1- 1/p)・(1-1/{n^(1/t)})^(t-1)
≧Π_[p≦n^(1/t)] (1- 1/p)・(1-1/{n^(1/t)})^(t-1)
={e^(-γ)/log n^(1/t)}・e^{f(n^(1/t) )}・(1-1/{n^(1/t)})^(t-1)
={t・e^(-γ)・e^{f(n^(1/t) )}・(1-1/{n^(1/t)})^(t-1)}/log n

ここで、C=28^6、t=6とおくと
f(n)≦-α/log 28=-0.22985…だから
{t・e^(-γ)}・e^{f(n^(1/t) )}・(1-1/{n^(1/t)})^(t-1)
≧6・e^(-γ-α/log 28)・(1- 1/28)^5
=2.23188…だから

φ(n)≧2.2318・n/log nがいえる。

したがって、nと互いに素な合成数の個数は
少なくとも、2.23188・n/log n - 1.10556・n/log n=1.12632・n/log n>1.10556・n/log n
となるから、1やnと互いに素な素数の個数よりも多い。

よって、n≧28^6のとき、nは互いに素な合成数の和で必ず書けることが言えた。
771132人目の素数さん:2012/02/09(木) 01:11:41.64
>>769>>770の続き
28^6は481890304だから5億に近いので、>>715の題意をみたさない。
しかし、nは有限の値だから、以下のような議論が出来る。

しかし議論は長すぎるので、大まかな方針にとどめる。

まず、3・5・7・11・13・17・19・23・29=3234846615>481890304=28^6だから
nは3,5,7,11,13,17,19,23,29のいずれかで割り切れないことがいえる。

nを割り切らない最小の素数pをとる。
pで割った時の余りがrの素数からなる有限集合をP(p,k)とする。
(pより小さな素数の積)・Π_[q∈P(p,k)]q>28^6ならば、
P(p,k)の元のどれかはnを割り切らないことがいえる。
この性質をフル活用する。

上記の事実を用いて、nの素因数ではない素数q(1),q(2)をn≡q(1)q(2) (mod p)かつ、
q(1)q(2)が出来るだけ小さくするようにとる(q(1)とq(2)は異なる必要はない).。

p=2、3、5、7、11、13、17、19、23、29に関して上記作業を行うと
結局、n>30030をみたす整数nについて、nは互いに素な合成数の和で書けることが言える。

同様な議論を繰り返すと、結局n>900をみたす整数nについて、nは互いに素な合成数の和で書けることが言える。
772132人目の素数さん:2012/02/09(木) 02:46:28.82
>>771
訂正
×「nを割り切らない最小の素数pをとる。
pで割った時の余りがrの素数からなる有限集合をP(p,k)とする。
(pより小さな素数の積)・Π_[q∈P(p,k)]q>28^6ならば、
P(p,k)の元のどれかはnを割り切らないことがいえる。
この性質をフル活用する。 」

○「nを割り切らない最小の素数pをとる。
pで割った時の余りがrの素数からなる有限集合をP(p,r)とする。
(pより小さな素数の積)・Π_[q∈P(p,r)]q>28^6ならば、
P(p,r)の元のどれかはnを割り切らないことがいえる。
この性質をフル活用する。 」
773132人目の素数さん:2012/02/09(木) 05:07:07.44
900=7^2+23*37

900より小さくできるが下限はいくつだろうか
774132人目の素数さん:2012/02/09(木) 08:00:00.51
210.
775132人目の素数さん:2012/02/09(木) 21:12:54.33
>>769の訂正
×「以後、α=1 -log(log2) -Mとおくことにする。」

○「以後、α=(1 -log(log2) -M)・log 2とおくことにする。」
776132人目の素数さん:2012/02/11(土) 12:24:44.68
適当な整数 a, b によって
p = a^2 + 5ab + 7 b^2
と書ける素数 p は無限に存在する事を示せ
777132人目の素数さん:2012/02/11(土) 14:57:56.18
>>776
http://ja.wikipedia.org/wiki/%E4%BA%8C%E5%80%8B%E3%81%AE%E5%B9%B3%E6%96%B9%E6%95%B0%E3%81%AE%E5%92%8C#.E7.B4.A0.E6.95.B0.E3.81.AB.E3.81.A4.E3.81.84.E3.81.A6.E3.81.AE.E8.A8.BC.E6.98.8E
の議論で、r^2≡-1 (mod p)をr^2≡-3 (mod p)に変えると
pが奇素数でかつ(-3/.p)=1のとき、pはx^2 +3y^2=zp、1≦z≦3と書けることが分かる。
mod 4で考えると、z=2の場合はありえないことがすぐに言える。
z=3のときは、p=y^2 +3(x/3)^2がいえるから、
p≡1 (mod 6)のとき、p=x^2 +3y^2と書けることが言える。
あとはx=a +5・b/2 、y=b/2 をみたす整数a.bをとれば
p=a^2 +5ab+7b^2とかけるから、題意は言えた。
778132人目の素数さん:2012/02/11(土) 16:04:11.72
>>777
それはthueの方法だわさ

[別解(略)]
f(x,y)=x^2+5xy+7y^2 とおく。
これは正定値2次形式で、判別式は-3である。
一方、2次体Q(√-3)の類数は1であるから、結局、
x^2≡-3 (mod 4p) が解ける条件を求めれば十分である。

p=2のとき、明らかに解なしである。
p=3のとき、明らかに解ける。
pが3より大きい素数のとき、
p≡1(mod 6)が解けるための必要十分条件である。

以上より、条件を満たす全ての素数pは、
p≡1(mod 6)なるもの、および、p=3の2種である。
779132人目の素数さん:2012/02/11(土) 16:18:41.28
適当な整数 a, b によって
p = 2a^2 + ab + 3b^2
と書ける素数 p は無限に存在する事を示せ

これを2次形式としてみたときの判別式は-23であり、
Q(√-23)の類数は3だから、少し面倒なことになる。
780779:2012/02/11(土) 16:33:39.46
やはり問題を取り下げる。
無限に存在することを初等数論の方法で示せるきがしない。
781132人目の素数さん:2012/02/11(土) 19:08:43.58
適当な整数 a, b によって
p=4a^2 +8ab +9b^2
と書ける素数 p は無限に存在する事を示せ
782779:2012/02/11(土) 19:59:11.95
>>781
Q(√-5)の類数は2だから,この場合はなんとかなるよ
たとえば >>777 のようにthueの方法でもいいとおもうよ

つまり、x^2+5y^2=kp (1≦k≦5)
とやって、kの候補を3つ(k=1,2,5)に絞る。
で、あとは(2次形式を)うまく変形すればいい

また同じことするの?ルーチンワークだよっという
783779:2012/02/11(土) 20:10:16.06
おおっと、kの候補は全部だったな。
じゃあthueの方法だけじゃうまくいかない。

まあそれでもやはり4x^2+8xy+9y^2っていうのは
x^2+5y^2 か 2x^2+2xy+3y^2 のどれかに帰着されて、
素数p(≠2)はこのどれかで必ず表現できる。
重要なのは2次の指標を用いて2つのケースを区別できる。
だからやはりそういう意味で初等的に解ける。
784779:2012/02/11(土) 20:26:38.78
と、だらだら無意味な言葉を並べたが全部撤回する
よく考えたら問題の2次形式の判別式は-80だから
x^2+5y^2 か 2x^2+2xy+3y^2 のどれかに帰着されるとは限らないね
785779:2012/02/11(土) 20:38:21.43
せっかくだから回答を書いた。
p=(2X+2Y)^2+5Y^2

4p=x^2+5y^2 を満たす整数x,yが取れるような素数pは無限に存在する。
このとき、xは偶数だから問題は解けたことになる。

ちなみに自分の過去の発言で
『素数p(≠2)はこのどれかで必ず表現できる』
とかあるが、んなことあるわけないので、無視で。
786779:2012/02/11(土) 20:42:03.69
もうだめだアルコールで脳がいっちょる
正常な人に任せた!
787132人目の素数さん:2012/02/11(土) 21:28:41.47
★4x^2+5y^2
☆4x^2+4xy+6y^2
☆3x^2±2xy+7y^2
☆2x^2+10y^2
☆x^2+20y^2
788132人目の素数さん:2012/02/11(土) 21:42:44.99
4x^2+5y^2とx^2+20y^2が有理整数のmoduloで区別できないから
>>781 の問題は初等的に解けない気がする
789781:2012/02/12(日) 02:45:49.66
だめだ、>>781は却下します

問題を変えます。
x^2 +5y^2=pと書けるための、pに関する必要十分条件を求めよ。
これだと、thueの方法では駄目なはず。
790781:2012/02/12(日) 02:46:40.49
あと、>>789のpは奇素数です。
791エトス:2012/02/12(日) 09:39:24.06
>>769
正解です☆

>>789
できなくはないですね...

(-5/p)=(-1/p)=1 を満たす素数p>5を取ります.
thueの方法(前の投稿を参照してね)より,
x^2+5y^2=kp を満たす整数x,y,k(1≦k≦5)の組が取れます.
k≠2,3であるならば, pは条件を満たしています.(要確認)
k=2,3であるとき,次のようにして矛盾が導かれます.

k=2,3であるとき, x^2+5y^2=kp から,とくに,
(±2p/5) = 1 が成立するとなりますが,(±2/5) = -1 ですから,
あわせて, (p/5) = -1 となります.しかしながら,
(p/5) = (5/p) = (-1/p)*(-5/p) = 1*1 = 1
792132人目の素数さん:2012/02/12(日) 12:06:15.42
>>764
>>768
証明は
793781:2012/02/12(日) 17:47:04.52
>>791
正解です。thueでもOKでしたね、勘違いしてました。

問題です。
適当な整数a,bを用いて、p=a^2 +3ab +6b^2と書けるための、奇素数pに関する必要十分条件を求めよ。
794132人目の素数さん:2012/02/13(月) 05:01:30.16
>>791
>>715 の用意してあった解答は?
795132人目の素数さん:2012/02/15(水) 19:13:33.00
>>793
p≠3,5の場合を考えれば十分である。

p=a^2 +3ab +6b^2は4p=(2a+3b)^2 +15b^2より、(-15/p)=(p/5)=(p/3)=1がいえる。

逆に、(-15/p)=(p/5)=(p/3)=1がいえるとき
http://aozoragakuen.sakura.ne.jp/suuron/node65.html
より、x^2≡-15 (mod p)の解をsとするとき、|pu+sv|<15^(1/4)・p^(1/2)、|v|<15^(-1/4)・p^(1/2)
をみたす原点と異なる格子点(u,v)の存在が言える。
このとき、(pu+sv)^2 +15v^2≡0 (mod p)かつ0<(pu+sv)^2 +15v^2<2√15・p<8pがいえるから
x^2 +15y^2=kp、1≦k≦7をみたす整数x,y,kの存在が言える。
x^2 +15y^2≡0,1,3 (mod 4)よりk=2,6はありえない、また(-15/7)=-1よりk=7もありえない。
k=3,5の場合は、(p/5)=(p/3)=1を用いると、>>791と同様にして矛盾が導ける。

したがって求める必要十分条件は、(-15/p)=(p/5)=(p/3)=1である。

問題
pをp≡3 (mod 8)をみたす素数とする。pが合同数になりえないことを証明せよ。
http://ja.wikipedia.org/wiki/%E5%90%88%E5%90%8C%E6%95%B0
796132人目の素数さん:2012/02/15(水) 19:20:45.81
>>795の補足
k=4のとき、x,yが奇数のとき、x^2 +15y^2≡0 (mod 8)となるから、x^2 +15y^2≡4 (mod 8)となるのはx,yが偶数の場合に限られる。
よってp=(x/2)^2 +15(y/2)^2となり、k=1の場合に帰着する。
>>795より、(-15/p)=(p/5)=(p/3)=1のとき、整数x,yを用いてp=x^2 +15y^2と書ける。
このときx=a +3・b/2,y=b/2とおくとp=a^2 +3ab +6b^2とかけることがすぐにわかる。
797786:2012/02/15(水) 20:20:46.35
GJ。(786さんがイイネと言っています)
皆知ってのとおり本質的には判別式の値が解の表現に関係している

たとえば 779の問題は既に有理数のmoduloの言葉のみで表現できない
その場合はたとえば3次方程式を用いて解を表現できる

x^2+3xy+6y^2の問題なら判別式は -15
それを判別式としてもつ(整数係数)正定値2次形式は
本質的には2つ存在する
(モ変形(笑,高木貞治の初等整数論の言葉から引用)
により互いにうつりあうのを同一視するという意味で"本質的")

その2つを具体的に求めるのは簡単で
問題はその2つを有理整数のmoduloで分類することができるかどうか

-15≡1 (mod 4) だから、2次体K=Q(√-15)の判別式は -15
ここで数体Kの類数は2だから、この場合はそれが可能であることが"みえる"
798786:2012/02/15(水) 20:46:11.25
>>795 の人の補足をすると
上のaosoraなんとかのURLの定理53において
h=15^(1/4)*√p, k=15^(-1/4)*√p
α=p, β=s, γ=0, ω=1, Δ=p
(当然、sはZの上にのっける)
799エトス:2012/02/15(水) 23:48:16.60
>>794
予め用意していたというほど大げさなものではありませんが一応...
(あとの2つの不等式の証明はかなり面倒なので省きます.
しかしながら初等的な解析(実解析のみという意味)だけで可能です)

正整数nが2つの互いに素な合成数の和で表現できないとすれば,
π(n)≧-1+φ(n)/2 の成立がいえます.
(∵nと互いに素なn以下の正整数はφ(n)個ありますが,
そのような数を任意に1つ取り,aとするとき,
n-aもまたそのような数になり,足し合わせるとnになります.
この操作により,φ(n)個の数からφ(n)/2個のペアが作れます.
ここで,1は合成数でないので,(1,n-1)はペアから除外します.)

n≧3.6*10^5 とするとき次の不等式の成立がいえます.
φ(n)>n/(1+e^γlog(log(n))) (γ=オイラーの定数)
π(n)<s*n/log(n) (s=1.0935)

この2つを最初の不等式に直接適用すれば矛盾が導かれます.
(π(n)≧-1+φ(n)/2の時点で,これが十分大きな整数nで成立しないのは
わかってしまうので,その程度の応用を意図していたのだと思ってもらえれば)
800エトス:2012/02/16(木) 00:28:10.00
>>795
合同数の問題の回答です.

次を示せば十分です.

「pをp≡3(mod 8)なる素数(定数)とするとき,
xy(x^2-y^2)=pz^2 を満たす正整数x,y,zの組は存在しない」

(証明)
そのような組が存在したとし,
とくにpz^2が最小となるような組を改めて(x,y,z)とする.
このとき,gcd(x,y)=1であることはすぐにわかる.
よって,x,y,x^2-y^2はどの2つをとっても互いに素である.
pは素数なので,x,y,x^2-y^2のどれかを割り切る.
以後,〜組が取れるといったとき,自動的に互いに素であり,
しかも正整数からなる組であるとする.

(1) p|y であるとき
x=A^2,y=pB^2,x^2-y^2=C^2を満たすようなA,B,Cの組が取れる.
ここから次の2つの場合が考えられる

1-(@)
A^2=D^2+E^2,pB^2=2DEを満たすD,Eの組が取れるとき
p(B/2)^2=FG(F^2-G^2)を満たすF,Gの組が取れるから
(x,y,z)の取り方に反しているので矛盾.

1-(A)
A^2=D^2+E^2,pB^2=D^2-E^2を満たすD,Eの組が取れるとき
mod pでみることで,2つ目の等式はD^2=E^2となるので,
1つ目に適用すれば,A^2=D^2+D^2=2D^2.
これから,(2/p)=1を得る.これは矛盾である.
801エトス:2012/02/16(木) 00:29:51.12
(2) p|x であるとき
x=pA^2,y=B^2,x^2-y^2=C^2を満たすようなA,B,Cの組が取れる.
pA^2=D^2+E^2を満たす互いに素なD,Eの組が取れるので,
これはp≡-1(mod 4)に矛盾している.


(3) p|x^2-y^2 であるとき
x=A^2,y=B^2,x^2-y^2=pC^2を満たすA,B,Cの組が取れる.
あわせて,(A^2+B^2)(A^2-B^2)=pC^2 を得る.
p≡-1(mod 4)より,pはA^2-B^2を割り切る.
d:=gcd(A^2+B^2,A^2-B^2)は2の約数である.
次の2つのケースにわけられる.

3-(@)
d=1 であるとき
A^2+B^2=D^2, A^2-B^2=pE^2 を満たすD,Eの組が取れる.
2つの等式を足し合わせて, 2A^2=D^2+pE^2
これから,(2/p)=1を得る.これは矛盾である

3-(A)
d=2 であるとき
A^2+B^2=2D^2,A^2-B^2=2pE^2を満たすD,Eの組が取れる.
A,Bのパリティは等しいので,A=F+G,B=F-Gを満たすF,Gの組が取れる.
これを用いれば,F^2+G^2=D^2, 2FG=pE^2 を得る,
よって, HI(H^2-I^2)=p(E/2)^2を満たすH,Iの組が取れる.
これは(x,y,z)の組の取り方に反している.

以上より全ての場合を尽くし,いずれの場合も矛盾が導かれた.
802エトス:2012/02/16(木) 00:41:04.64
[問題]
fをdeg(f)>1なる既約な整数係数多項式とする.
各正整数nに対して,p(n)はnの最大の素因数を表すとする,
このとき,lim[n→∞]p(f(n)) = +∞ を示せ.

(難しいとおもいます.しかしながら,
たとえば,Faltings'theoremは基本的です)

http://en.wikipedia.org/wiki/Faltings'_theorem
803132人目の素数さん:2012/02/16(木) 01:32:47.08
ファルティングは厨定理だけど
この問題の場合はSiegelで十分な気がするが?
804132人目の素数さん:2012/02/16(木) 01:47:54.72
>>801
正解です

>>802
f(n)の定数項は0ではないんですねw

lim[n→∞]p(f(n))<+∞と仮定する。
f(n)の素因数になりうるものが、p(1)、…、p(k)のみとなる。
十分大きな整数s,tをとると、f(s・{p(1)・...・p(k)}^t)>|{p(1)・...・p(k)}^t)|がいえる。
このとき、f(s・{p(1)・...・p(k)}^t)の素因数の候補は、p(1)、…、p(k)のみだから、
その中に、{p(i)}^tがf(s・{p(1)・...・p(k)}^t)を割り切るような、p(i)の存在が言える。
したがって、多項式f(n)の定数項cが{p(i)}^tで割り切れることが言えるが、
tを大きくとると、|c|<{p(i)}^tとなるから矛盾。
805132人目の素数さん:2012/02/16(木) 01:53:23.92
駄目だ、>>804はlim[n→∞]sup{p(f(n) )}=+∞の証明でしたね。
すみません、>>804は却下してください。
806L:2012/02/16(木) 17:22:08.09
>>802
fの最高次の係数は正であるとしておきます。
lim[n→∞]p(f(n)) = +∞ でないと仮定します。
このとき、ある定数k>0が存在していて、
p(f(n))<k を満たす正整数nが無限に存在するとなります。

kより小さい全ての素数をp1,p2,.,psとします。
p(f(n))<kかつf(n)>0 を満たす正整数nを任意に1つ取るとき、
f(n)=h*m^3 を満たす正整数h,mの組が取れます。
ただし、hのどの素因数の指数も2以下とします。
807L:2012/02/16(木) 17:23:58.70
ここで、任意に正整数tを固定し、
g_t(x,y):=f(x)-ty^3 として、
g_t(x,y)=0で定まるaffine代数曲線C_tを考えます。
このとき、Cは非特異で種数は1以上です。
ここではそのことを示すことにします。

(α,β)が特異点だと仮定すると、
f(α)=tβ^3
f'(α)=0
3tβ^2=0
の3つの等式が同時に成立するとなるので、
αはfの重根となり、fの既約性に矛盾します。
(重根となるところにdeg(f)>1も用いています)

Cは非特異なので、種数は公式により、(d-1)(d-2)/2 (d=deg(g_t))
これとd≧3より、Cの種数が1以上であることがいえました。

さて、仮定により、ある正整数hが存在して、
affine代数曲線C_h上に整点が無限に存在するとなりますが、
これはSiegelの整点に関する定理に矛盾します。

http://en.wikipedia.org/wiki/Siegel's_theorem_on_integral_points
808L:2012/02/16(木) 17:35:44.09

[問題]
p≡1(mod 20)を満たす素数pは
x^2+20y^2, x^2+100y^2 の両方の形で表現できるか、
どちらの形にもならないことを示せ。

Kaplansky's theorem が出典です。(比較的新しい?)
これは初等的に示すことができます。
この問題の面白いところは
1つ1つの形の条件がいわゆる合同式の言葉で
表現できないところにあるとおもいます。

http://en.wikipedia.org/wiki/Kaplansky's_theorem_on_quadratic_forms
809132人目の素数さん:2012/02/16(木) 19:10:00.37
x^5+y^5=3.
810132人目の素数さん:2012/02/16(木) 19:30:10.38
>>809
mod 11で解なし ⇒ とくに整数解なし
有理数解をこのスレできいているなら難問
なんでも使っていいならほとんど明らかかもしれない
811132人目の素数さん:2012/02/16(木) 20:41:34.79
>>804
limsup の方の証明とみるならば
lim[n→∞]p(f(n))<+∞ と仮定するじゃなくて
limsup[n→∞]p(f(n))<+∞ と仮定するのが正しいと思う

>>810
1の原始5乗根をQに添加させてできる数体の整数環はUFD(非自明)
だからそれを認めれば初等的といえる解答がつくれるかもしれない
812132人目の素数さん:2012/02/17(金) 04:58:48.71
>>811
読み替えればいいだけだろ、それくらい融通きかせろや。
813132人目の素数さん:2012/02/17(金) 05:26:55.61
>>812
lim[n→∞]p(f(n)) = +∞ の否定は
lim[n→∞]p(f(n))<+∞ とはならない
これだとp(f(n))が有限の値に収束しているようにみえる
なので、ここから矛盾がでてきたとしても意味がないきがする

融通が利かないかもしれない他の人達(自分含む)のために指摘している
せっかくのlimsupの場合の証明に関する投稿を無駄にしたくないので
814132人目の素数さん:2012/02/17(金) 05:46:34.74
(掲示板を使うのは)難しい
815794:2012/02/18(土) 15:20:04.85
>>799
ありがとう
わかりやすいです
816132人目の素数さん:2012/02/19(日) 03:55:54.81
          __ノ)-'´ ̄ ̄`ー- 、_
        , '´  _. -‐'''"二ニニ=-`ヽ、
      /   /:::::; -‐''"        `ーノ
     /   /:::::/           \
     /    /::::::/          | | |  |
     |   |:::::/ /     |  | | | |  |
      |   |::/ / / |  | ||  | | ,ハ .| ,ハ|
      |   |/ / / /| ,ハノ| /|ノレ,ニ|ル' 
     |   |  | / / レ',二、レ′ ,ィイ|゙/   私は只の数ヲタなんかとは付き合わないわ。
.     |   \ ∠イ  ,イイ|    ,`-' |      頭が良くて数学が出来てかっこいい人。それが必要条件よ。
     |     l^,人|  ` `-'     ゝ  |        さらに Ann.of Math に論文書けば十分条件にもなるわよ。
      |      ` -'\       ー'  人          一番嫌いなのは論文数を増やすためにくだらない論文を書いて
    |        /(l     __/  ヽ、           良い論文の出版を遅らせるお馬鹿な人。
     |       (:::::`‐-、__  |::::`、     ヒニニヽ、         あなたの論文が Ann of Math に accept される確率は?
    |      / `‐-、::::::::::`‐-、::::\   /,ニニ、\            それとも最近は Inv. Math. の方が上かしら?
   |      |::::::::::::::::::|` -、:::::::,ヘ ̄|'、  ヒニ二、 \
.   |      /::::::::::::::::::|::::::::\/:::O`、::\   | '、   \
   |      /:::::::::::::::::::/:::::::::::::::::::::::::::::'、::::\ノ  ヽ、  |
  |      |:::::/:::::::::/:::::::::::::::::::::::::::::::::::'、',::::'、  /:\__/‐、
  |      |/:::::::::::/::::::::::::::::::::::::::::::::::O::| '、::| く::::::::::::: ̄|
   |     /_..-'´ ̄`ー-、:::::::::::::::::::::::::::::::::::|/:/`‐'::\;;;;;;;_|
   |    |/::::::::::::::::::::::\:::::::::::::::::::::::::::::|::/::::|::::/:::::::::::/
    |   /:::::::::::::::::::::::::::::::::|:::::::::::::::::::::O::|::|::::::|:::::::::::::::/
817132人目の素数さん:2012/02/19(日) 04:08:50.18
質問ですマジレスお願いします。ガチです><
100未満を、四捨五入。100未満が、四捨五入。

12万6750 = 答え(      )

答えはいくつになりますか?
818132人目の素数さん:2012/02/19(日) 07:39:16.90
12万6800
12万6705ー>12万6710ではなく12万6700
819132人目の素数さん:2012/02/19(日) 18:00:00.10
文字は正の整数を表すものとする。
2x^4+1=y^2を満たすx,yのうちxが最小のものをとる。
y=2a+1。
x^4=2a(a+1)。
x=2b。
8b^4=a(a+1)。
a=8c^4。
a+1=d^4。
b=cd。
8c^4+1=d^4。
d^2=4e+1。
c^4=e(2e+1)。
e=f^4。
2e+1=g^4。
2f^4+1=g^4。
x>b≧d>2f^2>f。
820132人目の素数さん:2012/02/19(日) 18:11:46.86
>>819
8b^4=a(a+1)まではいいけど、
a=c^4。
a+1=8d^4。
の場合が抜け落ちている

この場合はすいすいとは進まないきがする
821132人目の素数さん:2012/02/19(日) 18:20:00.30
c^4+1≡1,2(mod.4)。
8d^4≡0(mod.4)。
822132人目の素数さん:2012/02/19(日) 18:26:38.23
>>821
おおうまくいったね
完遂おめでとう
823エトス:2012/02/19(日) 19:05:37.24
x^2-2y^4 = -1 だと有名な難問(?)になりますね.
これだと例えば,(x,y)=(239,13)という非自明な整数解があります.

しかしながら,Z[√-1]で分解して考えれば,
f(m,n)=整数定数 という不定方程式を考える問題に帰着されて,
ここで,fは4次の整数係数多項式で,斉次かつ既約ですから,
結局,整数解の存在範囲を上から計算可能な定数で抑えることができます.

http://mathworld.wolfram.com/ThueEquation.html
824132人目の素数さん:2012/02/19(日) 19:31:59.56

2つの「三角数の2乗」の和が平方数となることは無限に起こりうる

825132人目の素数さん:2012/02/19(日) 20:49:17.11
>>824
s^2 -2t^2=-1をみたす正の整数の組(s,t)をとると、{s(s-1)/2}^2 +{s(s+1)/2}^2=(st)^2となる。
826132人目の素数さん:2012/02/19(日) 21:00:00.24
(133・132/2)^2+(144・143/2)^2=(165・164/2)^2。
827132人目の素数さん:2012/02/19(日) 21:25:36.97
>>826
>>824の題意を勘違いしてないか?
「三角数の平方の和が平方数になるすべての場合を求めよ」という問題じゃないんだよね。
もし、>>826>>825の反例のつもりなら、筋違いもいいところ。
828132人目の素数さん:2012/02/19(日) 21:37:26.45
単にこんなカッコイイペアもあるぜ!ってことじゃないかい?
本人じゃないから知らないけどw

三角数の2乗の和がまた三角数の2乗になる問題だったら
ずっとずっと難しい問題だとおもうねえ
それだったら未解決だったという記憶がある
829132人目の素数さん:2012/02/19(日) 21:53:38.00
4次+4次=4次
解の有限性を出せる人がいても
具体的なバウンドを計算するのはずっと難しい
830132人目の素数さん:2012/02/19(日) 22:02:54.66
まあ同次に帰着できるかそうでないかで全然話が違う。
しかも3変数だから式の形が良くないとダメだわさ。
831132人目の素数さん:2012/02/22(水) 06:28:02.77
おちんちんきもちいお(;ω;)
832132人目の素数さん:2012/02/23(木) 13:04:15.22
世界には初等整数論・鬼マニアの人がいて
やっぱ2000程度までの整数の素因数分解とか
計算せんで九九なみにそらで頭に入ってるんでしょうね

でないと鼻が利きませんものね

ガウス少年は10000までカバーしてたとか
833132人目の素数さん:2012/02/23(木) 13:05:00.01
世界には初等整数論・鬼マニアの人がけっこうゴロゴロしてて
やっぱ2000程度までの整数の素因数分解とか
計算せんで九九なみにそらで頭に入ってるんでしょうね

でないと鼻が利きませんものね

ガウス少年は10000までカバーしてたとか
834132人目の素数さん:2012/02/23(木) 13:20:03.18
発達障害のいとこの子供がGNU coreutilsのfactorで一日中遊んでる
でもとくに数学的才能はないようだ
835132人目の素数さん:2012/02/23(木) 14:09:19.68
あるかもよ
分りやすく現れてないだけで
836132人目の素数さん:2012/02/24(金) 11:56:16.42
>ガウス少年は10000までカバーしてたとか

ブッブー

ガウスは少年時代、数年で300万までの素数を全てリストアップしてみせました
ガウスが300万まで素因数分解を計算してみせた、ということは
それ以後は瞬時に結果を取り出せる、ということを意味します

ガウスゲー

フェルマーの定理を問われて
「そのての法則性は数多ありすぎて(証明はともかく)興味ない」
といってみせたのは真実そのとおりだったのでしょう
837エトス:2012/02/29(水) 01:29:02.55
>>807
非特異であることをいうにはカスプも調べる必要があります.
(射影閉包を調べるわけですが)

[問題]
A∈M_n(Z) ⇒ tr(A^p)≡tr(A) (mod p)

838132人目の素数さん:2012/02/29(水) 02:20:09.22
839エトス:2012/02/29(水) 02:46:34.17
>>838
過去スレどうもです!
301を用いれば明らかですが,
313自体は誤っているようです.
しかし固有値を考えた時点で方針は正しい予感がします.
840132人目の素数さん:2012/02/29(水) 13:00:00.41
301 : 伊丹公理 : 04/12/10 00:30:19
p : 素数、 A : 整数を成分に持つ n 次正方行列とし、 f (t), g (t) を
A, A^p 固有多項式とする。このとき、このとき、これらの多項式の
i 次の係数は、 p を法として合同である (i = 0, 1, 2, .... n)。

313 : 132人目の素数さん : 04/12/15 22:00:00
 f(t^p)
=f(t)^p
=|tE−A|^p
=|(tE−A)^p|
=|t^pE−A^p|
=g(t^p)。
841エトス:2012/02/29(水) 16:46:34.21
>>840
訂正しておきます.313は正しいです!
なにか見間違いをしていたみたい;;
842132人目の素数さん:2012/03/01(木) 19:28:37.04
105より大きい任意の奇数は、互いに素な合成数の和で表現できることを証明せよ、
>>715の類題ですが、>>715よりはるかに簡単です。
843L:2012/03/02(金) 00:44:12.89
>>842
n>105は条件を満たさない奇数であると仮定します。
まず >>382 の人と同様な方法により、
105|n であることがいえます。
(7|nであることは105<128から微妙ですが
107,114,121,128はどれも5で割り切れないので
そこに着目すれば問題はないはずです)

各自然数kに対して、k番目の"奇"素数をq_kとします。
(q_1=3, q_2=5, q_3=7, q_4=11, ...)
nを割り切らない最小の"奇"素数をq_sとします。
このとき、n>(q_s)^2 がいえます。これを示します。

s=4 のとき n>105 かつ 105|n より
n≧315>11^2=121 だから上記の不等式は成立しています。

s≧5 のときは Π[i=1,s-1]q_i>(q_s)^2 より
(この不等式自体は例えばChebyshevの不等式からすぐでます)
やはり上記の不等式は成立しています。

k=n-(q_s)^2 とおきます。先ほど示したことからk≧1がいえます。
n,q_sはともに奇数なので、kは偶数であることがいえます。
5|n でしたので k=2 がありえないことがいえます。
よって、kは2より大きい偶数です。これは矛盾です。
844132人目の素数さん:2012/03/02(金) 01:10:00.37
2^3=1.
845L:2012/03/02(金) 01:29:38.15
>>844
そういえば2のmod 7での位数は3なので
7|n は"同様"の方法からでは明らかではないですね。

とにかく7|nがいえれば
あとの議論から問題はないはずです。
じゃあこうしましょう。7|nでないとすると、
n-7^2 は2より大きい偶数だから矛盾となる。
これで問題は消えたとおもいます。

というよりは最初からこの手法のほうがよかったです。
つまり、105|n でないとすれば
あるr∈{3,5,7}があって、gcd(n,r)=1 ですが、
このとき、n-r^2は2より大きい偶数となっている。
846L:2012/03/02(金) 01:33:16.20

汚くなったので、証明自体を書き直しておきます。
さらに、Chebyshevの不等式からすぐでるとか言っていたものも
きちんと証明を書いておくことにしておきます。
847L(書き直し):2012/03/02(金) 01:46:29.31
>>844
n>105は条件を満たさない奇数であると仮定します。
まず、3*5*7|n がいえます。
なぜなら、もしそうでないとすると、
あるr∈{3,5,7}に対して、gcd(n,r)=1 となっていて、
このとき、n-r^2は2より大きい偶数となってしまうので。
105|n かつ n>105 より、n≧315>121+2 がいえます。
ここから 11|n であることがいえます。
というのも、もしそうでないとすると、
n-11^2は2より大きい偶数となりますので。

各自然数kに対して、k番目の"奇"素数をq_kとします。
(q_1=3, q_2=5, q_3=7, q_4=11, ...)
nを割り切らない最小の"奇"素数をq_sとします。
上で示したことから、s≧5 であることがいえます。
このとき、n>(q_s)^2 がいえます。これを示します。

Chebyshevの不等式からとくに
q_s<2q_(s-1) かつ q_(s-1)<2q_(s-2) がいえますので、
(q_s)^2<4(q_(s-1))^2<4q_(s-1)*q_(s-1)
<4q_(s-1)*2q_(s-2)=2*4*q_(s-2)*q_(s-1)
<3*5*q_(s-2)*q_(s-1)≦Π[i=1,s-1]q_i≦n
よって示せました。

d=n-(q_s)^2 とおきます。先ほど示したことからd≧1がいえます。
n, q_sはともに奇数なので、dは偶数であることがいえます。
5|n でしたので d=2 がありえないことがいえます。
よって、dは2より大きい偶数です。これは矛盾です。
848L:2012/03/02(金) 02:16:15.74
おまけで条件を満たさない奇数n≦105を全て求めることにします。
まずn<19が条件を満たしていないことは明らかです。
(和が最小となる2つの互いに素な合成数のペアは(4,15)だから)

奇数n≧19が条件を満たしていないと仮定します。
19>11=3^2+2 より、3|n  がいえます。
n≧19かつ3|nを満たす奇数nで、n≦27を満たすものは
n=21,27の2つだけです。これらは条件を満たさないので、
(なぜなら、4+5^2=29>27 だから)
n>27としておきます。このとき、5|n がいえます。
n>27かつ15|nを満たす奇数nで、n≦51を満たすものは
n=45しかありませんが、これは条件を満たしません。(4+49>45 だから)
よって、n>51としておきます。このとき、7|n がいえます。

最後にn=105は条件を満たさないことに注意します(121+4>105だから)
ということはとくにnは105より大きい奇数として問題ありません。
以上より条件を満たさないすべての奇数nは
n=1,3,5,7,9,11,13,15,17,21,27,45,105 の13個となります。
849132人目の素数さん:2012/03/02(金) 04:10:00.06
1,3,5,7,9,11,15,21,27,45,105.
13=4+9.
17=8+9.
850132人目の素数さん:2012/03/02(金) 04:57:58.53
>>849
正解

>>848
ここが誤っている
「和が最小となる2つの互いに素な合成数のペアは(4,15)だから」

(4,9)が最小のペアだとおもう。
4+9>11=3^2+2 だから あとは問題ないとおもう。
最小ペアは(4,15)であるとしたことで生じた歪めが
13,17<19という2つのあやまった解を導き出したということ。
851132人目の素数さん:2012/03/02(金) 05:00:07.00
○歪(ひずみ)
×歪め
852132人目の素数さん:2012/03/03(土) 21:00:02.30
nの約数でない最小の奇素数をpとすると
nはp未満の全ての奇素数の倍数でpの倍数でない。
nがp未満の全ての奇素数の倍数でpの倍数でないとき
nがp^2+2より大きいと互いの素な合成数の和で表せ
nがp^2+2以下のとき互いの素な合成数の和で表せない。

p=3のとき
nが1の倍数で3の倍数でなく3^2+2以下のとき互いの素な合成数の和で表せない。
n=1,5,7,11。

p=5のとき
nが3の倍数で5の倍数でなく5^2+2以下のとき互いの素な合成数の和で表せない。
n=3,9,21,27。

p=7のとき
nが15の倍数で7の倍数でなく7^2+2以下のとき互いの素な合成数の和で表せない。
n=15,45。

p=11のとき
nが105の倍数で11の倍数でなく11^2+2以下のとき互いの素な合成数の和で表せない。
n=105。

pが13以上のとき
p未満の全ての奇素数の倍数はp^2+2以下ではない。
853132人目の素数さん:2012/03/04(日) 07:48:48.62
射精終了と証明終了の違いは?
854132人目の素数さん:2012/03/05(月) 03:23:48.43
pqr pq qr rp p q r
855132人目の素数さん:2012/03/05(月) 19:08:47.67
856132人目の素数さん:2012/03/11(日) 00:01:36.65
          __ノ)-'´ ̄ ̄`ー- 、_
        , '´  _. -‐'''"二ニニ=-`ヽ、
      /   /:::::; -‐''"        `ーノ
     /   /:::::/           \
     /    /::::::/          | | |  |
     |   |:::::/ /     |  | | | |  |
      |   |::/ / / |  | ||  | | ,ハ .| ,ハ|
      |   |/ / / /| ,ハノ| /|ノレ,ニ|ル' 
     |   |  | / / レ',二、レ′ ,ィイ|゙/   私は只の数ヲタなんかとは付き合わないわ。
.     |   \ ∠イ  ,イイ|    ,`-' |      頭が良くて数学が出来てかっこいい人。それが必要条件よ。
     |     l^,人|  ` `-'     ゝ  |        さらに Ann.of Math に論文書けば十分条件にもなるわよ。
      |      ` -'\       ー'  人          一番嫌いなのは論文数を増やすためにくだらない論文を書いて
    |        /(l     __/  ヽ、           良い論文の出版を遅らせるお馬鹿な人。
     |       (:::::`‐-、__  |::::`、     ヒニニヽ、         あなたの論文が Ann of Math に accept される確率は?
    |      / `‐-、::::::::::`‐-、::::\   /,ニニ、\            それとも最近は Inv. Math. の方が上かしら?
   |      |::::::::::::::::::|` -、:::::::,ヘ ̄|'、  ヒニ二、 \
.   |      /::::::::::::::::::|::::::::\/:::O`、::\   | '、   \
   |      /:::::::::::::::::::/:::::::::::::::::::::::::::::'、::::\ノ  ヽ、  |
  |      |:::::/:::::::::/:::::::::::::::::::::::::::::::::::'、',::::'、  /:\__/‐、
  |      |/:::::::::::/::::::::::::::::::::::::::::::::::O::| '、::| く::::::::::::: ̄|
   |     /_..-'´ ̄`ー-、:::::::::::::::::::::::::::::::::::|/:/`‐'::\;;;;;;;_|
   |    |/::::::::::::::::::::::\:::::::::::::::::::::::::::::|::/::::|::::/:::::::::::/
    |   /:::::::::::::::::::::::::::::::::|:::::::::::::::::::::O::|::|::::::|:::::::::::::::/
857132人目の素数さん:2012/03/11(日) 00:06:38.47
          __ノ)-'´ ̄ ̄`ー- 、_
        , '´  _. -‐'''"二ニニ=-`ヽ、
      /   /:::::; -‐''"        `ーノ
     /   /:::::/           \
     /    /::::::/          | | |  |
     |   |:::::/ /     |  | | | |  |
      |   |::/ / / |  | ||  | | ,ハ .| ,ハ|
      |   |/ / / /| ,ハノ| /|ノレ,ニ|ル' 
     |   |  | / / レ',二、レ′ ,ィイ|゙/   私は只の数ヲタなんかとは付き合わないわ。
.     |   \ ∠イ  ,イイ|    ,`-' |      頭が良くて数学が出来てかっこいい人。それが必要条件よ。
     |     l^,人|  ` `-'     ゝ  |        さらに Ann.of Math に論文書けば十分条件にもなるわよ。
      |      ` -'\       ー'  人          一番嫌いなのは論文数を増やすためにくだらない論文を書いて
    |        /(l     __/  ヽ、           良い論文の出版を遅らせるお馬鹿な人。
     |       (:::::`‐-、__  |::::`、     ヒニニヽ、         あなたの論文が Ann of Math に accept される確率は?
    |      / `‐-、::::::::::`‐-、::::\   /,ニニ、\            それとも最近は Inv. Math. の方が上かしら?
   |      |::::::::::::::::::|` -、:::::::,ヘ ̄|'、  ヒニ二、 \
.   |      /::::::::::::::::::|::::::::\/:::O`、::\   | '、   \
   |      /:::::::::::::::::::/:::::::::::::::::::::::::::::'、::::\ノ  ヽ、  |
  |      |:::::/:::::::::/:::::::::::::::::::::::::::::::::::'、',::::'、  /:\__/‐、
  |      |/:::::::::::/::::::::::::::::::::::::::::::::::O::| '、::| く::::::::::::: ̄|
   |     /_..-'´ ̄`ー-、:::::::::::::::::::::::::::::::::::|/:/`‐'::\;;;;;;;_|
   |    |/::::::::::::::::::::::\:::::::::::::::::::::::::::::|::/::::|::::/:::::::::::/
    |   /:::::::::::::::::::::::::::::::::|:::::::::::::::::::::O::|::|::::::|:::::::::::::::/
858132人目の素数さん:2012/03/11(日) 00:09:21.95
          __ノ)-'´ ̄ ̄`ー- 、_
        , '´  _. -‐'''"二ニニ=-`ヽ、
      /   /:::::; -‐''"        `ーノ
     /   /:::::/           \
     /    /::::::/          | | |  |
     |   |:::::/ /     |  | | | |  |
      |   |::/ / / |  | ||  | | ,ハ .| ,ハ|
      |   |/ / / /| ,ハノ| /|ノレ,ニ|ル' 
     |   |  | / / レ',二、レ′ ,ィイ|゙/   私は只の数ヲタなんかとは付き合わないわ。
.     |   \ ∠イ  ,イイ|    ,`-' |      頭が良くて数学が出来てかっこいい人。それが必要条件よ。
     |     l^,人|  ` `-'     ゝ  |        さらに Ann.of Math に論文書けば十分条件にもなるわよ。
      |      ` -'\       ー'  人          一番嫌いなのは論文数を増やすためにくだらない論文を書いて
    |        /(l     __/  ヽ、           良い論文の出版を遅らせるお馬鹿な人。
     |       (:::::`‐-、__  |::::`、     ヒニニヽ、         あなたの論文が Ann of Math に accept される確率は?
    |      / `‐-、::::::::::`‐-、::::\   /,ニニ、\            それとも最近は Inv. Math. の方が上かしら?
   |      |::::::::::::::::::|` -、:::::::,ヘ ̄|'、  ヒニ二、 \
.   |      /::::::::::::::::::|::::::::\/:::O`、::\   | '、   \
   |      /:::::::::::::::::::/:::::::::::::::::::::::::::::'、::::\ノ  ヽ、  |
  |      |:::::/:::::::::/:::::::::::::::::::::::::::::::::::'、',::::'、  /:\__/‐、
  |      |/:::::::::::/::::::::::::::::::::::::::::::::::O::| '、::| く::::::::::::: ̄|
   |     /_..-'´ ̄`ー-、:::::::::::::::::::::::::::::::::::|/:/`‐'::\;;;;;;;_|
   |    |/::::::::::::::::::::::\:::::::::::::::::::::::::::::|::/::::|::::/:::::::::::/
    |   /:::::::::::::::::::::::::::::::::|:::::::::::::::::::::O::|::|::::::|:::::::::::::::/
859132人目の素数さん:2012/03/11(日) 00:09:49.80
          __ノ)-'´ ̄ ̄`ー- 、_
        , '´  _. -‐'''"二ニニ=-`ヽ、
      /   /:::::; -‐''"        `ーノ
     /   /:::::/           \
     /    /::::::/          | | |  |
     |   |:::::/ /     |  | | | |  |
      |   |::/ / / |  | ||  | | ,ハ .| ,ハ|
      |   |/ / / /| ,ハノ| /|ノレ,ニ|ル' 
     |   |  | / / レ',二、レ′ ,ィイ|゙/   私は只の数ヲタなんかとは付き合わないわ。
.     |   \ ∠イ  ,イイ|    ,`-' |      頭が良くて数学が出来てかっこいい人。それが必要条件よ。
     |     l^,人|  ` `-'     ゝ  |        さらに Ann.of Math に論文書けば十分条件にもなるわよ。
      |      ` -'\       ー'  人          一番嫌いなのは論文数を増やすためにくだらない論文を書いて
    |        /(l     __/  ヽ、           良い論文の出版を遅らせるお馬鹿な人。
     |       (:::::`‐-、__  |::::`、     ヒニニヽ、         あなたの論文が Ann of Math に accept される確率は?
    |      / `‐-、::::::::::`‐-、::::\   /,ニニ、\            それとも最近は Inv. Math. の方が上かしら?
   |      |::::::::::::::::::|` -、:::::::,ヘ ̄|'、  ヒニ二、 \
.   |      /::::::::::::::::::|::::::::\/:::O`、::\   | '、   \
   |      /:::::::::::::::::::/:::::::::::::::::::::::::::::'、::::\ノ  ヽ、  |
  |      |:::::/:::::::::/:::::::::::::::::::::::::::::::::::'、',::::'、  /:\__/‐、
  |      |/:::::::::::/::::::::::::::::::::::::::::::::::O::| '、::| く::::::::::::: ̄|
   |     /_..-'´ ̄`ー-、:::::::::::::::::::::::::::::::::::|/:/`‐'::\;;;;;;;_|
   |    |/::::::::::::::::::::::\:::::::::::::::::::::::::::::|::/::::|::::/:::::::::::/
    |   /:::::::::::::::::::::::::::::::::|:::::::::::::::::::::O::|::|::::::|:::::::::::::::/
860132人目の素数さん:2012/03/11(日) 00:10:24.01
          __ノ)-'´ ̄ ̄`ー- 、_
        , '´  _. -‐'''"二ニニ=-`ヽ、
      /   /:::::; -‐''"        `ーノ
     /   /:::::/           \
     /    /::::::/          | | |  |
     |   |:::::/ /     |  | | | |  |
      |   |::/ / / |  | ||  | | ,ハ .| ,ハ|
      |   |/ / / /| ,ハノ| /|ノレ,ニ|ル' 
     |   |  | / / レ',二、レ′ ,ィイ|゙/   私は只の数ヲタなんかとは付き合わないわ。
.     |   \ ∠イ  ,イイ|    ,`-' |      頭が良くて数学が出来てかっこいい人。それが必要条件よ。
     |     l^,人|  ` `-'     ゝ  |        さらに Ann.of Math に論文書けば十分条件にもなるわよ。
      |      ` -'\       ー'  人          一番嫌いなのは論文数を増やすためにくだらない論文を書いて
    |        /(l     __/  ヽ、           良い論文の出版を遅らせるお馬鹿な人。
     |       (:::::`‐-、__  |::::`、     ヒニニヽ、         あなたの論文が Ann of Math に accept される確率は?
    |      / `‐-、::::::::::`‐-、::::\   /,ニニ、\            それとも最近は Inv. Math. の方が上かしら?
   |      |::::::::::::::::::|` -、:::::::,ヘ ̄|'、  ヒニ二、 \
.   |      /::::::::::::::::::|::::::::\/:::O`、::\   | '、   \
   |      /:::::::::::::::::::/:::::::::::::::::::::::::::::'、::::\ノ  ヽ、  |
  |      |:::::/:::::::::/:::::::::::::::::::::::::::::::::::'、',::::'、  /:\__/‐、
  |      |/:::::::::::/::::::::::::::::::::::::::::::::::O::| '、::| く::::::::::::: ̄|
   |     /_..-'´ ̄`ー-、:::::::::::::::::::::::::::::::::::|/:/`‐'::\;;;;;;;_|
   |    |/::::::::::::::::::::::\:::::::::::::::::::::::::::::|::/::::|::::/:::::::::::/
    |   /:::::::::::::::::::::::::::::::::|:::::::::::::::::::::O::|::|::::::|:::::::::::::::/
861132人目の素数さん:2012/03/11(日) 00:10:47.86
          __ノ)-'´ ̄ ̄`ー- 、_
        , '´  _. -‐'''"二ニニ=-`ヽ、
      /   /:::::; -‐''"        `ーノ
     /   /:::::/           \
     /    /::::::/          | | |  |
     |   |:::::/ /     |  | | | |  |
      |   |::/ / / |  | ||  | | ,ハ .| ,ハ|
      |   |/ / / /| ,ハノ| /|ノレ,ニ|ル' 
     |   |  | / / レ',二、レ′ ,ィイ|゙/   私は只の数ヲタなんかとは付き合わないわ。
.     |   \ ∠イ  ,イイ|    ,`-' |      頭が良くて数学が出来てかっこいい人。それが必要条件よ。
     |     l^,人|  ` `-'     ゝ  |        さらに Ann.of Math に論文書けば十分条件にもなるわよ。
      |      ` -'\       ー'  人          一番嫌いなのは論文数を増やすためにくだらない論文を書いて
    |        /(l     __/  ヽ、           良い論文の出版を遅らせるお馬鹿な人。
     |       (:::::`‐-、__  |::::`、     ヒニニヽ、         あなたの論文が Ann of Math に accept される確率は?
    |      / `‐-、::::::::::`‐-、::::\   /,ニニ、\            それとも最近は Inv. Math. の方が上かしら?
   |      |::::::::::::::::::|` -、:::::::,ヘ ̄|'、  ヒニ二、 \
.   |      /::::::::::::::::::|::::::::\/:::O`、::\   | '、   \
   |      /:::::::::::::::::::/:::::::::::::::::::::::::::::'、::::\ノ  ヽ、  |
  |      |:::::/:::::::::/:::::::::::::::::::::::::::::::::::'、',::::'、  /:\__/‐、
  |      |/:::::::::::/::::::::::::::::::::::::::::::::::O::| '、::| く::::::::::::: ̄|
   |     /_..-'´ ̄`ー-、:::::::::::::::::::::::::::::::::::|/:/`‐'::\;;;;;;;_|
   |    |/::::::::::::::::::::::\:::::::::::::::::::::::::::::|::/::::|::::/:::::::::::/
    |   /:::::::::::::::::::::::::::::::::|:::::::::::::::::::::O::|::|::::::|:::::::::::::::/
862132人目の素数さん:2012/03/11(日) 00:11:14.11
          __ノ)-'´ ̄ ̄`ー- 、_
        , '´  _. -‐'''"二ニニ=-`ヽ、
      /   /:::::; -‐''"        `ーノ
     /   /:::::/           \
     /    /::::::/          | | |  |
     |   |:::::/ /     |  | | | |  |
      |   |::/ / / |  | ||  | | ,ハ .| ,ハ|
      |   |/ / / /| ,ハノ| /|ノレ,ニ|ル' 
     |   |  | / / レ',二、レ′ ,ィイ|゙/   私は只の数ヲタなんかとは付き合わないわ。
.     |   \ ∠イ  ,イイ|    ,`-' |      頭が良くて数学が出来てかっこいい人。それが必要条件よ。
     |     l^,人|  ` `-'     ゝ  |        さらに Ann.of Math に論文書けば十分条件にもなるわよ。
      |      ` -'\       ー'  人          一番嫌いなのは論文数を増やすためにくだらない論文を書いて
    |        /(l     __/  ヽ、           良い論文の出版を遅らせるお馬鹿な人。
     |       (:::::`‐-、__  |::::`、     ヒニニヽ、         あなたの論文が Ann of Math に accept される確率は?
    |      / `‐-、::::::::::`‐-、::::\   /,ニニ、\            それとも最近は Inv. Math. の方が上かしら?
   |      |::::::::::::::::::|` -、:::::::,ヘ ̄|'、  ヒニ二、 \
.   |      /::::::::::::::::::|::::::::\/:::O`、::\   | '、   \
   |      /:::::::::::::::::::/:::::::::::::::::::::::::::::'、::::\ノ  ヽ、  |
  |      |:::::/:::::::::/:::::::::::::::::::::::::::::::::::'、',::::'、  /:\__/‐、
  |      |/:::::::::::/::::::::::::::::::::::::::::::::::O::| '、::| く::::::::::::: ̄|
   |     /_..-'´ ̄`ー-、:::::::::::::::::::::::::::::::::::|/:/`‐'::\;;;;;;;_|
   |    |/::::::::::::::::::::::\:::::::::::::::::::::::::::::|::/::::|::::/:::::::::::/
    |   /:::::::::::::::::::::::::::::::::|:::::::::::::::::::::O::|::|::::::|:::::::::::::::/
863132人目の素数さん:2012/03/11(日) 00:11:37.46
          __ノ)-'´ ̄ ̄`ー- 、_
        , '´  _. -‐'''"二ニニ=-`ヽ、
      /   /:::::; -‐''"        `ーノ
     /   /:::::/           \
     /    /::::::/          | | |  |
     |   |:::::/ /     |  | | | |  |
      |   |::/ / / |  | ||  | | ,ハ .| ,ハ|
      |   |/ / / /| ,ハノ| /|ノレ,ニ|ル' 
     |   |  | / / レ',二、レ′ ,ィイ|゙/   私は只の数ヲタなんかとは付き合わないわ。
.     |   \ ∠イ  ,イイ|    ,`-' |      頭が良くて数学が出来てかっこいい人。それが必要条件よ。
     |     l^,人|  ` `-'     ゝ  |        さらに Ann.of Math に論文書けば十分条件にもなるわよ。
      |      ` -'\       ー'  人          一番嫌いなのは論文数を増やすためにくだらない論文を書いて
    |        /(l     __/  ヽ、           良い論文の出版を遅らせるお馬鹿な人。
     |       (:::::`‐-、__  |::::`、     ヒニニヽ、         あなたの論文が Ann of Math に accept される確率は?
    |      / `‐-、::::::::::`‐-、::::\   /,ニニ、\            それとも最近は Inv. Math. の方が上かしら?
   |      |::::::::::::::::::|` -、:::::::,ヘ ̄|'、  ヒニ二、 \
.   |      /::::::::::::::::::|::::::::\/:::O`、::\   | '、   \
   |      /:::::::::::::::::::/:::::::::::::::::::::::::::::'、::::\ノ  ヽ、  |
  |      |:::::/:::::::::/:::::::::::::::::::::::::::::::::::'、',::::'、  /:\__/‐、
  |      |/:::::::::::/::::::::::::::::::::::::::::::::::O::| '、::| く::::::::::::: ̄|
   |     /_..-'´ ̄`ー-、:::::::::::::::::::::::::::::::::::|/:/`‐'::\;;;;;;;_|
   |    |/::::::::::::::::::::::\:::::::::::::::::::::::::::::|::/::::|::::/:::::::::::/
    |   /:::::::::::::::::::::::::::::::::|:::::::::::::::::::::O::|::|::::::|:::::::::::::::/
864132人目の素数さん:2012/03/11(日) 00:12:00.81
          __ノ)-'´ ̄ ̄`ー- 、_
        , '´  _. -‐'''"二ニニ=-`ヽ、
      /   /:::::; -‐''"        `ーノ
     /   /:::::/           \
     /    /::::::/          | | |  |
     |   |:::::/ /     |  | | | |  |
      |   |::/ / / |  | ||  | | ,ハ .| ,ハ|
      |   |/ / / /| ,ハノ| /|ノレ,ニ|ル' 
     |   |  | / / レ',二、レ′ ,ィイ|゙/   私は只の数ヲタなんかとは付き合わないわ。
.     |   \ ∠イ  ,イイ|    ,`-' |      頭が良くて数学が出来てかっこいい人。それが必要条件よ。
     |     l^,人|  ` `-'     ゝ  |        さらに Ann.of Math に論文書けば十分条件にもなるわよ。
      |      ` -'\       ー'  人          一番嫌いなのは論文数を増やすためにくだらない論文を書いて
    |        /(l     __/  ヽ、           良い論文の出版を遅らせるお馬鹿な人。
     |       (:::::`‐-、__  |::::`、     ヒニニヽ、         あなたの論文が Ann of Math に accept される確率は?
    |      / `‐-、::::::::::`‐-、::::\   /,ニニ、\            それとも最近は Inv. Math. の方が上かしら?
   |      |::::::::::::::::::|` -、:::::::,ヘ ̄|'、  ヒニ二、 \
.   |      /::::::::::::::::::|::::::::\/:::O`、::\   | '、   \
   |      /:::::::::::::::::::/:::::::::::::::::::::::::::::'、::::\ノ  ヽ、  |
  |      |:::::/:::::::::/:::::::::::::::::::::::::::::::::::'、',::::'、  /:\__/‐、
  |      |/:::::::::::/::::::::::::::::::::::::::::::::::O::| '、::| く::::::::::::: ̄|
   |     /_..-'´ ̄`ー-、:::::::::::::::::::::::::::::::::::|/:/`‐'::\;;;;;;;_|
   |    |/::::::::::::::::::::::\:::::::::::::::::::::::::::::|::/::::|::::/:::::::::::/
    |   /:::::::::::::::::::::::::::::::::|:::::::::::::::::::::O::|::|::::::|:::::::::::::::/
865猫vs運営 ◆MuKUnGPXAY :2012/03/11(日) 01:32:09.70

866132人目の素数さん:2012/03/11(日) 23:47:09.42
問題投下
奇数の不思議数が1つでも存在するとき、奇数の不思議数が無数に存在することを証明せよ。

不思議数については下記サイトを参考にしてください。
http://ja.wikipedia.org/wiki/%E4%B8%8D%E6%80%9D%E8%AD%B0%E6%95%B0
867132人目の素数さん:2012/03/12(月) 03:15:25.68
aを不思議数とするとき、
p>σ(a)-a なる任意の素数pに対して、
apは不思議数となることを示す。

(証明)
aは過剰数だから、σ(a)-a>a がいえるので、gcd(p,a)=1 である。
aの正の約数全体の集合をSとする。

apが擬似完全数であると仮定すると、
ある集合A,B⊂Sが存在していて、
(注意:A,Bは空になることもある)
Σ[d∈A]d+Σ[d'∈B]pd' = ap が成立している。
これから、Σ[d∈A]d/p+Σ[d'∈B]d' = a がいえる。
よって、Σ[d∈A]d/p は非負整数であるといえる。
しかし、p>σ(a)-a であったから、Σ[d∈A]d = 0 となる。
(つまり、Aは空集合であるということ)
したがって、Σ[d'∈B]d' = a がいえるが、
これは aが不思議数であるという仮定に矛盾している。
(不思議数ならば擬似完全数でない)

最後に過剰数の倍数は過剰数(証明は簡単なので省く)であるから、
結局、apが不思議数であることがいえた。
868867の修正ver:2012/03/12(月) 03:24:28.11
aを不思議数とするとき、
p>σ(a) なる任意の素数pに対して、
apは不思議数となることを示す。

(証明)
p>σ(a)>a より、gcd(p,a)=1 である。
aの正の約数全体の集合をSとする。

apが擬似完全数であると仮定すると、
ある集合A,B⊂Sが存在していて、
(注意:A,Bは空になることもある)
Σ[d∈A]d+Σ[d'∈B]pd' = ap が成立している。
これから、Σ[d∈A]d/p+Σ[d'∈B]d' = a がいえる。
よって、Σ[d∈A]d/p は非負整数であるといえる。
しかし、p>σ(a) であったから、Σ[d∈A]d = 0 となる。
(つまり、Aは空集合であるということ)
したがって、Σ[d'∈B]d' = a がいえるが、
これは aが不思議数であるという仮定に矛盾している。
(不思議数ならば擬似完全数でない)

最後に過剰数の倍数は過剰数(証明は簡単なので省く)であるから、
結局、apが不思議数であることがいえた。
869132人目の素数さん:2012/03/12(月) 03:29:06.53
Σ[d∈A]d≦σ(a)-a とは限らないので、
(apを考えている段階で、例えばA=Sの可能性はありえる)
その点を誤ったのが最初の方です。
つまり、pの取り方が甘かったということ。

訂正verではpの取り方が確実に信頼できる方法ですので、
(Σ[d∈A]d ≦ σ(a) の成立は明らかですから)
問題がないということです。
870132人目の素数さん:2012/03/12(月) 20:36:46.24
(x-a)(x-b)...(x-z) = ????
871132人目の素数さん:2012/03/12(月) 22:22:06.17
(x-a)(x-b)…(x-w)(x-x)(x-y)(x-z)
というオチだったりして
872132人目の素数さん:2012/03/15(木) 07:42:23.75
          __ノ)-'´ ̄ ̄`ー- 、_
        , '´  _. -‐'''"二ニニ=-`ヽ、
      /   /:::::; -‐''"        `ーノ
     /   /:::::/           \
     /    /::::::/          | | |  |
     |   |:::::/ /     |  | | | |  |
      |   |::/ / / |  | ||  | | ,ハ .| ,ハ|
      |   |/ / / /| ,ハノ| /|ノレ,ニ|ル' 
     |   |  | / / レ',二、レ′ ,ィイ|゙/   私は只の数ヲタなんかとは付き合わないわ。
.     |   \ ∠イ  ,イイ|    ,`-' |      頭が良くて数学が出来てかっこいい人。それが必要条件よ。
     |     l^,人|  ` `-'     ゝ  |        さらに Ann.of Math に論文書けば十分条件にもなるわよ。
      |      ` -'\       ー'  人          一番嫌いなのは論文数を増やすためにくだらない論文を書いて
    |        /(l     __/  ヽ、           良い論文の出版を遅らせるお馬鹿な人。
     |       (:::::`‐-、__  |::::`、     ヒニニヽ、         あなたの論文が Ann of Math に accept される確率は?
    |      / `‐-、::::::::::`‐-、::::\   /,ニニ、\            それとも最近は Inv. Math. の方が上かしら?
   |      |::::::::::::::::::|` -、:::::::,ヘ ̄|'、  ヒニ二、 \
.   |      /::::::::::::::::::|::::::::\/:::O`、::\   | '、   \
   |      /:::::::::::::::::::/:::::::::::::::::::::::::::::'、::::\ノ  ヽ、  |
  |      |:::::/:::::::::/:::::::::::::::::::::::::::::::::::'、',::::'、  /:\__/‐、
  |      |/:::::::::::/::::::::::::::::::::::::::::::::::O::| '、::| く::::::::::::: ̄|
   |     /_..-'´ ̄`ー-、:::::::::::::::::::::::::::::::::::|/:/`‐'::\;;;;;;;_|
   |    |/::::::::::::::::::::::\:::::::::::::::::::::::::::::|::/::::|::::/:::::::::::/
    |   /:::::::::::::::::::::::::::::::::|:::::::::::::::::::::O::|::|::::::|:::::::::::::::/
873132人目の素数さん:2012/03/15(木) 13:55:28.24
新作問題。

f_i (z) , i = 1, 2, ... , k を高々 d_i 次複素係数多項式、
a_i , i = 1, 2, ... , k を 0 で無い複素数、
F(m) = f_1 (n)*(a_1)^n + f_2 (n)*(a_2)^n + ... + f_n (n)*(a_k)^n
と置く。 F(n) が n = 1, 2, 3, ..... , Σ_i d_i, (Σ_i d_i) + 1 に対して整数なら、
全ての n に対し、 F(n) は整数となる。
874132人目の素数さん:2012/03/15(木) 14:30:00.41
F(m)=2(1/2)^m。
F(1)=1。
F(2)=1/2。
875132人目の素数さん:2012/03/15(木) 17:50:32.74
間違った。出直してくる。
876873 :2012/03/18(日) 09:44:40.86
問題訂正

f_i (z) , i = 1, 2, ... , k を高々 d_i 次複素係数多項式、
a_i , i = 1, 2, ... , k を 0 で無い複素数で、代数的整数になっている物とする。
F(m) = f_1 (n)*(a_1)^n + f_2 (n)*(a_2)^n + ... + f_n (n)*(a_k)^n
と置く。F(n) が

n = 1, 2, 3, ..... , Σ_i d_i, (Σ_i d_i) + 1, Σ_i d_i, (Σ_i d_i) + 2, .... , Σ_i d_i, (Σ_i d_i) + k

に対して整数なら、全ての n に対し、 F(n) は整数となる。
877132人目の素数さん:2012/03/18(日) 10:38:01.20
0でないは必要?F(m)は定数?全てって?
878132人目の素数さん:2012/03/18(日) 10:38:59.81
f_n
879132人目の素数さん:2012/03/18(日) 12:00:00.01
F(m)=(2√(2))^m−2(√(2))^m。
F(1)=0。
F(2)=4。
F(3)=12√(2)。
880132人目の素数さん:2012/03/18(日) 17:53:39.99
また間違った。修正は無期延期。
881132人目の素数さん:2012/03/25(日) 20:50:21.00
          __ノ)-'´ ̄ ̄`ー- 、_
        , '´  _. -‐'''"二ニニ=-`ヽ、
      /   /:::::; -‐''"        `ーノ
     /   /:::::/           \
     /    /::::::/          | | |  |
     |   |:::::/ /     |  | | | |  |
      |   |::/ / / |  | ||  | | ,ハ .| ,ハ|
      |   |/ / / /| ,ハノ| /|ノレ,ニ|ル' 
     |   |  | / / レ',二、レ′ ,ィイ|゙/   私は只の数ヲタなんかとは付き合わないわ。
.     |   \ ∠イ  ,イイ|    ,`-' |      頭が良くて数学が出来てかっこいい人。それが必要条件よ。
     |     l^,人|  ` `-'     ゝ  |        さらに Ann.of Math に論文書けば十分条件にもなるわよ。
      |      ` -'\       ー'  人          一番嫌いなのは論文数を増やすためにくだらない論文を書いて
    |        /(l     __/  ヽ、           良い論文の出版を遅らせるお馬鹿な人。
     |       (:::::`‐-、__  |::::`、     ヒニニヽ、         あなたの論文が Ann of Math に accept される確率は?
    |      / `‐-、::::::::::`‐-、::::\   /,ニニ、\            それとも最近は Inv. Math. の方が上かしら?
   |      |::::::::::::::::::|` -、:::::::,ヘ ̄|'、  ヒニ二、 \
.   |      /::::::::::::::::::|::::::::\/:::O`、::\   | '、   \
   |      /:::::::::::::::::::/:::::::::::::::::::::::::::::'、::::\ノ  ヽ、  |
  |      |:::::/:::::::::/:::::::::::::::::::::::::::::::::::'、',::::'、  /:\__/‐、
  |      |/:::::::::::/::::::::::::::::::::::::::::::::::O::| '、::| く::::::::::::: ̄|
   |     /_..-'´ ̄`ー-、:::::::::::::::::::::::::::::::::::|/:/`‐'::\;;;;;;;_|
   |    |/::::::::::::::::::::::\:::::::::::::::::::::::::::::|::/::::|::::/:::::::::::/
    |   /:::::::::::::::::::::::::::::::::|:::::::::::::::::::::O::|::|::::::|:::::::::::::::/
882132人目の素数さん:2012/03/26(月) 10:45:39.72
数列{a_{n}}(n=0,1,2,3,...)を以下の漸化式で定める。
a_{0}=1
a_{n+1}=10*a_{n}+4

a_{m}*a_{n}が平方数となる(m,n)(m<n)の組をすべて求めよ。
883132人目の素数さん:2012/03/26(月) 11:00:40.78
          __ノ)-'´ ̄ ̄`ー- 、_
        , '´  _. -‐'''"二ニニ=-`ヽ、
      /   /:::::; -‐''"        `ーノ
     /   /:::::/           \
     /    /::::::/          | | |  |
     |   |:::::/ /     |  | | | |  |
      |   |::/ / / |  | ||  | | ,ハ .| ,ハ|
      |   |/ / / /| ,ハノ| /|ノレ,ニ|ル' 
     |   |  | / / レ',二、レ′ ,ィイ|゙/   私は只の数ヲタなんかとは付き合わないわ。
.     |   \ ∠イ  ,イイ|    ,`-' |      頭が良くて数学が出来てかっこいい人。それが必要条件よ。
     |     l^,人|  ` `-'     ゝ  |        さらに Ann.of Math に論文書けば十分条件にもなるわよ。
      |      ` -'\       ー'  人          一番嫌いなのは論文数を増やすためにくだらない論文を書いて
    |        /(l     __/  ヽ、           良い論文の出版を遅らせるお馬鹿な人。
     |       (:::::`‐-、__  |::::`、     ヒニニヽ、         あなたの論文が Ann of Math に accept される確率は?
    |      / `‐-、::::::::::`‐-、::::\   /,ニニ、\            それとも最近は Inv. Math. の方が上かしら?
   |      |::::::::::::::::::|` -、:::::::,ヘ ̄|'、  ヒニ二、 \
.   |      /::::::::::::::::::|::::::::\/:::O`、::\   | '、   \
   |      /:::::::::::::::::::/:::::::::::::::::::::::::::::'、::::\ノ  ヽ、  |
  |      |:::::/:::::::::/:::::::::::::::::::::::::::::::::::'、',::::'、  /:\__/‐、
  |      |/:::::::::::/::::::::::::::::::::::::::::::::::O::| '、::| く::::::::::::: ̄|
   |     /_..-'´ ̄`ー-、:::::::::::::::::::::::::::::::::::|/:/`‐'::\;;;;;;;_|
   |    |/::::::::::::::::::::::\:::::::::::::::::::::::::::::|::/::::|::::/:::::::::::/
    |   /:::::::::::::::::::::::::::::::::|:::::::::::::::::::::O::|::|::::::|:::::::::::::::/
884132人目の素数さん:2012/03/26(月) 12:32:23.66
          __ノ)-'´ ̄ ̄`ー- 、_
        , '´  _. -‐'''"二ニニ=-`ヽ、
      /   /:::::; -‐''"        `ーノ
     /   /:::::/           \
     /    /::::::/          | | |  |
     |   |:::::/ /     |  | | | |  |
      |   |::/ / / |  | ||  | | ,ハ .| ,ハ|
      |   |/ / / /| ,ハノ| /|ノレ,ニ|ル' 
     |   |  | / / レ',二、レ′ ,ィイ|゙/   私は只の数ヲタなんかとは付き合わないわ。
.     |   \ ∠イ  ,イイ|    ,`-' |      頭が良くて数学が出来てかっこいい人。それが必要条件よ。
     |     l^,人|  ` `-'     ゝ  |        さらに Ann.of Math に論文書けば十分条件にもなるわよ。
      |      ` -'\       ー'  人          一番嫌いなのは論文数を増やすためにくだらない論文を書いて
    |        /(l     __/  ヽ、           良い論文の出版を遅らせるお馬鹿な人。
     |       (:::::`‐-、__  |::::`、     ヒニニヽ、         あなたの論文が Ann of Math に accept される確率は?
    |      / `‐-、::::::::::`‐-、::::\   /,ニニ、\            それとも最近は Inv. Math. の方が上かしら?
   |      |::::::::::::::::::|` -、:::::::,ヘ ̄|'、  ヒニ二、 \
.   |      /::::::::::::::::::|::::::::\/:::O`、::\   | '、   \
   |      /:::::::::::::::::::/:::::::::::::::::::::::::::::'、::::\ノ  ヽ、  |
  |      |:::::/:::::::::/:::::::::::::::::::::::::::::::::::'、',::::'、  /:\__/‐、
  |      |/:::::::::::/::::::::::::::::::::::::::::::::::O::| '、::| く::::::::::::: ̄|
   |     /_..-'´ ̄`ー-、:::::::::::::::::::::::::::::::::::|/:/`‐'::\;;;;;;;_|
   |    |/::::::::::::::::::::::\:::::::::::::::::::::::::::::|::/::::|::::/:::::::::::/
    |   /:::::::::::::::::::::::::::::::::|:::::::::::::::::::::O::|::|::::::|:::::::::::::::/
885132人目の素数さん:2012/04/17(火) 21:32:18.03
D を、次の条件を満たす自然数とする

i) D ≡ 1 mod. 4
ii) D は平方数では無い

この時不定方程式

x^2 - D*y^2 = 4

は無限に多くの整数解を持つ事を示せ
886132人目の素数さん:2012/04/18(水) 01:51:39.82
>>885
x^2-Dy^2=1 の整数解が無限に存在することから明らか^^;
条件i)は不要です^^;
887:2012/04/20(金) 19:03:20.47
[問題]
a0>1,a_(n+1)=[(3/2)a_n]より整数列(a_n)を定めるならば、
6|a_n を満たすような自然数nが無限に存在することを示せ。

[・]は床関数(またはガウスの記号)とします。
8888 8 8 :2012/04/21(土) 01:11:31.08
8 8 8
889132人目の素数さん:2012/04/21(土) 01:11:43.00
6|a_n  って a_nが6で割り切れるって意味でいいんだっけ?
890132人目の素数さん:2012/04/21(土) 01:46:06.27
>>885
Dが平方数ではない自然数であれば、以下のように解ける(初等整数論の有名問題)。

ペル方程式の非自明な解a,bをとる。
正の整数nに対してx(n)、y(n)を、x(n)+y(n)√D=(a+b√D)^nをみたすように取る。
このとき(x(n),y(n) )はx^2 -Dy^2=1をみたす。

尚、ペル方程式の非自明な解の存在については
http://aozoragakuen.sakura.ne.jp/suuron/node72.html
にある。
891132人目の素数さん:2012/04/21(土) 07:45:33.14
          __ノ)-'´ ̄ ̄`ー- 、_
        , '´  _. -‐'''"二ニニ=-`ヽ、
      /   /:::::; -‐''"        `ーノ
     /   /:::::/           \
     /    /::::::/          | | |  |
     |   |:::::/ /     |  | | | |  |
      |   |::/ / / |  | ||  | | ,ハ .| ,ハ|
      |   |/ / / /| ,ハノ| /|ノレ,ニ|ル' 
     |   |  | / / レ',二、レ′ ,ィイ|゙/   私は只の数ヲタなんかとは付き合わないわ。
.     |   \ ∠イ  ,イイ|    ,`-' |      頭が良くて数学が出来てかっこいい人。それが必要条件よ。
     |     l^,人|  ` `-'     ゝ  |        さらに Ann.of Math に論文書けば十分条件にもなるわよ。
      |      ` -'\       ー'  人          一番嫌いなのは論文数を増やすためにくだらない論文を書いて
    |        /(l     __/  ヽ、           良い論文の出版を遅らせるお馬鹿な人。
     |       (:::::`‐-、__  |::::`、     ヒニニヽ、         あなたの論文が Ann of Math に accept される確率は?
    |      / `‐-、::::::::::`‐-、::::\   /,ニニ、\            それとも最近は Inv. Math. の方が上かしら?
   |      |::::::::::::::::::|` -、:::::::,ヘ ̄|'、  ヒニ二、 \
.   |      /::::::::::::::::::|::::::::\/:::O`、::\   | '、   \
   |      /:::::::::::::::::::/:::::::::::::::::::::::::::::'、::::\ノ  ヽ、  |
  |      |:::::/:::::::::/:::::::::::::::::::::::::::::::::::'、',::::'、  /:\__/‐、
  |      |/:::::::::::/::::::::::::::::::::::::::::::::::O::| '、::| く::::::::::::: ̄|
   |     /_..-'´ ̄`ー-、:::::::::::::::::::::::::::::::::::|/:/`‐'::\;;;;;;;_|
   |    |/::::::::::::::::::::::\:::::::::::::::::::::::::::::|::/::::|::::/:::::::::::/
    |   /:::::::::::::::::::::::::::::::::|:::::::::::::::::::::O::|::|::::::|:::::::::::::::/
892132人目の素数さん:2012/04/21(土) 07:49:50.44
          __ノ)-'´ ̄ ̄`ー- 、_
        , '´  _. -‐'''"二ニニ=-`ヽ、
      /   /:::::; -‐''"        `ーノ
     /   /:::::/           \
     /    /::::::/          | | |  |
     |   |:::::/ /     |  | | | |  |
      |   |::/ / / |  | ||  | | ,ハ .| ,ハ|
      |   |/ / / /| ,ハノ| /|ノレ,ニ|ル' 
     |   |  | / / レ',二、レ′ ,ィイ|゙/   私は只の数ヲタなんかとは付き合わないわ。
.     |   \ ∠イ  ,イイ|    ,`-' |      頭が良くて数学が出来てかっこいい人。それが必要条件よ。
     |     l^,人|  ` `-'     ゝ  |        さらに Ann.of Math に論文書けば十分条件にもなるわよ。
      |      ` -'\       ー'  人          一番嫌いなのは論文数を増やすためにくだらない論文を書いて
    |        /(l     __/  ヽ、           良い論文の出版を遅らせるお馬鹿な人。
     |       (:::::`‐-、__  |::::`、     ヒニニヽ、         あなたの論文が Ann of Math に accept される確率は?
    |      / `‐-、::::::::::`‐-、::::\   /,ニニ、\            それとも最近は Inv. Math. の方が上かしら?
   |      |::::::::::::::::::|` -、:::::::,ヘ ̄|'、  ヒニ二、 \
.   |      /::::::::::::::::::|::::::::\/:::O`、::\   | '、   \
   |      /:::::::::::::::::::/:::::::::::::::::::::::::::::'、::::\ノ  ヽ、  |
  |      |:::::/:::::::::/:::::::::::::::::::::::::::::::::::'、',::::'、  /:\__/‐、
  |      |/:::::::::::/::::::::::::::::::::::::::::::::::O::| '、::| く::::::::::::: ̄|
   |     /_..-'´ ̄`ー-、:::::::::::::::::::::::::::::::::::|/:/`‐'::\;;;;;;;_|
   |    |/::::::::::::::::::::::\:::::::::::::::::::::::::::::|::/::::|::::/:::::::::::/
    |   /:::::::::::::::::::::::::::::::::|:::::::::::::::::::::O::|::|::::::|:::::::::::::::/
893132人目の素数さん:2012/04/21(土) 07:55:42.21
x^2 - D*y^2 = 4
16-3*4=4
894:2012/04/21(土) 13:21:57.43
>>889
はい
895132人目の素数さん:2012/04/21(土) 16:04:48.11
>>893
勘違いしていたね。
でも(x,y)がx^2-Dy^2=1の解なら、(2x,2y)がx^2-Dy^2=4の解になるのはいいよね。
896132人目の素数さん:2012/04/29(日) 16:04:49.06
          __ノ)-'´ ̄ ̄`ー- 、_
        , '´  _. -‐'''"二ニニ=-`ヽ、
      /   /:::::; -‐''"        `ーノ
     /   /:::::/           \
     /    /::::::/          | | |  |
     |   |:::::/ /     |  | | | |  |
      |   |::/ / / |  | ||  | | ,ハ .| ,ハ|
      |   |/ / / /| ,ハノ| /|ノレ,ニ|ル' 
     |   |  | / / レ',二、レ′ ,ィイ|゙/   私は只の数ヲタなんかとは付き合わないわ。
.     |   \ ∠イ  ,イイ|    ,`-' |      頭が良くて数学が出来てかっこいい人。それが必要条件よ。
     |     l^,人|  ` `-'     ゝ  |        さらに Ann.of Math に論文書けば十分条件にもなるわよ。
      |      ` -'\       ー'  人          一番嫌いなのは論文数を増やすためにくだらない論文を書いて
    |        /(l     __/  ヽ、           良い論文の出版を遅らせるお馬鹿な人。
     |       (:::::`‐-、__  |::::`、     ヒニニヽ、         あなたの論文が Ann of Math に accept される確率は?
    |      / `‐-、::::::::::`‐-、::::\   /,ニニ、\            それとも最近は Inv. Math. の方が上かしら?
   |      |::::::::::::::::::|` -、:::::::,ヘ ̄|'、  ヒニ二、 \
.   |      /::::::::::::::::::|::::::::\/:::O`、::\   | '、   \
   |      /:::::::::::::::::::/:::::::::::::::::::::::::::::'、::::\ノ  ヽ、  |
  |      |:::::/:::::::::/:::::::::::::::::::::::::::::::::::'、',::::'、  /:\__/‐、
  |      |/:::::::::::/::::::::::::::::::::::::::::::::::O::| '、::| く::::::::::::: ̄|
   |     /_..-'´ ̄`ー-、:::::::::::::::::::::::::::::::::::|/:/`‐'::\;;;;;;;_|
   |    |/::::::::::::::::::::::\:::::::::::::::::::::::::::::|::/::::|::::/:::::::::::/
    |   /:::::::::::::::::::::::::::::::::|:::::::::::::::::::::O::|::|::::::|:::::::::::::::/
897132人目の素数さん:2012/05/01(火) 06:26:01.40
問題出題!

x, y, c, Mは自然数ね

| 3^x - 2^y | > c
任意のcが与えられたときに、十分大きいMを用意して
x > M, y > Mとすれば
これが常に満たされるようにできるかどうか
898132人目の素数さん:2012/05/01(火) 12:57:24.54
できるわけねー
899132人目の素数さん:2012/05/01(火) 16:38:38.86
>>897
できることはほとんど明らか。
ax^3+by^3=c という不定方程式を考えると
特殊な場合を除いて整数解を有限個しか持たないので。
900132人目の素数さん:2012/05/14(月) 23:26:29.61
n 自然数とする。

与えられた整数 a_1, a_2, a_3, a_4, a_5, a_6, a_7, a_8, a_9 が

a_1*a_2*a_3 + a_4*a_5*a_6 + a_7*a_8*a_9 ≡ 1 (mod. n)

を満たすなら、

b_1*b_2*b_3 + b_4*b_5*b_6 + b_7*b_8*b_9 = 1,

a_i ≡ b_i (mod. n), i = 1, 2, ....... , 9

なる整数 b_1, b_2, b_3, b_4, b_5, b_6, b_7, b_8, b_9 が存在する事を示せ。
901132人目の素数さん:2012/05/20(日) 21:02:31.20
902132人目の素数さん:2012/05/20(日) 21:04:49.61
903900:2012/05/25(金) 15:09:47.75
誰も解けないのかね、この問題。難しくは無いのだが。
見た事の無い問題にはお手上げなんだな、このスレの住人よ。
904132人目の素数さん:2012/06/10(日) 08:46:27.70
          __ノ)-'´ ̄ ̄`ー- 、_
        , '´  _. -‐'''"二ニニ=-`ヽ、
      /   /:::::; -‐''"        `ーノ
     /   /:::::/           \
     /    /::::::/          | | |  |
     |   |:::::/ /     |  | | | |  |
      |   |::/ / / |  | ||  | | ,ハ .| ,ハ|
      |   |/ / / /| ,ハノ| /|ノレ,ニ|ル' 
     |   |  | / / レ',二、レ′ ,ィイ|゙/   私は只の数ヲタなんかとは付き合わないわ。
.     |   \ ∠イ  ,イイ|    ,`-' |      頭が良くて数学が出来てかっこいい人。それが必要条件よ。
     |     l^,人|  ` `-'     ゝ  |        さらに Ann.of Math に論文書けば十分条件にもなるわよ。
      |      ` -'\       ー'  人          一番嫌いなのは論文数を増やすためにくだらない論文を書いて
    |        /(l     __/  ヽ、           良い論文の出版を遅らせるお馬鹿な人。
     |       (:::::`‐-、__  |::::`、     ヒニニヽ、         あなたの論文が Ann of Math に accept される確率は?
    |      / `‐-、::::::::::`‐-、::::\   /,ニニ、\            それとも最近は Inv. Math. の方が上かしら?
   |      |::::::::::::::::::|` -、:::::::,ヘ ̄|'、  ヒニ二、 \
.   |      /::::::::::::::::::|::::::::\/:::O`、::\   | '、   \
   |      /:::::::::::::::::::/:::::::::::::::::::::::::::::'、::::\ノ  ヽ、  |
  |      |:::::/:::::::::/:::::::::::::::::::::::::::::::::::'、',::::'、  /:\__/‐、
  |      |/:::::::::::/::::::::::::::::::::::::::::::::::O::| '、::| く::::::::::::: ̄|
   |     /_..-'´ ̄`ー-、:::::::::::::::::::::::::::::::::::|/:/`‐'::\;;;;;;;_|
   |    |/::::::::::::::::::::::\:::::::::::::::::::::::::::::|::/::::|::::/:::::::::::/
    |   /:::::::::::::::::::::::::::::::::|:::::::::::::::::::::O::|::|::::::|:::::::::::::::/
905132人目の素数さん:2012/06/10(日) 11:13:26.89
          __ノ)-'´ ̄ ̄`ー- 、_
        , '´  _. -‐'''"二ニニ=-`ヽ、
      /   /:::::; -‐''"        `ーノ
     /   /:::::/           \
     /    /::::::/          | | |  |
     |   |:::::/ /     |  | | | |  |
      |   |::/ / / |  | ||  | | ,ハ .| ,ハ|
      |   |/ / / /| ,ハノ| /|ノレ,ニ|ル' 
     |   |  | / / レ',二、レ′ ,ィイ|゙/   私は只の数ヲタなんかとは付き合わないわ。
.     |   \ ∠イ  ,イイ|    ,`-' |      頭が良くて数学が出来てかっこいい人。それが必要条件よ。
     |     l^,人|  ` `-'     ゝ  |        さらに Ann.of Math に論文書けば十分条件にもなるわよ。
      |      ` -'\       ー'  人          一番嫌いなのは論文数を増やすためにくだらない論文を書いて
    |        /(l     __/  ヽ、           良い論文の出版を遅らせるお馬鹿な人。
     |       (:::::`‐-、__  |::::`、     ヒニニヽ、         あなたの論文が Ann of Math に accept される確率は?
    |      / `‐-、::::::::::`‐-、::::\   /,ニニ、\            それとも最近は Inv. Math. の方が上かしら?
   |      |::::::::::::::::::|` -、:::::::,ヘ ̄|'、  ヒニ二、 \
.   |      /::::::::::::::::::|::::::::\/:::O`、::\   | '、   \
   |      /:::::::::::::::::::/:::::::::::::::::::::::::::::'、::::\ノ  ヽ、  |
  |      |:::::/:::::::::/:::::::::::::::::::::::::::::::::::'、',::::'、  /:\__/‐、
  |      |/:::::::::::/::::::::::::::::::::::::::::::::::O::| '、::| く::::::::::::: ̄|
   |     /_..-'´ ̄`ー-、:::::::::::::::::::::::::::::::::::|/:/`‐'::\;;;;;;;_|
   |    |/::::::::::::::::::::::\:::::::::::::::::::::::::::::|::/::::|::::/:::::::::::/
    |   /:::::::::::::::::::::::::::::::::|:::::::::::::::::::::O::|::|::::::|:::::::::::::::/
906132人目の素数さん:2012/06/12(火) 07:15:03.62
2
907132人目の素数さん:2012/06/12(火) 08:31:26.73
          __ノ)-'´ ̄ ̄`ー- 、_
        , '´  _. -‐'''"二ニニ=-`ヽ、
      /   /:::::; -‐''"        `ーノ
     /   /:::::/           \
     /    /::::::/          | | |  |
     |   |:::::/ /     |  | | | |  |
      |   |::/ / / |  | ||  | | ,ハ .| ,ハ|
      |   |/ / / /| ,ハノ| /|ノレ,ニ|ル' 
     |   |  | / / レ',二、レ′ ,ィイ|゙/   私は只の数ヲタなんかとは付き合わないわ。
.     |   \ ∠イ  ,イイ|    ,`-' |      頭が良くて数学が出来てかっこいい人。それが必要条件よ。
     |     l^,人|  ` `-'     ゝ  |        さらに Ann.of Math に論文書けば十分条件にもなるわよ。
      |      ` -'\       ー'  人          一番嫌いなのは論文数を増やすためにくだらない論文を書いて
    |        /(l     __/  ヽ、           良い論文の出版を遅らせるお馬鹿な人。
     |       (:::::`‐-、__  |::::`、     ヒニニヽ、         あなたの論文が Ann of Math に accept される確率は?
    |      / `‐-、::::::::::`‐-、::::\   /,ニニ、\            それとも最近は Inv. Math. の方が上かしら?
   |      |::::::::::::::::::|` -、:::::::,ヘ ̄|'、  ヒニ二、 \
.   |      /::::::::::::::::::|::::::::\/:::O`、::\   | '、   \
   |      /:::::::::::::::::::/:::::::::::::::::::::::::::::'、::::\ノ  ヽ、  |
  |      |:::::/:::::::::/:::::::::::::::::::::::::::::::::::'、',::::'、  /:\__/‐、
  |      |/:::::::::::/::::::::::::::::::::::::::::::::::O::| '、::| く::::::::::::: ̄|
   |     /_..-'´ ̄`ー-、:::::::::::::::::::::::::::::::::::|/:/`‐'::\;;;;;;;_|
   |    |/::::::::::::::::::::::\:::::::::::::::::::::::::::::|::/::::|::::/:::::::::::/
    |   /:::::::::::::::::::::::::::::::::|:::::::::::::::::::::O::|::|::::::|:::::::::::::::/
908132人目の素数さん:2012/06/12(火) 12:05:00.31
          __ノ)-'´ ̄ ̄`ー- 、_
        , '´  _. -‐'''"二ニニ=-`ヽ、
      /   /:::::; -‐''"        `ーノ
     /   /:::::/           \
     /    /::::::/          | | |  |
     |   |:::::/ /     |  | | | |  |
      |   |::/ / / |  | ||  | | ,ハ .| ,ハ|
      |   |/ / / /| ,ハノ| /|ノレ,ニ|ル' 
     |   |  | / / レ',二、レ′ ,ィイ|゙/   私は只の数ヲタなんかとは付き合わないわ。
.     |   \ ∠イ  ,イイ|    ,`-' |      頭が良くて数学が出来てかっこいい人。それが必要条件よ。
     |     l^,人|  ` `-'     ゝ  |        さらに Ann.of Math に論文書けば十分条件にもなるわよ。
      |      ` -'\       ー'  人          一番嫌いなのは論文数を増やすためにくだらない論文を書いて
    |        /(l     __/  ヽ、           良い論文の出版を遅らせるお馬鹿な人。
     |       (:::::`‐-、__  |::::`、     ヒニニヽ、         あなたの論文が Ann of Math に accept される確率は?
    |      / `‐-、::::::::::`‐-、::::\   /,ニニ、\            それとも最近は Inv. Math. の方が上かしら?
   |      |::::::::::::::::::|` -、:::::::,ヘ ̄|'、  ヒニ二、 \
.   |      /::::::::::::::::::|::::::::\/:::O`、::\   | '、   \
   |      /:::::::::::::::::::/:::::::::::::::::::::::::::::'、::::\ノ  ヽ、  |
  |      |:::::/:::::::::/:::::::::::::::::::::::::::::::::::'、',::::'、  /:\__/‐、
  |      |/:::::::::::/::::::::::::::::::::::::::::::::::O::| '、::| く::::::::::::: ̄|
   |     /_..-'´ ̄`ー-、:::::::::::::::::::::::::::::::::::|/:/`‐'::\;;;;;;;_|
   |    |/::::::::::::::::::::::\:::::::::::::::::::::::::::::|::/::::|::::/:::::::::::/
    |   /:::::::::::::::::::::::::::::::::|:::::::::::::::::::::O::|::|::::::|:::::::::::::::/
909132人目の素数さん:2012/06/16(土) 19:58:16.72
6
910132人目の素数さん:2012/06/19(火) 22:08:26.12
          __ノ)-'´ ̄ ̄`ー- 、_
        , '´  _. -‐'''"二ニニ=-`ヽ、
      /   /:::::; -‐''"        `ーノ
     /   /:::::/           \
     /    /::::::/          | | |  |
     |   |:::::/ /     |  | | | |  |
      |   |::/ / / |  | ||  | | ,ハ .| ,ハ|
      |   |/ / / /| ,ハノ| /|ノレ,ニ|ル' 
     |   |  | / / レ',二、レ′ ,ィイ|゙/   私は只の数ヲタなんかとは付き合わないわ。
.     |   \ ∠イ  ,イイ|    ,`-' |      頭が良くて数学が出来てかっこいい人。それが必要条件よ。
     |     l^,人|  ` `-'     ゝ  |        さらに Ann.of Math に論文書けば十分条件にもなるわよ。
      |      ` -'\       ー'  人          一番嫌いなのは論文数を増やすためにくだらない論文を書いて
    |        /(l     __/  ヽ、           良い論文の出版を遅らせるお馬鹿な人。
     |       (:::::`‐-、__  |::::`、     ヒニニヽ、         あなたの論文が Ann of Math に accept される確率は?
    |      / `‐-、::::::::::`‐-、::::\   /,ニニ、\            それとも最近は Inv. Math. の方が上かしら?
   |      |::::::::::::::::::|` -、:::::::,ヘ ̄|'、  ヒニ二、 \
.   |      /::::::::::::::::::|::::::::\/:::O`、::\   | '、   \
   |      /:::::::::::::::::::/:::::::::::::::::::::::::::::'、::::\ノ  ヽ、  |
  |      |:::::/:::::::::/:::::::::::::::::::::::::::::::::::'、',::::'、  /:\__/‐、
  |      |/:::::::::::/::::::::::::::::::::::::::::::::::O::| '、::| く::::::::::::: ̄|
   |     /_..-'´ ̄`ー-、:::::::::::::::::::::::::::::::::::|/:/`‐'::\;;;;;;;_|
   |    |/::::::::::::::::::::::\:::::::::::::::::::::::::::::|::/::::|::::/:::::::::::/
    |   /:::::::::::::::::::::::::::::::::|:::::::::::::::::::::O::|::|::::::|:::::::::::::::/
911132人目の素数さん:2012/06/26(火) 06:00:46.75
          __ノ)-'´ ̄ ̄`ー- 、_
        , '´  _. -‐'''"二ニニ=-`ヽ、
      /   /:::::; -‐''"        `ーノ
     /   /:::::/           \
     /    /::::::/          | | |  |
     |   |:::::/ /     |  | | | |  |
      |   |::/ / / |  | ||  | | ,ハ .| ,ハ|
      |   |/ / / /| ,ハノ| /|ノレ,ニ|ル' 
     |   |  | / / レ',二、レ′ ,ィイ|゙/   私は只の数ヲタなんかとは付き合わないわ。
.     |   \ ∠イ  ,イイ|    ,`-' |      頭が良くて数学が出来てかっこいい人。それが必要条件よ。
     |     l^,人|  ` `-'     ゝ  |        さらに Ann.of Math に論文書けば十分条件にもなるわよ。
      |      ` -'\       ー'  人          一番嫌いなのは論文数を増やすためにくだらない論文を書いて
    |        /(l     __/  ヽ、           良い論文の出版を遅らせるお馬鹿な人。
     |       (:::::`‐-、__  |::::`、     ヒニニヽ、         あなたの論文が Ann of Math に accept される確率は?
    |      / `‐-、::::::::::`‐-、::::\   /,ニニ、\            それとも最近は Inv. Math. の方が上かしら?
   |      |::::::::::::::::::|` -、:::::::,ヘ ̄|'、  ヒニ二、 \
.   |      /::::::::::::::::::|::::::::\/:::O`、::\   | '、   \
   |      /:::::::::::::::::::/:::::::::::::::::::::::::::::'、::::\ノ  ヽ、  |
  |      |:::::/:::::::::/:::::::::::::::::::::::::::::::::::'、',::::'、  /:\__/‐、
  |      |/:::::::::::/::::::::::::::::::::::::::::::::::O::| '、::| く::::::::::::: ̄|
   |     /_..-'´ ̄`ー-、:::::::::::::::::::::::::::::::::::|/:/`‐'::\;;;;;;;_|
   |    |/::::::::::::::::::::::\:::::::::::::::::::::::::::::|::/::::|::::/:::::::::::/
    |   /:::::::::::::::::::::::::::::::::|:::::::::::::::::::::O::|::|::::::|:::::::::::::::/
912132人目の素数さん:2012/06/29(金) 21:33:10.58
ではちょいと難しいのを。

p を奇素数、 q を p^p - 1 の最大の素因数とする。この時、

(x - 1)(x - p)(x - p^2) ........... (x - p^(p-1)) ≡ x^p - 1 (mod. q)

を示せ。
913132人目の素数さん:2012/07/01(日) 15:58:25.87
AoPS も最近暇だなー
914132人目の素数さん:2012/07/05(木) 18:13:13.23
>>912
qは(p^p -1)/(p-1)の素因数とする。ord_q(p)=pだから
1,p,p^2,…,p^(p-1)はmod qですべて異なる。
(ちなみにord_q(p)とはmod qでのpの位数の意味)

位数qの有限体F_q上の多項式f(x)を以下のようにとる。
f(x)=(x-1)(x-p)…{x-p^(p-1)} - (x^p -1)
f(x)はp個の値1,p,…,p^(p-1)に対して0となること、そしてf(x)の次数が高々p-1であることを考慮すると
f(x)の係数はすべて0にならざるを得ない。
915132人目の素数さん:2012/07/07(土) 21:01:33.51
>>914
>1,p,p^2,…,p^(p-1)はmod qですべて異なる。
なぜ?
>ord_q(p)=pだから
How?
916132人目の素数さん:2012/07/08(日) 00:14:14.73
>>914
勉強不足、初等整数論を根本的に勉強し直す事を強く勧める。
917132人目の素数さん:2012/07/10(火) 01:01:25.96
>>914
なるほど、分かった。
918132人目の素数さん:2012/07/22(日) 17:42:06.15
          __ノ)-'´ ̄ ̄`ー- 、_
        , '´  _. -‐'''"二ニニ=-`ヽ、
      /   /:::::; -‐''"        `ーノ
     /   /:::::/           \
     /    /::::::/          | | |  |
     |   |:::::/ /     |  | | | |  |
      |   |::/ / / |  | ||  | | ,ハ .| ,ハ|
      |   |/ / / /| ,ハノ| /|ノレ,ニ|ル' 
     |   |  | / / レ',二、レ′ ,ィイ|゙/   私は只の数ヲタなんかとは付き合わないわ。
.     |   \ ∠イ  ,イイ|    ,`-' |      頭が良くて数学が出来てかっこいい人。それが必要条件よ。
     |     l^,人|  ` `-'     ゝ  |        さらに Ann.of Math に論文書けば十分条件にもなるわよ。
      |      ` -'\       ー'  人          一番嫌いなのは論文数を増やすためにくだらない論文を書いて
    |        /(l     __/  ヽ、           良い論文の出版を遅らせるお馬鹿な人。
     |       (:::::`‐-、__  |::::`、     ヒニニヽ、         あなたの論文が Ann of Math に accept される確率は?
    |      / `‐-、::::::::::`‐-、::::\   /,ニニ、\            それとも最近は Inv. Math. の方が上かしら?
   |      |::::::::::::::::::|` -、:::::::,ヘ ̄|'、  ヒニ二、 \
.   |      /::::::::::::::::::|::::::::\/:::O`、::\   | '、   \
   |      /:::::::::::::::::::/:::::::::::::::::::::::::::::'、::::\ノ  ヽ、  |
  |      |:::::/:::::::::/:::::::::::::::::::::::::::::::::::'、',::::'、  /:\__/‐、
  |      |/:::::::::::/::::::::::::::::::::::::::::::::::O::| '、::| く::::::::::::: ̄|
   |     /_..-'´ ̄`ー-、:::::::::::::::::::::::::::::::::::|/:/`‐'::\;;;;;;;_|
   |    |/::::::::::::::::::::::\:::::::::::::::::::::::::::::|::/::::|::::/:::::::::::/
    |   /:::::::::::::::::::::::::::::::::|:::::::::::::::::::::O::|::|::::::|:::::::::::::::/
919132人目の素数さん:2012/09/09(日) 17:36:19.69
          __ノ)-'´ ̄ ̄`ー- 、_
        , '´  _. -‐'''"二ニニ=-`ヽ、
      /   /:::::; -‐''"        `ーノ
     /   /:::::/           \
     /    /::::::/          | | |  |
     |   |:::::/ /     |  | | | |  |
      |   |::/ / / |  | ||  | | ,ハ .| ,ハ|
      |   |/ / / /| ,ハノ| /|ノレ,ニ|ル' 
     |   |  | / / レ',二、レ′ ,ィイ|゙/   私は只の数ヲタなんかとは付き合わないわ。
.     |   \ ∠イ  ,イイ|    ,`-' |      頭が良くて数学が出来てかっこいい人。それが必要条件よ。
     |     l^,人|  ` `-'     ゝ  |        さらに Ann.of Math に論文書けば十分条件にもなるわよ。
      |      ` -'\       ー'  人          一番嫌いなのは論文数を増やすためにくだらない論文を書いて
    |        /(l     __/  ヽ、           良い論文の出版を遅らせるお馬鹿な人。
     |       (:::::`‐-、__  |::::`、     ヒニニヽ、         あなたの論文が Ann of Math に accept される確率は?
    |      / `‐-、::::::::::`‐-、::::\   /,ニニ、\            それとも最近は Inv. Math. の方が上かしら?
   |      |::::::::::::::::::|` -、:::::::,ヘ ̄|'、  ヒニ二、 \
.   |      /::::::::::::::::::|::::::::\/:::O`、::\   | '、   \
   |      /:::::::::::::::::::/:::::::::::::::::::::::::::::'、::::\ノ  ヽ、  |
  |      |:::::/:::::::::/:::::::::::::::::::::::::::::::::::'、',::::'、  /:\__/‐、
  |      |/:::::::::::/::::::::::::::::::::::::::::::::::O::| '、::| く::::::::::::: ̄|
   |     /_..-'´ ̄`ー-、:::::::::::::::::::::::::::::::::::|/:/`‐'::\;;;;;;;_|
   |    |/::::::::::::::::::::::\:::::::::::::::::::::::::::::|::/::::|::::/:::::::::::/
    |   /:::::::::::::::::::::::::::::::::|:::::::::::::::::::::O::|::|::::::|:::::::::::::::/
920132人目の素数さん:2012/10/08(月) 08:05:03.65
          __ノ)-'´ ̄ ̄`ー- 、_
        , '´  _. -‐'''"二ニニ=-`ヽ、
      /   /:::::; -‐''"        `ーノ
     /   /:::::/           \
     /    /::::::/          | | |  |
     |   |:::::/ /     |  | | | |  |
      |   |::/ / / |  | ||  | | ,ハ .| ,ハ|
      |   |/ / / /| ,ハノ| /|ノレ,ニ|ル' 
     |   |  | / / レ',二、レ′ ,ィイ|゙/   私は只の数ヲタなんかとは付き合わないわ。
.     |   \ ∠イ  ,イイ|    ,`-' |      頭が良くて数学が出来てかっこいい人。それが必要条件よ。
     |     l^,人|  ` `-'     ゝ  |        さらに Ann.of Math に論文書けば十分条件にもなるわよ。
      |      ` -'\       ー'  人          一番嫌いなのは論文数を増やすためにくだらない論文を書いて
    |        /(l     __/  ヽ、           良い論文の出版を遅らせるお馬鹿な人。
     |       (:::::`‐-、__  |::::`、     ヒニニヽ、         あなたの論文が Ann of Math に accept される確率は?
    |      / `‐-、::::::::::`‐-、::::\   /,ニニ、\            それとも最近は Inv. Math. の方が上かしら?
   |      |::::::::::::::::::|` -、:::::::,ヘ ̄|'、  ヒニ二、 \
.   |      /::::::::::::::::::|::::::::\/:::O`、::\   | '、   \
   |      /:::::::::::::::::::/:::::::::::::::::::::::::::::'、::::\ノ  ヽ、  |
  |      |:::::/:::::::::/:::::::::::::::::::::::::::::::::::'、',::::'、  /:\__/‐、
  |      |/:::::::::::/::::::::::::::::::::::::::::::::::O::| '、::| く::::::::::::: ̄|
   |     /_..-'´ ̄`ー-、:::::::::::::::::::::::::::::::::::|/:/`‐'::\;;;;;;;_|
   |    |/::::::::::::::::::::::\:::::::::::::::::::::::::::::|::/::::|::::/:::::::::::/
    |   /:::::::::::::::::::::::::::::::::|:::::::::::::::::::::O::|::|::::::|:::::::::::::::/
921132人目の素数さん:2012/10/14(日) 09:14:35.82
          __ノ)-'´ ̄ ̄`ー- 、_
        , '´  _. -‐'''"二ニニ=-`ヽ、
      /   /:::::; -‐''"        `ーノ
     /   /:::::/           \
     /    /::::::/          | | |  |
     |   |:::::/ /     |  | | | |  |
      |   |::/ / / |  | ||  | | ,ハ .| ,ハ|
      |   |/ / / /| ,ハノ| /|ノレ,ニ|ル' 
     |   |  | / / レ',二、レ′ ,ィイ|゙/   私は只の数ヲタなんかとは付き合わないわ。
.     |   \ ∠イ  ,イイ|    ,`-' |      頭が良くて数学が出来てかっこいい人。それが必要条件よ。
     |     l^,人|  ` `-'     ゝ  |        さらに Ann.of Math に論文書けば十分条件にもなるわよ。
      |      ` -'\       ー'  人          一番嫌いなのは論文数を増やすためにくだらない論文を書いて
    |        /(l     __/  ヽ、           良い論文の出版を遅らせるお馬鹿な人。
     |       (:::::`‐-、__  |::::`、     ヒニニヽ、         あなたの論文が Ann of Math に accept される確率は?
    |      / `‐-、::::::::::`‐-、::::\   /,ニニ、\            それとも最近は Inv. Math. の方が上かしら?
   |      |::::::::::::::::::|` -、:::::::,ヘ ̄|'、  ヒニ二、 \
.   |      /::::::::::::::::::|::::::::\/:::O`、::\   | '、   \
   |      /:::::::::::::::::::/:::::::::::::::::::::::::::::'、::::\ノ  ヽ、  |
  |      |:::::/:::::::::/:::::::::::::::::::::::::::::::::::'、',::::'、  /:\__/‐、
  |      |/:::::::::::/::::::::::::::::::::::::::::::::::O::| '、::| く::::::::::::: ̄|
   |     /_..-'´ ̄`ー-、:::::::::::::::::::::::::::::::::::|/:/`‐'::\;;;;;;;_|
   |    |/::::::::::::::::::::::\:::::::::::::::::::::::::::::|::/::::|::::/:::::::::::/
    |   /:::::::::::::::::::::::::::::::::|:::::::::::::::::::::O::|::|::::::|:::::::::::::::/
922132人目の素数さん:2012/10/14(日) 09:45:47.51
スターリングの公式を証明しなさい
923132人目の素数さん:2012/10/19(金) 14:00:42.41
$(x - 1)(x + 11)(x - 11^2)(x + 11^3)(x - 11^4)(x + 11^5)(x - 11^6)(x + 11^7)(x - 11^8)(x + 11^9)(x - 11^{10}) \equiv x^{11} - 1 \mod p $.
なる素数 p を全て求めよ。
924132人目の素数さん:2012/11/13(火) 12:07:41.53
          __ノ)-'´ ̄ ̄`ー- 、_
        , '´  _. -‐'''"二ニニ=-`ヽ、
      /   /:::::; -‐''"        `ーノ
     /   /:::::/           \
     /    /::::::/          | | |  |
     |   |:::::/ /     |  | | | |  |
      |   |::/ / / |  | ||  | | ,ハ .| ,ハ|
      |   |/ / / /| ,ハノ| /|ノレ,ニ|ル' 
     |   |  | / / レ',二、レ′ ,ィイ|゙/   私は只の数ヲタなんかとは付き合わないわ。
.     |   \ ∠イ  ,イイ|    ,`-' |      頭が良くて数学が出来てかっこいい人。それが必要条件よ。
     |     l^,人|  ` `-'     ゝ  |        さらに Ann.of Math に論文書けば十分条件にもなるわよ。
      |      ` -'\       ー'  人          一番嫌いなのは論文数を増やすためにくだらない論文を書いて
    |        /(l     __/  ヽ、           良い論文の出版を遅らせるお馬鹿な人。
     |       (:::::`‐-、__  |::::`、     ヒニニヽ、         あなたの論文が Ann of Math に accept される確率は?
    |      / `‐-、::::::::::`‐-、::::\   /,ニニ、\            それとも最近は Inv. Math. の方が上かしら?
   |      |::::::::::::::::::|` -、:::::::,ヘ ̄|'、  ヒニ二、 \
.   |      /::::::::::::::::::|::::::::\/:::O`、::\   | '、   \
   |      /:::::::::::::::::::/:::::::::::::::::::::::::::::'、::::\ノ  ヽ、  |
  |      |:::::/:::::::::/:::::::::::::::::::::::::::::::::::'、',::::'、  /:\__/‐、
  |      |/:::::::::::/::::::::::::::::::::::::::::::::::O::| '、::| く::::::::::::: ̄|
   |     /_..-'´ ̄`ー-、:::::::::::::::::::::::::::::::::::|/:/`‐'::\;;;;;;;_|
   |    |/::::::::::::::::::::::\:::::::::::::::::::::::::::::|::/::::|::::/:::::::::::/
    |   /:::::::::::::::::::::::::::::::::|:::::::::::::::::::::O::|::|::::::|:::::::::::::::/
925令嬢:2012/12/15(土) 20:55:40.37
          __ノ)-'´ ̄ ̄`ー- 、_
        , '´  _. -‐'''"二ニニ=-`ヽ、
      /   /:::::; -‐''"        `ーノ
     /   /:::::/           \
     /    /::::::/          | | |  |
     |   |:::::/ /     |  | | | |  |
      |   |::/ / / |  | ||  | | ,ハ .| ,ハ|
      |   |/ / / /| ,ハノ| /|ノレ,ニ|ル' 
     |   |  | / / レ',二、レ′ ,ィイ|゙/   
.     |   \ ∠イ  ,イイ|    ,`-' |      
     |     l^,人|  ` `-'     ゝ  |        このスレは馬と鹿と豚さんばかりね。
      |      ` -'\       ー'  人            
    |        /(l     __/  ヽ、          
     |       (:::::`‐-、__  |::::`、     ヒニニヽ、         
    |      / `‐-、::::::::::`‐-、::::\   /,ニニ、\            
   |      |::::::::::::::::::|` -、:::::::,ヘ ̄|'、  ヒニ二、 \
.   |      /::::::::::::::::::|::::::::\/:::O`、::\   | '、   \
   |      /:::::::::::::::::::/:::::::::::::::::::::::::::::'、::::\ノ  ヽ、  |
  |      |:::::/:::::::::/:::::::::::::::::::::::::::::::::::'、',::::'、  /:\__/‐、
  |      |/:::::::::::/::::::::::::::::::::::::::::::::::O::| '、::| く::::::::::::: ̄|
926令嬢:2012/12/16(日) 13:36:04.88
          __ノ)-'´ ̄ ̄`ー- 、_
        , '´  _. -‐'''"二ニニ=-`ヽ、
      /   /:::::; -‐''"        `ーノ
     /   /:::::/           \
     /    /::::::/          | | |  |
     |   |:::::/ /     |  | | | |  |
      |   |::/ / / |  | ||  | | ,ハ .| ,ハ|
      |   |/ / / /| ,ハノ| /|ノレ,ニ|ル' 
     |   |  | / / レ',二、レ′ ,ィイ|゙/   
.     |   \ ∠イ  ,イイ|    ,`-' |      
     |     l^,人|  ` `-'     ゝ  |        このスレは馬と鹿と豚さんばかりね。
      |      ` -'\       ー'  人            
    |        /(l     __/  ヽ、          
     |       (:::::`‐-、__  |::::`、     ヒニニヽ、         
    |      / `‐-、::::::::::`‐-、::::\   /,ニニ、\            
   |      |::::::::::::::::::|` -、:::::::,ヘ ̄|'、  ヒニ二、 \
.   |      /::::::::::::::::::|::::::::\/:::O`、::\   | '、   \
   |      /:::::::::::::::::::/:::::::::::::::::::::::::::::'、::::\ノ  ヽ、  |
  |      |:::::/:::::::::/:::::::::::::::::::::::::::::::::::'、',::::'、  /:\__/‐、
  |      |/:::::::::::/::::::::::::::::::::::::::::::::::O::| '、::| く::::::::::::: ̄|
   |     /_..-'´ ̄`ー-、:::::::::::::::::::::::::::::::::::|/:/`‐'::\;;;;;;;_|
   |    |/::::::::::::::::::::::\:::::::::::::::::::::::::::::|::/::::|::::/:::::::::::/
    |   /:::::::::::::::::::::::::::::::::|:::::::::::::::::::::O::|::|::::::|:::::::::::::::/
927132人目の素数さん:2013/01/15(火) 20:36:22.48
          __ノ)-'´ ̄ ̄`ー- 、_
        , '´  _. -‐'''"二ニニ=-`ヽ、
      /   /:::::; -‐''"        `ーノ
     /   /:::::/           \
     /    /::::::/          | | |  |
     |   |:::::/ /     |  | | | |  |
      |   |::/ / / |  | ||  | | ,ハ .| ,ハ|
      |   |/ / / /| ,ハノ| /|ノレ,ニ|ル' 
     |   |  | / / レ',二、レ′ ,ィイ|゙/   
.     |   \ ∠イ  ,イイ|    ,`-' |      
     |     l^,人|  ` `-'     ゝ  |        このスレは馬と鹿と豚さんばかりね。
      |      ` -'\       ー'  人            
    |        /(l     __/  ヽ、          
     |       (:::::`‐-、__  |::::`、     ヒニニヽ、         
    |      / `‐-、::::::::::`‐-、::::\   /,ニニ、\            
   |      |::::::::::::::::::|` -、:::::::,ヘ ̄|'、  ヒニ二、 \
.   |      /::::::::::::::::::|::::::::\/:::O`、::\   | '、   \
   |      /:::::::::::::::::::/:::::::::::::::::::::::::::::'、::::\ノ  ヽ、  |
  |      |:::::/:::::::::/:::::::::::::::::::::::::::::::::::'、',::::'、  /:\__/‐、
  |      |/:::::::::::/::::::::::::::::::::::::::::::::::O::| '、::| く::::::::::::: ̄|
   |     /_..-'´ ̄`ー-、:::::::::::::::::::::::::::::::::::|/:/`‐'::\;;;;;;;_|
   |    |/::::::::::::::::::::::\:::::::::::::::::::::::::::::|::/::::|::::/:::::::::::/
    |   /:::::::::::::::::::::::::::::::::|:::::::::::::::::::::O::|::|::::::|:::::::::::::::/
928132人目の素数さん:2013/01/18(金) 23:45:23.29
ブルーバックスの本に
「3^100(mod.15)を求めよ」っていうネタがあって

「3の累乗、2^1,3^2,3^3,3^4,…
を計算すると、法15に関する剰余は
3,9,12,6,… (この順で循環する)
したがって、100=4×25より
 3^100=(3^4)^25≡6^25≡6 (mod.15) となる」
とあった

俺、6^25≡6 (mod.15) となる理由がわからん
929132人目の素数さん:2013/01/19(土) 00:44:21.90
>>928
6^2≡6 (mod.15) 
6^3≡6・6^2≡6 (mod.15) 
930132人目の素数さん:2013/01/19(土) 01:07:53.53
ありがとうございます

本当にありがとう
931あぼーん:あぼーん
あぼーん
932あぼーん:あぼーん
あぼーん
933あぼーん:あぼーん
あぼーん
934132人目の素数さん:2013/04/18(木) 22:59:40.18
a^2+2^a=b^2を満たす自然数a,bを求めよ
935132人目の素数さん:2013/07/14(日) 15:24:27.11
前から思ってたんだが、
合同式より計算機記号の % を使った方がよっぽどすっきりしないか?
%を剰余とすると一言書いて

6^2 % 15 = 6
6^3 % 15 = 6・6^2 %15 =6

少なくとも考えながら記述するのはこっちの方が書きやすい
936132人目の素数さん:2013/07/14(日) 15:26:31.84
新課程では整数論とか導入されたみたいだが、
今の高校数学カリキュラムが創設されて以来、何十年も、受験ではちょくちょく出題されるのに、
数学科以外だと大学でも整数については履修しないまま。

受験でよく出題されるレベルの整数について知りたいと思い受験参考書調べると
マスターオブ整数とか
http://skredu.mods.jp/seisu/seisuu-gensoku.pdf
http://skredu.mods.jp/seisu/seisuu-kiso.pdf
受験での整数に関する解説本がいろいろあるんだな
こんなのが出版される前の受験生はどーしてた?
これやりこまないとそら難関校の2次試験で出題されるレベルの整数の問題なんか絶対解けない
937132人目の素数さん:2013/07/15(月) 09:28:47.40
>>935
一つの問題の中では同じ法を用いる場合が多いんだから
mod 15として
6^2≡6
6^3≡6*6^2≡6
と書いたほうが短いし楽だと思うんだが
938132人目の素数さん:2013/07/15(月) 11:51:00.77
>>937

式毎に mod 15を記述する必要あるだろ
それとも合同式をひとくくりでmod 15と明記するか?
moduloがいろいろ混在する場合はどーするよ?書いてる方が混乱しそうだ。
そもそも%15 は合同式などという概念がなくとも使える
939132人目の素数さん:2013/07/20(土) 00:55:02.84
>>936
ところが解けたんだな。
940132人目の素数さん:2013/07/20(土) 09:10:11.71
>>939
a,b を互いに素
ax = 1 (mod b)
x∈整数 となる解 x が必ず存在することを証明せよ

こんなの知らずに解けるかよ
941132人目の素数さん:2013/07/23(火) 00:33:22.24
>>940
ユークリッドの互助法くらいはならったでしょ?
後の展開は行列で表現すると楽チンだ、wikipedia に書いといたから参考にしてね
942132人目の素数さん:2013/07/23(火) 10:26:13.44
>>940
>ユークリッドの互助法くらいはならったでしょ?

互助法て、お前は葬儀屋か?wwww
ちなみに、
ユークリッドの互除法なんて少なくともこれまでの高校までのカリキュラムにはない
まぁ、教えてもらえば馬鹿でもない限りすぐに理解できるけど、
入試もユークリッドの互除法前提の問題は出題されないはず。
俺はBerlekamp法の別アルゴリズムとして知られる今井秀樹のユークリッド法で大学、
それも院試合格した後の4回生で初めて知った

難関大学は必ずといっていいほど出題されるが、
出題するなら文科省の方でもカリキュラム導入はきちんとやって、
いくつかの性質覚えさせて問題演習させなきゃ意味ないと思う。
新課程では導入されたらしいからそれでいいと思うけどね。
数学科以外、数論としてやりこむ必要はないが、
大学入試程度に必要な程度の知識は知っていていいと思う。
943132人目の素数さん:2013/07/23(火) 22:52:47.48
>>936
つ赤チャート数T
つモノグラフ整数
944132人目の素数さん:2013/07/26(金) 09:24:27.91
a=0,b=1
945132人目の素数さん:2013/07/26(金) 09:27:56.86
なんで大学入試の問題に文科省の高校カリキュラムがとか言い出すの?
946132人目の素数さん:2013/07/27(土) 22:58:02.05
>>942
あったぞ?
947132人目の素数さん:2013/07/30(火) 03:33:02.67
>>945
大学入試の問題だからこそ文科省の高校課程が絡むのは当然だろ。
それとも、チョンか?それなら日本の高校カリキュラムは関係ない罠 ぎゃはははは
948132人目の素数さん:2013/07/30(火) 11:25:18.73
その考え方がおかしい。
大学は必要とする学生の能力を試すために、自由な内容の出題をして良い。

実際、芸術系の大学などは、明らかに高校カリキュラムとはかけ離れている
課題を与える場合がある。
949132人目の素数さん:2013/07/31(水) 14:21:29.56
>>948
>大学は必要とする学生の能力を試すために、自由な内容の出題をして良い。

おかしいのはお前だ。というかオツムは確かか?
入学試験も資格試験も出題範囲をきっちり決めてその範疇から出題される。
そこから外れるのは反則だし、そんな問題は出題されない。
整数の問題も新たな知識を必要としないという前提で出題される。

東京大学""高等学校段階までの学習で身につけてほしいこと
http://www.u-tokyo.ac.jp/stu03/e01_01_18_j.html
>本学の入学試験においては,高等学校学習指導要領の範囲を超えた数学の知識や技術が要求されることはありません

残念だったな。大学入学試験は指導要領が大前提なのだよ常識の欠如したお前
そんなことも知らずよく大学合格できたな?
950132人目の素数さん:2013/07/31(水) 14:52:40.86
あげ
951132人目の素数さん:2013/08/01(木) 20:52:43.54
芸術系は特別な例外だな。
952132人目の素数さん:2013/08/01(木) 21:06:05.60
東京藝大に数学科をつくればいいな
953132人目の素数さん:2013/08/01(木) 23:07:20.42
結局寝言ほざいてた>>948は逃亡か?
ま、常識のなさがこれほど見事に晒されたんじゃ恥ずかしくて出てこれまいて。
954132人目の素数さん:2013/08/02(金) 02:13:29.91
>>949
それって東京大学がそのように宣言しているということで
すべての大学がそうでなくてはならないという意味ではないんじゃないの?
955132人目の素数さん:2013/08/02(金) 02:14:06.69
「本学の」と断ってるのはそういう意味だろうな
956132人目の素数さん:2013/08/02(金) 13:42:44.08
>>954

そら各種学校、社会人選抜大学は選抜試験に高校の指導要領ははなからムシしてるわなぁ。ゲラゲラ
出題範囲が結果に大きく反映される数学を選抜試験に科す大学で、
文科省の定めた高校指導要領、高校カリキュラムに準拠する旨を謳ってない大学があればそれをさらしてみろや
出題範囲を明記しない選抜試験なんてないんだから、そんな大学があるというなら証拠見せてみせてみな。
957132人目の素数さん:2013/08/02(金) 13:44:26.05
学力試験として 数3とか高校での科目名を断ってんなら高校範囲てのもあるだろうが
なにも芸術系でなくてもそれ以外の試験なんていくらでもやってるだろ。
科目の分類におさまらないテーマの小論文や作文もそうだし
建築で未来都市デザインとか、他にもプレゼンとか
958132人目の素数さん:2013/08/02(金) 13:46:24.82
>>956

「高校カリキュラム範囲からの逸脱」から
「出題範囲を明記しない」に 話題がネジ曲がってんぞ
959132人目の素数さん:2013/08/02(金) 13:51:14.52
>>956
聞きたいんだが、 高校卒業「程度」 てのは
「カリキュラムを一切逸脱しない」という意味なのか?
だとしたら、どの年度に準拠なのかも書かなくてはならないか
過去のカリキュラムすべての共通要素からしか出題できないという意味なのか?
960132人目の素数さん:2013/08/02(金) 14:11:30.18
>>954
>すべての大学がそうでなくてはならないという意味ではないんじゃないの?

ってのはどういう意味だ?文科省の意向に反する選抜試験がまかり通るって意味かい?
お前、文科省が毎年選抜試験要項を大学側に通知してるってのを知ってるのか?

http://www.mext.go.jp/a_menu/koutou/senbatsu/
>第6 学力検査等
>1 個別学力検査
>(1) 各大学が実施する学力検査(以下、「個別学力検査」という。)は、高等学校学習指導要
>領(平成11年文部省告示第58号。以下、「学習指導要領」という。)に準拠し、高等学校教
>育の正常な発展の障害とならないよう十分留意しつつ、適切な方法により実施する。

ちゃーんと指導要領準拠を要請されてるのだよ。常識のかけらもないお前
それでもなお、文科省の意向に反する試験を実施するような度胸のある大学があればさらしてみろや
961132人目の素数さん:2013/08/02(金) 14:20:54.45
>>958

はぁ?文盲だろおまえは
試験は必ず出題範囲を明記してる。
もし、高校カリキュラムの範囲逸脱がなされるなら入試要項にその旨記載されてるんだから、
ネットで探せるだろうと言ってるのだよ低脳のお前
アホは全角英数使うっていうがその通りだなお前
962132人目の素数さん:2013/08/02(金) 14:25:43.37
結局、
>>945
>>948
>>954

あたりは己の無知、常識のなさを世間に晒しただけだったねぇゲラゲラ
も、いっかい大学受験でもやり直せやウスノロ
>>958に至っては文章も理解できないときた。
アホは、所詮こーだろな。常識がないってのはそれなりの理由がある
963132人目の素数さん:2013/08/02(金) 15:04:51.24
>>957
>科目の分類におさまらないテーマの小論文や作文もそうだし
>建築で未来都市デザインとか、他にもプレゼンとか

指導要領が5教科だけとでも思ってるのか?
指導要領っては各教科以外に芸術、保健体育、生活、
普通科学生だけじゃないから当然、商業、農業、水産、工業なんて項目もあるのだよ。
論文や作文はちゃーんと国語の中で盛り込まれてるのと、
プレゼンあたりはほとんどの教科で表現力の育成が謳われてる。
選抜試験項目がおかしいと思うなら選抜方法に関して大学に問い合わせてみな。
必ず指導要領のどの部分を選考基準にしてるかを説明してくるはずだ。

↓こういうアホもいるんだが。
>>948
>実際、芸術系の大学などは、明らかに高校カリキュラムとはかけ離れている
>課題を与える場合がある。

芸術は指導要領のなかにきちんと盛り込まれてる。
試験内容も指導要領に準拠した(少なくとも大学側は
第3者に準拠となる根拠を説明できる)形で出題されるのだよ
常識のないアホ江
964132人目の素数さん:2013/08/02(金) 15:06:56.56
>>959
>聞きたいんだが、 高校卒業「程度」 てのは

どこの大学だ?
965132人目の素数さん:2013/08/23(金) 22:20:24.68
>>940

 S = {a,2a,・・・・,(b-1)a} の要素はいずれもbで割り切れない。
また,ia≡ja (mod b) ⇒ (i-j)a ≡ 0 (mod b) ⇒ i-j≡0 (mod b) ⇒ i-j=0
よって,Sは1〜b-1を1個ずつ含む。
 {a,2a,・・・・,(b-1)a} ≡ {1,2,・・・・,b-1} (mod b)
966132人目の素数さん:2013/08/27(火) 00:58:56.20
>>965
ax=1 mod.b <---> ax=by+1 <--->
ax-by=1ならばaとbの共通因子は1、
じゃあかんの?
967132人目の素数さん:2013/08/27(火) 21:09:35.31
d,nを正整数とする。任意の(2^d)(n-1)+1個のd次元整数ベクトルがあったとき、
その中からn個のベクトルを取り出し、それらの和がすべての要素がnの倍数となるようにできることを示せ。
968132人目の素数さん:2013/08/27(火) 22:40:35.29
>>966

 ax-by=1 <--- aとbの公約数は1のみ.

をどう示すか...
969132人目の素数さん:2013/08/28(水) 00:10:02.68
>>968
書き込みテストで背理法
970132人目の素数さん:2013/08/28(水) 00:11:52.26
ごめんなさい書き込めてしまった。普通は互除法?
971132人目の素数さん:2013/08/29(木) 08:15:18.11
>>965
あのさ。一度でも証明を見れば馬鹿でも解けるが、
高校数学課程においてカケラも載ってもないのに課程準拠が必要な受験で、
出題そのものが問題をはらんでるexampleとして引き合いに出してるのに、
話の流れも理解せず何を間抜けにまんま解いてんの
つーか写してんの
972132人目の素数さん:2013/08/29(木) 14:13:07.32
>971
> あのさ。一度でも証明を見れば馬鹿でも解けるが、

馬鹿に期待しすぎ。
973132人目の素数さん:2013/09/04(水) 02:17:17.78
 S は (Z/nZ) = {0,1,・・・・,n-1} の真部分集合とする。0 < #S < n,

 S の各要素にa(≠0)をたしたものを S~ とおく。 #S~ = #S

〔補題〕
nが素数のとき、S~ ≠ S.

(略証)
 aも#Sもnで割り切れないから、
 Σ_S~ = Σ_S + a・#S
    ≠ Σ_S   (mod n)
974132人目の素数さん:2013/09/04(水) 03:04:35.33
 y_1, y_2, ・・・・・, y_(n-1) はnで割り切れないとする。

 {y_1, ・・・・, y_(n-1)} のすべての部分集合(空集合φも含む)に対し、
要素の和をnで割った余りを考える。

 その全体は (Z/nZ) である。(0〜n-1をすべて含む。)

(略証)
 {y_1, ・・・・, y_k} の部分集合(空集合φも含む)に対し、要素の和をnで割った余りを考える。

 その全体集合を S_k とする。

 S_1 = {0, y_1} ゆえ #S_1 = 2,

 また、補題より、S_k = S_(k-1) ∪ S_(k-1)~ ⊃ S_(k-1),

∴ #S_k > #S_(k-1),

∴ #S_k ≧ k+1,

∴ #S_(n-1) = n,
975132人目の素数さん:2013/09/04(水) 17:10:15.19
>>974 nが素数のとき、だよね
976132人目の素数さん:2013/09/04(水) 22:56:10.90
>>975 thx.

nが素数のとき、
2n-1個の整数から、和がnで割り切れるn個組を選出できる。

(略証)
・nで割った余りが同じものがn個以上あれば、その中のn個を選べばよい。
・nで割った余りが同じものは高々n-1個以下とする。
 余りの順に x_1, x_2, ・・・・・, x_(2n-1) とし、非合同ペア
  {x_1, x_(n+1)}
  {x_2, x_(n+2)}
  ・・・・
  {x_(n-1), x_(2n-1)}
  {x_n}
 から1個づつ選んでn個組とする。
 その和をnで割った余りの集合は (Z/nZ)である。(0〜n-1をすべて含む)
 よって、和がnで割り切れるn個組を選出できる。
977132人目の素数さん:2013/09/05(木) 01:59:33.36
nが自然数のときも、
2n-1個の整数から、和がnで割り切れるn個組を選出できる。

(略証)
nの因数についての帰納法による。
・nが素数のときは成立つ。 >>976
・nが合成数のとき、pを素数として、n=pm と書ける。
 2n-1 個から、和がpで割り切れるp個組を選出し、除去する。
 これを2m-1回繰り返す。
 それらをpで割った商を z_1, z_2, ・・・・・, z_(2m-1) とする。
 この2m-1個の整数から、和がmで割り切れるm個組を選出できる。
よって、2pm-1個の整数から、和がpmで割り切れるpm個組を選出できる。
978132人目の素数さん:2013/09/05(木) 02:12:58.75
〔蛇足〕
2n-2個の整数から、和がnで割り切れるn個組を選出できない例
{a,・・・・,a, a+1,・・・・・,a+1} (各n-1個)
979132人目の素数さん:2013/09/22(日) 02:50:09.08
相異なる1より大きな任意の整数a,bについて、(a^n-1)(b^n-1)が平方数とならないような正整数nが存在することを示せ
980132人目の素数さん:2013/10/24(木) 06:18:20.69
>>945
>なんで大学入試の問題に文科省の高校カリキュラムがとか言い出すの?

>>948
>その考え方がおかしい。
>大学は必要とする学生の能力を試すために、自由な内容の出題をして良い。

>実際、芸術系の大学などは、明らかに高校カリキュラムとはかけ離れている
>課題を与える場合がある。

結局この常識のかけらもない馬鹿ば逃亡か?
981132人目の素数さん:2013/10/24(木) 12:31:21.95
ハミルトンケーリーの定理に関してはこんな話を聞いたことがある(W

N先生:全くもって指導要領の範囲外(大学院生の予備校講師から受験テクニックとして掻い摘んだものが広まったらしい)。

学生:証明できるので使用して良いのでは?

N先生:はぁ?そもそも2次正方行列限定でハミルトンケーリーの定理?何んやそれ?

N先生:というかそんな中途半端なテクニック使わん方が簡単かつ本質的な解答となるのやが。

N先生:全然数学力の評価にならへんし採点せーへんよ!
982132人目の素数さん:2013/10/24(木) 13:38:00.15
はぁ?ケイレイ・ハミルトンは教科書にちゃーんと紹介されてる。
背景は隠して、単なる等式としてだ。
2次の正方行列を前提とするなら、
A^2-(a+d)A+(ad-bc)E=O
は、高校カリキュラムで十分証明可能だ

>>981
アホかお前
983132人目の素数さん:2013/10/24(木) 14:02:02.20
>>982
> 2次の正方行列を前提とするなら、
はぁ?
984132人目の素数さん:2013/10/24(木) 14:04:39.78
>>981
その話を聞いたのはいつ頃のことですか?
985132人目の素数さん:2013/10/24(木) 14:52:41.64
>>984
30年くらい前です。
986132人目の素数さん:2013/10/24(木) 23:29:13.35
二年三百四十二日。
987132人目の素数さん:2013/10/25(金) 23:29:13.53
二年三百四十三日。
988132人目の素数さん:2013/10/26(土) 08:57:48.05
>>983

高校数学における行列の取り扱いで3次以上がカリキュラムに登場したことなんかねぇんだよ馬鹿たれ
したがってハミルトン=ケーリーも2次に限定すれば、簡単に成分計算で証明できるし、
次数下げ公式として使うことに何の問題もないつってんだよアホ
堂々と教科書に記載されてるのに何寝言ほざいてんだって話だ
989132人目の素数さん:2013/10/26(土) 11:12:00.97
直角二等辺三角形四つ繋いで
「直角二等辺三角形の斜辺の長さは直角を挟む辺の長さの√2倍」を証明して
これを三平方の定理と呼ぶようなものか。

三平方の定理=「直角二等辺三角形の斜辺の長さは直角を挟む辺の長さの√2倍」
990132人目の素数さん:2013/10/26(土) 12:14:10.38
次数によらず成り立つから定理なのに、命題や補題とすら呼ぶに値するかどうか
というほど自明なn=2に限定したのならその定理の名前を冠しちゃダメだろ。
991132人目の素数さん:2013/10/26(土) 12:23:56.30
そこまでいくと難癖だな
ハミルトンケーリーの定理が指導要領の範囲外との勘違いを正すという趣旨から逸れまくってるし
992132人目の素数さん:2013/10/26(土) 12:24:50.48
そこまでいくと難癖だな
ハミルトンケーリーの定理(n=2の場合)が指導要領の範囲外との勘違いを正すという趣旨から逸れまくってるし
993132人目の素数さん:2013/10/26(土) 12:55:25.40
お前にとっては、二次方程式が有限操作な有理拡大の範囲に根を持つのがガロワ論理で、4で割ると1余る素数が二つ冪の和で書けるのが類体論な(W
994132人目の素数さん:2013/10/26(土) 13:01:10.54
はいはい
呼び名が争点になっているのではないと理解できないのね
995132人目の素数さん:2013/10/26(土) 13:37:53.69
じゃケーリーハミルトンの定理がどのような対象の上で成立するのか必要十分に説明して下さいや(W
996132人目の素数さん:2013/10/26(土) 13:42:25.35
何言ってるのか全然わからん
997132人目の素数さん:2013/10/26(土) 13:44:42.25
可換環を成分にもつ正方行列一般に成立するのだが、二次の証明とは比較にならんよ。
998994:2013/10/26(土) 13:47:18.83
(だから何やねん…)
999132人目の素数さん:2013/10/26(土) 13:53:43.25
もともとの>>981が二次限定のケーリーハミルトンなんて無いわボケっていうネタの話なのにね
1000132人目の素数さん:2013/10/26(土) 14:21:22.13
当然、それと同様に実数限定でケーリーハミルトンってなんやねんって話だったと思うが?
10011001
このスレッドは1000を超えました。
もう書けないので、新しいスレッドを立ててくださいです。。。